Quiz Questions Flashcards
A 45 year old woman from Jamaica presents with a 6 month history of weight loss and a two week history of fever. On examination she had cervical lymphadenopathy. Her calcium was raised at 3.0 and CXR showed bilateral hilar lymphadenopathy.
A. Post immunisation B. Glandular fever C. Pneumonia D. HIV infection E. Malaria F. Drug reaction G. SLE H. Sarcoidosis I. Appendicitis J. Influenza K. Tuberculosis L. Hodgkin's lympoma M. Gastric carcinoma N. Pyelonephritis
H. Sarcoidosis - Sarcoidosis is a chronic multisystem disease with an unknown aetiology. Lymphadenopathy is a common presentation and nodes are enlarged but non-tender, typically involving the cervical and submandibular nodes. Although uncommon, the patient may present with unexplained modest weight loss (which is often mistaken for TB or lymphoma along with the abnormal CXR) and a low-grade fever. CXR will typically show bilateral hilar lymphadenopathy and CXR findings are used in the staging of disease. Additionally, serum calcium and ACE levels may be raised. A transbronchial biopsy is essential for diagnosis in most cases and shows the presence of non-caseating granulomas. Black people have a higher lifetime risk of sarcoidosis, as do those of Scandinavian origin. The mainstay of treatment for severe disease involves systemic corticosteroids. Skin manifestations include erythema nodosum which are tender erythematous nodules and lupus pernio presenting with indurated plaques with discoloration on the face.
A 25 year old man with a 3 day history of high temperatures, aching limbs and neck discomfort. Apart from temperatures of 39 degrees C and some mild conjunctivitis, examination was normal. Antibiotics were prescribed but had no effect.
A. Post immunisation B. Glandular fever C. Pneumonia D. HIV infection E. Malaria F. Drug reaction G. SLE H. Sarcoidosis I. Appendicitis J. Influenza K. Tuberculosis L. Hodgkin's lympoma M. Gastric carcinoma N. Pyelonephritis
J. Influenza - This is an acute respiratory tract infection caused by seasonal viral influenza A or B, hence antibiotics would have no effect. It is characterised by respiratory symptoms including rhinorrhoea, cough, fever, chills, headache and myalgia. Recent upper respiratory tract infection has led to associated viral conjuncitivitis. The examination in this case is of course otherwise unremarkable. Antigenic change presents a challenge in creating new vaccines as there is no incremental protection from previous vaccinations. There have been 4 pandemics since 1918, the recent one being ‘swine flu’ in April 2009, caused by new gene rearrangement of human, avian and swine influenza.
A 50 year woman normally resident in the UK returned from visiting relatives in Pakistan. She described intermittent fevers with rigors, diarrhoea and severe headaches. She is mildly jaundiced.
A. Post immunisation B. Glandular fever C. Pneumonia D. HIV infection E. Malaria F. Drug reaction G. SLE H. Sarcoidosis I. Appendicitis J. Influenza K. Tuberculosis L. Hodgkin's lympoma M. Gastric carcinoma N. Pyelonephritis
E. Malaria - In the Western world, almost all cases of malaria occurs in travellers so an adequate travel history is crucial or the diagnosis may be missed. Patients typically present with non-specific symptoms such as a fever, sweats, chills and myalgia. This can also include the diarrhoea which this patient is experiencing and headaches. The jaundice here suggests falciparum infection, which is always the cause in severe disease. This woman has just returned from an endemic area. Sometimes EMQs will describe patterns of fevers occuring at regular intervals of 48-72 hours associated with P. vivax, P. ovale and P. malariae infections but in most patients there is no specific pattern. Hepatosplenomegaly is a common presenting sign although not common at presentation in a first world setting. Thrombocytopenia is common with falciparum infection and a mild degree of anaemia ais commonly seen. WCC can be high, low or normal.
The severity of malaria depends partly on the species and also on host immunity. Therefore those who live in endemic areas may develop minimal symptoms due to IgG antibody and cell-mediated immunity and physiological tolerance of parasitaemia. Pregnant women affected by P. falciparum are also susceptible to the complications of pregnancy due to placental parasite sequestration. Treatment of malaria in pregnancy must be managed with an ID specialist and should be treated with IV antimalarial therapy.
The test of choice is Giesma-stained thick and thin blood smears. Thick films sensitively detect parasites whereas thin films allow species identification and calculation of parasitaemia to guide treatment. Studies have shown that for P falciparum, the most effective treatment is artesunate which is more effective than quinine without the risk of cinchonism. Numerous studies such as the AQUAMAT study in The Lancet showing that quinine should no longer be the established treatment of choice.
A 22 year old man presented with a two week history of fever and drenching night sweats. He had experienced severe itching during this time. Examination was normal except for swollen supraclavicular lymph nodes. CXR showed a mediastinal mass.
A. Post immunisation B. Glandular fever C. Pneumonia D. HIV infection E. Malaria F. Drug reaction G. SLE H. Sarcoidosis I. Appendicitis J. Influenza K. Tuberculosis L. Hodgkin's lympoma M. Gastric carcinoma N. Pyelonephritis
L. Hodgkins Lymphoma - This is a case of lymphoma. Reed-Sternberg cells are binucleate cells characteristically seen in Hodgkin’s lymphoma. Hodgkin’s is localised to a single group of nodes (normally the cervical and/or supraclavicular) and extranodal involvement is rare. Mediastinal involvement is common. Spread is contiguous and B symptoms may be present such as a low grade fever, weight loss and night sweats. Pruritis may be found in approximately 10% of cases but has no prognostic significance. 50% of cases is associated with EBV infection and distribution is bimodal with peaks in young and old. There is classically pain in lymph nodes on alcohol consumption.
A 35 year old pregnant woman developed a temperature with chills and increased urinary frequency. She is tender in the right loin and has vomited. Dipstick urinalysis is positive for leukocytes, nitrites and blood.
A. Post immunisation B. Glandular fever C. Pneumonia D. HIV infection E. Malaria F. Drug reaction G. SLE H. Sarcoidosis I. Appendicitis J. Influenza K. Tuberculosis L. Hodgkin's lympoma M. Gastric carcinoma N. Pyelonephritis
N. Pyelonephritis - Acute onset fever with chills, flank pain, vomiting and positive urine dipstick all point to the diagnosis of acute pyelonephritis. Urinalysis is highly sensitive but not very specific. Pregnancy is a risk factor for complicated disease as the enlarging uterus compresses the ureters and hormonal changes increase the likelihood of obstructive uropathy. In uncomplicated pyelonephritis, the most common cause is E. coli and gram stain will typically reveal gram negative rods, either E. coli, Proteus or Klebsiella. Gram positive cocci that could be implicated include enterococci and staphylococci. Older patients can often also present non-specifically. Treatment should start before culture results are received to prevent the patient from deteriorating, with empirical antibiotics.
A 50 year old woman presented with a temperature and aching joints 2 days prior to departure on holiday to Egypt. On examination she had a tender swelling on her left upper arm.
A. Post immunisation B. Glandular fever C. Pneumonia D. HIV infection E. Malaria F. Drug reaction G. SLE H. Sarcoidosis I. Appendicitis J. Influenza K. Tuberculosis L. Hodgkin's lympoma M. Gastric carcinoma N. Pyelonephritis
A. Post immunisation - Travel to Egypt may necessitate vaccinations for diseases such as typhoid and HAV. This is a side-effect of the vaccine and the history here points towards this. Vaccine side effects tend to be uncommon although this depends on the specific vaccine given. For example, some 10-30% of people will experience mild side effects such as muscle pain and headache after being given the yellow fever vaccine.
A 40 year old man returned from India 4 days ago & came down with fever. He went to his GP & got some paracetamol. His fever persisted, & he had a few bouts of diarrhoea & cough. He started getting drowsy & was admitted from casualty where his blood culture was taken. The next day the lab reported Gram negative bacilli seen on blood culture.
A. Mycobacterium tuberculosis B. Legionella pneumophila C. Dengue virus D. Falciparum malaria E. Lassa fever F. Entomoeba histolytica G. Streptococcus pneumoniae H. Salmonella typhi I. Neisseria meningitidis type B J. Influenza
H. Salmonella typhi - Typhoid (or enteric fever) is a faecal-oral illness caused by Salmonella enterica, serotype S typhi, Salmonella enterica and S paratyphi. There are over 2500 serovars for S enterica. In this country, it is mainly from people who have returned from a country where it is endemic – India, for example, which has the highest incidence of this disease. Mexico should also ring alarm bells. The water supplies are not treated and sanitary conditions are dire which prompts transmission of this infection. Humans are the only known reservoir. The vaccine only offers moderate protection and does not protect against paratyphoid infection. This person has a high fever which is a hallmark of infection (sometimes in a step-wise manner) which is not responding to paracetamol (it persists) and blood culture (you need a big sample of blood for testing as count is usually low) shows a gram negative bacilli, and he has returned from an endemic region. The fever of typhoid classically increases incrementally until a persistent fever with temperature 39-41 is established. There are also typically flu like symptoms after onset of fever and cough. Remember that characteristic findings such as bradycardia and rose spots may not be present and indeed rose spots may not be easy to spot in those with dark skin. Rose spots are blanching red lesions reported in 5-30% of cases usually occuring on the chest or abdomen.
This patient needs antibiotics though the temperature will fall over about week. A third generation cephalosporin is indicated due to resistance to fluoroquinolones in the Indian sub-continent. If the sensitivity panel returns and shows that this organism is sensitive to all antibiotics then ciprofloxacin should be given.
50 year old man went for Hajj. He was vaccinated against hepatitis & Group C meningococci. He came back with a high fever & was admitted with neck stiffness & drowsiness. He had a lumbar puncture & blood cultures. Gram stain of cerebrospinal fluid showed Gram negative diplococci.
A. Mycobacterium tuberculosis B. Legionella pneumophila C. Dengue virus D. Falciparum malaria E. Lassa fever F. Entomoeba histolytica G. Streptococcus pneumoniae H. Salmonella typhi I. Neisseria meningitidis type B J. Influenza
I. Neisseria meningitidis type B - This patient has meningitis. A big risk factor is crowding which occurs during Hajj. Commonly there will be a headache, fever and nuchal rigidity. There may also be an altered mental status, confusion, photophobia and vomiting. Kernig’s sign is uncommon but is positive when attempts to extend the leg are met with resistance when the patient is supine with the thigh flexed to 90 degrees. Another uncommon sign is Brudzinski’s sign and a petechial/purpuric rash, typically associated with meningococcal meningitis.
CT head should be considered before LP if there is any evidence of raised ICP. A LP will confirm the diagnosis with bacterial meningitis showing a low CSF glucose, elevated CSF protein and positive CSF culture/gram stain or meningococcal antigen. The CSF gram stain showing a gram negative diplococci here gives the organism away. If you know your gram stains etc you can get this one from just reading the last line.
40 year old social worker, lived in India & the Far East for the last 2 years. He came back with an intermittent fever of 2 months duration. On examination the GP noted tenderness & swelling of the right hypochondrium. He was sent to the hospital where on ultrasound a liver abscess was found.
A. Mycobacterium tuberculosis B. Legionella pneumophila C. Dengue virus D. Falciparum malaria E. Lassa fever F. Entomoeba histolytica G. Streptococcus pneumoniae H. Salmonella typhi I. Neisseria meningitidis type B J. Influenza
F. Entomoeba histolytica - This is amoebiasis caused by E. Histolytica. Again, there is history of travel to an endemic area. Additional risk factors include being male and male-male sex, both oral and anal. Diagnosis would be confirmed by the detection of antigen in stool samples, serology or PCR. As neither of these methods are 100% sensitive, it is normally good to use more than 1 test in diagnosis. The presentation is normally with diarrhoea, without blood or mucus, present for several days or longer, indicating intestinal infection. Half also report weight loss. This patient, has however got the fever more associated with hepatic infection. There may also be jaundice here and hepatomegaly on examination. The RUQ tenderness found here is indicative of hepatic infection and the mass here is likely a hepatic abscess, which itself may extend into the pleural or pericardial cavities in rare cases, causing effusions. Splenic abscesses and even brain abscesses with neurological abnormalities can also rarely occur. Treatment is with nitroimidazoles followed by agents like paromomycin.
30 year old stone mason came from India to work on a temple being constructed. He presented to the GP with history of fever, night sweats & cough of 3 months duration. Chest x-ray showed a cavitating shadow.
A. Mycobacterium tuberculosis B. Legionella pneumophila C. Dengue virus D. Falciparum malaria E. Lassa fever F. Entomoeba histolytica G. Streptococcus pneumoniae H. Salmonella typhi I. Neisseria meningitidis type B J. Influenza
A. Mycobacterium tuberculosis - It is important to have a high level of suspicion when evaluating patients with risk factors who present with suggestive symptoms. Night sweats, fever, malaise, cough, haemoptysis and erythema nodosum are all suggestive. In the first half of the 20th century, tuberculosis accounted for over 90% of cases of erythema nodosum. Other key risk factors for pulmonary TB include exposure to infection and returning from or being born in a high-risk region such as Asia, Africa and Latin America. If TB is suspected, the patient should be placed in isolation and a CXR obtained with 3 sputum samples cultured for AFB being the gold standard of diagnosis. Cavitating lesions like the one this patient has can be seen on CXR but is non-specific for TB. Culture takes several weeks so sputum smears will be done before culture results are known. Interferon-gamma release assays (IGRAs) are now used by some hospitals to rapidly determine a patient’s TB status. All patients who have TB should be tested for HIV within 2 months of diagnosis.
A 22-year-old student went to Thailand on holiday. A week following his return, he presented to his GP with a flu like illness and high fever. His GP presumed it was flu and told him to go home. Two days later, he re-presented to A&E, this time vomiting.
A. Mycobacterium tuberculosis B. Legionella pneumophila C. Dengue virus D. Falciparum malaria E. Lassa fever F. Entomoeba histolytica G. Streptococcus pneumoniae H. Salmonella typhi I. Neisseria meningitidis type B J. Influenza
D. Falciparum malaria - In the Western world, almost all cases of malaria occurs in travellers so an adequate travel history is crucial or the diagnosis may be missed. Patients typically present with non-specific symptoms such as a fever, sweats, chills and myalgia. This student has just returned from an endemic area. Sometimes EMQs will describe patterns of fevers occuring at regular intervals of 48-72 hours associated with P. vivax, P. ovale and P. malariae infections but in most patients there is no specific pattern. Hepatosplenomegaly is a common presenting sign although not common at presentation in a first world setting. Thrombocytopenia is common with falciparum infection and a mild degree of anaemia are commonly seen. WCC can be high, low or normal.
Pregnant women affected by P. falciparum are susceptible to the complications of pregnancy due to placental parasite sequestration. Treatment of malaria in pregnancy must be managed with an ID specialist and should be treated with IV antimalarial therapy.
A 22 year old female medical student returned from elective in Nigeria 3 months ago, she has had a fever & night sweats for 3 weeks.
choose the SINGLE investigation, most likely to confirm the diagnosis, from the above list of options:
A. Abdominal ultrasound B. Echocardiogram C. Urine microscopy & culture D. Thick blood film E. Liver function tests F. Lumbar puncture G. IVP H. Blood cultures I. Full blood count J. Clinical exam only K. CT brain scan L. Chest x-ray & sputum cultures M. Throat swabs
D. Thick blood film - In the Western world, almost all cases of malaria occurs in travellers so an adequate travel history is crucial or the diagnosis may be missed. Patients typically present with non-specific symptoms such as a fever, sweats, chills and myalgia. This medical student has just returned from an endemic area. Sometimes EMQs will describe patterns of fevers occuring at regular intervals of 48-72 hours associated with P. vivax, P. ovale and P. malariae infections but in most patients there is no specific pattern.
The test of choice is Giesma-stained thick and thin blood smears. Thick films sensitively detect parasites whereas thin films allow species identification and calculation of parasitaemia to guide treatment. Studies have shown that for P falciparum, the most effective treatment is artesunate which is more effective than quinine without the risk of cinchonism. However, not all hospitals are currently licensed to use it in the UK (in London, only the Hospital for Tropical Diseases and Northwick Park). Artesunate is manufactured by a pharmaceutical company in China and there are doubts over the quality of the product. However, there have now been numerous studies such as the AQUAMAT study in The Lancet showing that quinine should no longer be the established treatment of choice.
A 23 year old man has been living rough in London since being made homeless 6 months ago. He presents in A&E, unwell with 1 month history of cough, weight loss, fever & night sweats. Choose the SINGLE investigation, most likely to confirm the diagnosis, from the above list of options:
A. Abdominal ultrasound B. Echocardiogram C. Urine microscopy & culture D. Thick blood film E. Liver function tests F. Lumbar puncture G. IVP H. Blood cultures I. Full blood count J. Clinical exam only K. CT brain scan L. Chest x-ray & sputum cultures M. Throat swabs
L. Chest x-ray & sputum cultures - The patient’s symptoms point towards pulmonary TB. CXR is the first line test to order. Classically, in primary disease there are middle and lower zone infiltrates. Post-primary TB usually involves apical changes with or without cavitation. However, recent students have indicated that both presentations are seen in both primary and post-primary TB. HIV positive patients tend to have a more atypical CXR including effusion, lower zone involvement and a miliary pattern. Sputum cultures on LJ medium are the most sensitive and specific test but growth on solid media can take 4-8 weeks. A smear will be done in the meantime to look for AFB but the sensitivity is lower than that of a culture.
An 18 year old student started university 3 months ago. She has felt flu-like for 2 days. In the last 2 hours she has developed a severe headache, vomiting, temp of 390C & photophobia. On examination she has neck stiffness & a positive Kernig’s sign. Choose the SINGLE investigation, most likely to confirm the diagnosis, from the above list of options:
A. Abdominal ultrasound B. Echocardiogram C. Urine microscopy & culture D. Thick blood film E. Liver function tests F. Lumbar puncture G. IVP H. Blood cultures I. Full blood count J. Clinical exam only K. CT brain scan L. Chest x-ray & sputum cultures M. Throat swabs
F. Lumbar puncture - This patient has meningitis. Universities are common sites of outbreaks due to crowding. Commonly there will be a headache, fever and nuchal rigidity. There may also be an altered mental status, confusion, photophobia and vomiting. Kernig’s sign is uncommon but is positive when attempts to extend the leg are met with resistance when the patient is supine with the thigh flexed to 90 degrees. Another uncommon sign is Brudzinski’s sign and a petechial/purpuric rash, typically associated with meningococcal meningitis.
CT head should be considered before LP if there is any evidence of raised ICP. A LP will confirm the diagnosis with bacterial meningitis showing a low CSF glucose, elevated CSF protein and positive CSF culture/gram stain or meningococcal antigen.
5 days after a bowel resection for cancer, a 70 year old man gets a swinging fever & becomes confused. Choose the SINGLE investigation, most likely to confirm the diagnosis, from the above list of options:
A. Abdominal ultrasound B. Echocardiogram C. Urine microscopy & culture D. Thick blood film E. Liver function tests F. Lumbar puncture G. IVP H. Blood cultures I. Full blood count J. Clinical exam only K. CT brain scan L. Chest x-ray & sputum cultures M. Throat swabs
H. Blood cultures - Post-surgical patients are at risk of developing an abscess. An abscess is a collection of pus or infected material. A swinging fever is strongly indicative of an abscess and a blood culture is required to identify the pathogenic organism. Percutaneous or surgical drainage will usually be required with appropriate antimicrobial therapy.
A 3 year old girl is febrile and has been unwell for 12 hours. She complains of a headache and is drowsy but otherwise neurologically intact. Choose the single most discriminating investigation in the acute management from the list of options:
A. ESR B. Blood cultures C. Chest X-ray D. Urine culture E. Stool electron microscopy F. EEG G. Stool culture H. Culture of joint aspirate I. C-reactive protein J. Lumbar puncture K. Throat swab L. Full blood count M. CT brain
J. Lumbar puncture - Meningitis commonly affects the extremes of age (60 years) due to impaired immunity in the former, and waning immunity in the latter. A lumbar puncture to obtain CSF is the most important investigation when this diagnosis is considered. This should not however delay the starting of empirical antimicrobial therapy. When the specific organism is identified, treatment can be modified accordingly. Fever, headache and drowsiness should make you suspicious here of this diagnosis. Atypical presentations can also occur, and these tend to happen in the very young, older or immunocompromised patients. In older patients, frequently, the only presenting sign of meningitis is confusion or an altered mental status. In infants, the signs and symptoms can be very non-specific and may include lethargy, poor feeding, irritability and fever. A rash is noted in 80-90% of patients, commonly 4-18 hours after initial symptoms and is associated with meningococcal aetiology (although may be present with any bacterial meningitis).
In bacterial meningitis, the CSF pressure is usually raised and WBC count is elevated. The glucose level is decreased compared to the serum value and the protein level is increased. In those who are untreated, Gram stain and culture of CSF are usually positive for the causative organism. However, in those who get antibiotics before cultures are obtained, the diagnostic yield is much lower (as low as 20%). If a lumbar puncture is delayed or regarded as clinically unsafe (for instance, raised intracranial pressure), then blood samples should be obtained for culture. Of course, a head CT should be considered before LP if there is focal neurology, new onset seizures, papilloedema, altered consciousness or any sign which may indicate raised ICP.
A 2 month old child has had a fever and cough for three days. He is tachypnoeic with grunting and has nasal flaring. Choose the single most discriminating investigation in the acute management from the list of options:
A. ESR B. Blood cultures C. Chest X-ray D. Urine culture E. Stool electron microscopy F. EEG G. Stool culture H. Culture of joint aspirate I. C-reactive protein J. Lumbar puncture K. Throat swab L. Full blood count M. CT brain
C. Chest X-ray - This 2 month old child has pneumonia and is displaying signs of respiratory distress (tachypnoea, nasal flaring, grunting). The most sensitive and specific test to perform is a CXR (PA and lateral). What may be seen is an infiltrate, consolidation, effusions and cavitation. Initial treatment is empirical with antibiotics (amoxicillin is the first choice in children).
A 3 year old has had a high fever and sore throat for 2 days. This evening he had a generalised convulsion lasting 2 minutes. He is now drowsy but rousable with no localising signs. Choose the single most discriminating investigation in the acute management from the list of options:
A. ESR B. Blood cultures C. Chest X-ray D. Urine culture E. Stool electron microscopy F. EEG G. Stool culture H. Culture of joint aspirate I. C-reactive protein J. Lumbar puncture K. Throat swab L. Full blood count M. CT brain
K. Throat swab - This child has septicaemia secondary to a throat infection caused by group A beta-haemolytic streptococcus (Streptococcus pyogenes). This is no additional reason to believe that this is toxic shock syndrome or rheumatic fever, but the sore throat and fever for 2 weeks requires a culture of a throat swab to look for the causative organism. This is the most specific and sensitive test for GAS. However, a rapid antigen test can also be done for GAS which is about 80% sensitive and 95% specific compared to a throat culture but obviously much quicker. Blood cultures can also be done as the infection is now systemic but the throat culture is the best choice here as this is the obvious initial source of the infection. Treatment is with antibiotics and may be with penicillin or amoxicillin. GAS resistance to macrolides has been reported. Supportive care in this patient is also essential.
An 8 year old has developed a painful, swollen knee over the last day. O/E there is a tender, warm effusion of the left knee. She also has a pyrexia of 38 degrees. Choose the single most discriminating investigation in the acute management from the list of options:
A. ESR B. Blood cultures C. Chest X-ray D. Urine culture E. Stool electron microscopy F. EEG G. Stool culture H. Culture of joint aspirate I. C-reactive protein J. Lumbar puncture K. Throat swab L. Full blood count M. CT brain
H. Culture of joint aspirate - This 8 year old has septic arthritis. The septic knee joint here will be painful, hot, swollen and restricted movement of the affected joint. In all cases of suspected joint sepsis, the joint needs to be aspirated and empirical antibiotics need to be commenced once appropriate cultures have been taken. If the cause is a low virulence organism, TB or if in an older patient, the joint is prosthetic, then the presentation may be more insidious in onset. If there is already underlying joint disease then a septic joint should be suspected if the symptoms are out of proportion to normal disease activity. Some 20% of cases of septic arthritis is polyarticular. In sexually active patients, gonococcal arthritis is an important differential to consider. Presence or absence of fever is not a reliable indicator of joint sepsis and the diagnosis is largely one of clinical suspicion – treatment should therefore be commenced on this basis regardless of blood test results or microbiology. Synovial fluid Gram stain and culture is however positive in 70% of cases.
A 12 month old boy has a 24 hour history of profuse diarrhoea and irritability. He has a low grade fever. Choose the single most discriminating investigation in the acute management from the list of options:
A. ESR B. Blood cultures C. Chest X-ray D. Urine culture E. Stool electron microscopy F. EEG G. Stool culture H. Culture of joint aspirate I. C-reactive protein J. Lumbar puncture K. Throat swab L. Full blood count M. CT brain
G. Stool culture - The best investigation for presumed infectious diarrhoea here is to do a stool culture. It is not an absolutely necessary investigation in most cases as the mainstay of treatment is supportive with rehydration and the correction of electrolyte imbalance. However, stool cultures should be sent from patients with symptoms which are persistent or severe enough to prompt the patient to seek medical attention. Cultures are also mandatory in cases of bloody diarrhoea and signs of systemic involvement. The cultures should be sent within 3 days of admission as the yield is significantly reduced beyond this time period – and results take 2-4 days to become available.
This condition is more common in the extremes of age and complications are also more common in these groups. The commonly examined complication is haemolytic uraemic syndrome, which develops in 10% of patients with E coli O157 infection. Another complication is death, which is uncommonly examined but needless to say most patients who die from infectious diarrhoea are at the extremes of age. Diarrhoeal illness in young infants under 2 years of age is characteristically caused by EPEC (enteropathogenic E coli). Use of antibiotics is controversial and in most cases is not necessary (except possible in traveller’s diarrhoea).
A 30 year old, man became unwell 4 weeks after a holiday in Africa. He developed headaches, muscle pains, feeling cold, severe rigors, high fever, flushing, vomiting and profuse sweating.
A. Toxoplasmosis B. Syphilis C. Enteric fever D. CMV (cytomegalovirus) E. Tetanus F. Tuberculosis G. Giardiasis H. Malaria I. Herpes zoster (shingles) J. HIV K. Influenza L. Rabies M. Viral hepatitis N. Glandular fever O. Cholera P. Polio
H. Malaria - In the Western world, almost all cases of malaria occurs in travellers so an adequate travel history is crucial or the diagnosis may be missed. Malaria is endemic in many parts of Africa and this is a clue. Patients typically present with non-specific symptoms such as a fever, sweats, chills and myalgia. Sometimes EMQs will describe patterns of fevers occuring at regular intervals of 48-72 hours associated with P. vivax, P. ovale and P. malariae infections but in most patients there is no specific pattern. The test of choice is Giesma-stained thick and thin blood smears. Thick films sensitively detect parasites whereas thin films allow species identification and calculation of parasitaemia to guide treatment. Note that blackwater fever is a complication of malaria which occurs due to haemolysis and the release of haemoglobin into blood vessels and subsequently into the urine, which can frequently lead to renal failure.
A 50 year old man became unwell after a holiday in India. He developed a fever, tiredness, night sweats and a productive cough. He lost half a stone in weight since his return from holiday 6 weeks ago.
A. Toxoplasmosis B. Syphilis C. Enteric fever D. CMV (cytomegalovirus) E. Tetanus F. Tuberculosis G. Giardiasis H. Malaria I. Herpes zoster (shingles) J. HIV K. Influenza L. Rabies M. Viral hepatitis N. Glandular fever O. Cholera P. Polio
F. Tuberculosis - The patient’s symptoms and travel history point towards pulmonary TB. It is important to have a high level of suspicion when evaluating patients with risk factors who present with suggestive symptoms. Night sweats, fever, malaise, cough, haemoptysis and erythema nodosum are all suggestive. Key risk factors for pulmonary TB include exposure to infection and returning from or being born in a high-risk region such as Asia, Africa and Latin America. If TB is suspected, the patient should be placed in isolation and a CXR obtained with 3 sputum samples cultured for AFB being the gold standard of diagnosis. Culture on solid media can take 4-8 weeks so sputum smears will be done before culture results are known. Interferon-gamma release assays (IGRAs) are now used by some hospitals to rapidly determine a patient’s TB status. All patients who have TB should be tested for HIV within 2 months of diagnosis.
Classically, in primary disease CXR shows middle and lower zone infiltrates. Post-primary TB usually involves apical changes with or without cavitation. However, recent students have indicated that both presentations are seen in both primary and post-primary TB. HIV positive patients tend to have a more atypical CXR including effusion, lower zone involvement and a miliary pattern.
A 20 year old student presented with a sore throat, fever, anorexia, malaise and lymphadenopathy. She was treated for tonsillitis by her GP but did not complete the course because she developed an allergic rash.
A. Toxoplasmosis B. Syphilis C. Enteric fever D. CMV (cytomegalovirus) E. Tetanus F. Tuberculosis G. Giardiasis H. Malaria I. Herpes zoster (shingles) J. HIV K. Influenza L. Rabies M. Viral hepatitis N. Glandular fever O. Cholera P. Polio
N. Glandular fever - EBV causes a maculopapular pattern after ampicillin, which is what this patient has been given. Infectious mononucleosis (glandular fever) is caused by EBV and is characterised by fever, pharyngitis and lymphadenopathy. Enlargement of the spleen also begins in the first week and lasts 3-4 weeks, occuring in half of all cases. Risk factors for EBV transmission include kissing and sex (your 20 year old student here). A FBC will show an atypical lymphocytosis. Confirmation of IM involves detection of the existence of heterophile antibodies using the Paul Bunnell monospot. A more accurate test is a serological test detecting EBV specific antibodies. Treatment is usually symptomatic but IM carries rare but potentially life threatening complications.
A 60 year old diabetic man with fever, malaise, headache and muscle pains. After a few days he became very ill and is now confined to his bed with a hot water bottle.
A. Toxoplasmosis B. Syphilis C. Enteric fever D. CMV (cytomegalovirus) E. Tetanus F. Tuberculosis G. Giardiasis H. Malaria I. Herpes zoster (shingles) J. HIV K. Influenza L. Rabies M. Viral hepatitis N. Glandular fever O. Cholera P. Polio
K. Influenza - Diabetics are more suspectible to infectious such as flu. This is an acute respiratory tract infection caused by seasonal viral influenza A or B, hence antibiotics would have no effect. It is characterised by respiratory symptoms including rhinorrhoea, cough, fever, chills, headache and myalgia. Examination can be expected to be otherwise unremarkable. Antigenic change presents a challenge in creating new vaccines as there is no incremental protection from previous vaccinations. There have been 4 pandemics since 1918, the recent one being ‘swine flu’ in April 2009, caused by new gene rearrangement of human, avian and swine influenza.
Two weeks after a holiday in the Far East, a 30 year old lady presented with anorexia, fever and joint pains. Jaundice appeared a week later and on examination her liver and spleen were both enlarged and very tender.
A. Toxoplasmosis B. Syphilis C. Enteric fever D. CMV (cytomegalovirus) E. Tetanus F. Tuberculosis G. Giardiasis H. Malaria I. Herpes zoster (shingles) J. HIV K. Influenza L. Rabies M. Viral hepatitis N. Glandular fever O. Cholera P. Polio
M. Viral hepatitis - This is likely hepatitis A which is primarily transmitted via the faecal-oral route. After the virus is consumed and absorbed, it replicates in the liver and is excreted in the bile (to be re-transmitted). Transmission usually precedes symptoms by about 2 weeks and patients are non-infectious one week after onset of jaundice. The history can reveal risk factors such as living in an endemic area, contact with an infected person, homosexual sex or a known food-borne outbreak. This is classically, in EMQs, associated with shellfish which is harvested from sewage contaminated water.
The clinical course of HAV consists of a pre-icteric phase, lasting 5-7 days, consisting characteristically of N&V, abdominal pain, fever, malaise and headache. Rarer symptoms may be present such as arthralgias and even severe thrombocytopenia and signs that may be found include splenomegaly, RUQ tenderness and tender hepatomegaly as well as bradycardia. The icteric phase is characterised by dark urine, pale stools, jaundice and pruritis. When jaundice comes on, the pre-icteric phase symptoms usually diminish, and jaundice typically peaks at 2 weeks. However, a fulminant course runs in
A 20 year old lady became ill 2 weeks after a holiday in Brazil. She developed a fever, headache, cough & constipation which turned to diarrhoea 10 days later. She also has a blanching red lesion on her chest and abdomen. She is also found to be bradycardic.
A. Toxoplasmosis B. Syphilis C. Enteric fever D. CMV (cytomegalovirus) E. Tetanus F. Tuberculosis G. Giardiasis H. Malaria I. Herpes zoster (shingles) J. HIV K. Influenza L. Rabies M. Viral hepatitis N. Glandular fever O. Cholera P. Polio
C. Enteric fever - Typhoid (or enteric fever) is a faecal-oral illness caused by Salmonella enterica, serotype S typhi, Salmonella enterica and S paratyphi. There are over 2500 serovars for S enterica. In this country, it is mainly from people who have returned from a country where it is endemic. The water supplies in Brazil may not treated and sanitary conditions may be dire which prompts transmission of this infection. Humans are the only known reservoir. The vaccine only offers moderate protection and does not protect against paratyphoid infection. This person has a fever which is a hallmark of infection. The fever of typhoid classically increases incrementally until a persistent fever with temperature 39-41 is established. There are also typically flu like symptoms after onset of fever and cough. Remember that characteristic findings such as bradycardia and rose spots may not be present and indeed rose spots may not be easy to spot in those with dark skin. Rose spots are blanching red lesions reported in 5-30% of cases usually occuring on the chest or abdomen.
This patient needs antibiotics though the temperature will fall over about week. A third generation cephalosporin is indicated. If the sensitivity panel returns and shows that this organism is sensitive to all antibiotics then ciprofloxacin should be given.
A 40 year old business man travels frequently to the Far East and Africa. He became generally unwell with a fever, generalised lymphatic swelling, diarrhoea. On examination he was noted to have oral canididasis
A. Giardiasis B. Polio C. Glandular Fever D. Malaria E. Viral Hepatitis F. HIV G. Tuberculosis H. Herpes Zooster (Shingles) I. Cholera J. Toxoplasmosis K. CMV (cytomegalovirus) L. Syphilis M. Tetanus N. Influenza O. Rabies
F. HIV - The aim here is to establish the patient’s prognosis and begin the patient on HAART (highly active antiretroviral therapy). This patient needs to have a CD4 count, screen for HBV and HCV, VDRL, tuberculin skin test and a CXR. HIV viral load is also assessed at baseline. Generalised lymphadenopathy is common in HIV and present as painless enlarged nodes, usually in 2 or more non-contiguous sites, of >1cm for > 3months. Fevers and night sweats are also commonly seen, which may also indicate TB, which needs to be excluded (and malaria if from an endemic area). Unexplained diarrhoea of >1 month duration with no pathogen diagnosed is also seen in HIV. The oral candidiasis is also indicative of HIV and is an opportunitistic infection in an immunocompromised host. This patient has WHO stage 3 disease and would be placed in CDC category B, or symptomatic non-AIDS conditions. Oropharyngeal candidiasis is not AIDS indicating, but if this patient has candidiasis of the bronchi, trachea or lungs, or oesophagus then this would indicate an AIDS diagnosis (CDC category C – once the patient reaches category C, they stay in this category). I would not worry too much about clinical staging which is done according to the WHO or CDC criteria.
Other category C conditions, which indicate AIDS, include CMV retinitis, Kaposi’s sarcoma, Mycobacterium avium complex/tuberculosis/other, PCP and toxoplasmosis. Key risk factors for HIV include needle sharing with IVDU, unprotected homo- and heterosexual intercourse (a higher risk if the receptive partner, particular with receptive anal intercourse), HIV infected blood transfusion, needle stick injury and high maternal viral load (mother to child transmission).
A 25 year old man developed a fever, malaise, lymphadenopathy and a rash on the palms of his soles of his feet. 6 weeks earlier he had a painless ulcer on his penis.
A. Giardiasis B. Polio C. Glandular Fever D. Malaria E. Viral Hepatitis F. HIV G. Tuberculosis H. Herpes Zooster (Shingles) I. Cholera J. Toxoplasmosis K. CMV (cytomegalovirus) L. Syphilis M. Tetanus N. Influenza O. Rabies
L. Syphilis - Syphilis is a common STI which is caused by the spirochete bacterium Treponema pallidum (subspecies pallidum). This history is textbook syphilis. People at high risk include those who have had sexual contact with an infected person, men who have sex with men, those with HIV and other STDs, those with multiple partners, commercial sex workers and those who use illicit drugs. The presentation is often asymptomatic but can manifest in many different ways. There is a ulcer of primary syphilis here noticed 6 weeks ago. Initial infection is local and leads to development of a macule. This develops into a papule which then ulcerates to form a chancre (typically appears 9 to 90 days after exposure, usually between 2-3 weeks) and may not be noticed by the patient (as it is painless). It typically develops in the anogenital area or cervix. There may also be (discrete, painless, rubbery) regional lymphadenopathy. This ulcer heals spontaneously.
This patient has then gone on to develop secondary syphilis 6 weeks later (typically 4-8 weeks). Here there is disseminated treponemal infection which has multi-system manifestations and a diverse presentation. There are constitutional symptoms here as well as generalised lymphadenopathy. There may also, like this case, be this typical generalised symmetrical macular, papular or maculopapular diffuse rash which typically affects the palms of the hands and the plantar aspects of the feet. This rash may also occur on the trunk and scalp and may ulcerate. There may also be flesh-coloured wart-like lesions in the genital area which is known as condylomata lata.
The treatment is with penicillin. Untreated syphilis facilitates the transmission of HIV and can cause considerable problems such as cardiovascular and neurological disease, as well as a congenital syndrome in the newborn. In some parts of the developing world, syphilis in pregnancy is a major cause of miscarriage and stillbirth. Pregnant women with syphilis are at risk of transmitting the infection across the placenta to the fetus.
A 32 year old man lacerated his leg in the garden. Two months later he developed a fever and headache followed by a permanent grin-like posture, inability to close his mouth, arching of his body with hyperextension of his neck.
A. Giardiasis B. Polio C. Glandular Fever D. Malaria E. Viral Hepatitis F. HIV G. Tuberculosis H. Herpes Zooster (Shingles) I. Cholera J. Toxoplasmosis K. CMV (cytomegalovirus) L. Syphilis M. Tetanus N. Influenza O. Rabies
M. Tetanus - This is very obvious tetanus which is caused by the exotoxin of the bacterium Clostridium tetani. This should have been prevented by appropriate management of the initial tetanus-prone wound and with complete active immunisation, with passive immunisation given when required. There is trismus here which has resulted in a grimace which is described as ‘risus sardonicus’ or sardonic smile. During a generalised tetanic spasm, the patient classically arches their back and extends their legs, flexes their arms in abduction and clenches their fists. Apnoea may also be a feature of these spasms. Intermittent tonic contractions of skeletal muscles often occurs which causes intensely painful spasms which may last for minutes – these are often triggered by stimuli such as noise, light and physical contact. Tetanic spasms can also produce opisthotonus, board like abdominal wall rigidity, dysphagia and apnoeic periods. Management of clinical tetanus involves supportive care (airway management is crucial here as spasms may compromise ventilation – without mechanical ventilation facilities such as in the third world, asphyxia is the most common cause of death due to muscle spasm), wound debridement, antimicrobials, passive and active immunisation, control of spasms and the management of autonomic dysfunction. Case fatality rate is 12-53%.
A 30 year old man recently returned from a holiday in Bangladesh. He developed watery diarrhoea 20 hours ago which has increased in volume. There has been vomiting. Now there is an almost continuous passage of loose and pale stools.
A. Giardiasis B. Polio C. Glandular Fever D. Malaria E. Viral Hepatitis F. HIV G. Tuberculosis H. Herpes Zooster (Shingles) I. Cholera J. Toxoplasmosis K. CMV (cytomegalovirus) L. Syphilis M. Tetanus N. Influenza O. Rabies
I. Cholera - This is a secretory diarrhoea caused by the bacterial organism Vibrio cholerae. This organism releases a toxin which stimulates adenylate cyclase. It is classically a disease of poverty but is also well described in returning travellers. Vomiting is a common early feature. The most striking and characteristic feature of cholera is the loose rice-water stools – a high volume diarrhoea which remains pale and loose, without blood. The presentation tends to be with litres of this rice-water stools. Culture of the organism provides a definitive diagnosis and rapid dipstick tests are currently available. Most patients will recover if the effects of the profound volume depletion are corrected with either oral or IV rehydration. Antibiotics do shorten the duration and severity of disease but the rising rate of bacterial resistance is becoming a problem. Note that about 70-80% of those infected with Vibrio cholerae do remain asymptomatic.
What is interesting is that blood group O appears to lead to more severe disease, but may be protective against initial infection. Many infectious diseases indeed do show a relationship between blood group and disease susceptibility
A 20 year old man was bitten by a dog in France. 2 months later he developed a headache, fever and abnormal behaviour including the fear of water.
A. Giardiasis B. Polio C. Glandular Fever D. Malaria E. Viral Hepatitis F. HIV G. Tuberculosis H. Herpes Zooster (Shingles) I. Cholera J. Toxoplasmosis K. CMV (cytomegalovirus) L. Syphilis M. Tetanus N. Influenza O. Rabies
O. Rabies - This 20 year old man has rabies and is going to die. Symptomatic rabies is 100% fatal and there is no curative medical therapy. Palliative care is needed in this situation and given that spasms in rabies (hydrophobia and aerophobia) are very much stimulus driven, the recommendation is seclusion, room darkening and restraint. Rabies is one horrible way to die. Current recommendations are also to use haloperidol. (Prog Palliat Care. 2006;14:62-67.). Opioid analgesics may also be of use here as well as anticonvulsants for seizures and neuromuscular blockers. If interested, you can look up the Milwaukee protocol, the use of which has led to only 6 patients who have been known to survive symptomatic rabies without receiving the vaccine. This involves putting the patient into a drug induced coma and then giving them antiviral drugs
A 40 year old lady recently returned from a holiday in Leningrad developed tiredness, flatulence, abdominal bloating and loose stools.
A. Giardiasis B. Polio C. Glandular Fever D. Malaria E. Viral Hepatitis F. HIV G. Tuberculosis H. Herpes Zooster (Shingles) I. Cholera J. Toxoplasmosis K. CMV (cytomegalovirus) L. Syphilis M. Tetanus N. Influenza O. Rabies
A. Giardiasis - Giardiasis (caused by Giardia lamblia) is an intestinal infection with a flagellated protozoan parasite which is transmitted faecal-orally (transmission is by ingestion of water or food contaminated with cysts, and by person to person spread). Symptoms vary from the asymptomatic passage of cysts to acute self limiting diarrhoea or chronic diarrhoea. Presenting symptoms aside from diarrhoea include abdominal bloating with cramps, frequent belching with a sulphuric smell, nausea, anorexia and fatigue. The diagnosis is made on the detection of cysts or trophozoites (the two morphological forms of Giardia which exist) in a stool sample. The first line treatment is with metronidazole or tinidazole.
A 12 year old boy presents with a flu-like illness, fever, headache, vomiting, tremor of the left side of the body and weakness of his left leg.
A. Giardiasis B. Polio C. Glandular Fever D. Malaria E. Viral Hepatitis F. HIV G. Tuberculosis H. Herpes Zooster (Shingles) I. Cholera J. Toxoplasmosis K. CMV (cytomegalovirus) L. Syphilis M. Tetanus N. Influenza O. Rabies
B. Polio - Poliovirus infection is usually asymptomatic and when symptomatic the most common presentation is with a minor GI illness. There is no cure for poliovirus infection and treatment is primarily supportive. This patient has acute flaccid paralysis (AFP), or paralytic poliomyelitis, which is the hallmark of major illness. This can rarely progress to bulbar paralysis and respiratory compromise. Paralytic poliomyelitis presents with decreased tone and motor function, as well as reduced tendon reflexes and muscle atrophy of the affected limb. Lack of vaccination is a strong risk factor. Remember that there are two main types of polio vaccine – Sabin, which is the oral weakened strain in disease endemic regions, and Salk, which is inactivated poliovirus in the rest of the world.
A 22 year old woman presents with a postcoital bleed but denies having other symptoms. She is currently in a relationship but is concerned that her partner is having sex with other women. Examination with a speculum reveals a mucopurulent yellow and cloudy discharge from the cervical os. The cervix is friable.
1. Chlamydia 2. Vaginal candidiasis 3. Bacterial vaginosis 4. Trichomonas vaginitis
- Chlamydia - This is genital tract chlamydia infection which is one of the most common STDs in the world. Remember that many infected individuals are asymptomatic. Women may present with cervical inflammation or yellow, cloudy discharge from the cervical os. A friable cervix is often also found on examination – the cervix bleeds easily with friction from a Dacron swab.
A 5 year old boy presents with a fever, headache and a very itchy vesicular rash mainly on his chest and face. He has recently taking paracetamol for a sore throat. There has also been a high fever in the last 24 hours. Lung fields are however clear on CXR. In some areas the lesions are crusted over while in others they appear to be newly formed. A classmate at school has had similar symptoms recently.
1. Bartonella 2. Scabies 3. Stevens-Johnson syndrome 4. Varicella zoster
- Varicella zoster - This is VZV infection (chickenpox) which typically presents with a fever, malaise and a widespread vesicular and pruritic rash which primarily affects the torso and face. Most countries in Europe do not immunise children against varicella.
A 58 year old man with uncontrolled HIV infection and AIDS presents with 2 week history of blurred vision bilaterally. He also reports seeing visual floaters. Examination reveals a man who is severely cachectic with generalised lymphadenopathy. Fundoscopy reveals creamy coloured areas with overlying retinal haemorrhages.
1. Cytomegalovirus 2. Infectious mononucleosis 3. Mycobacterium avium complex 4. Human herpes virus 6
- Cytomegalovirus - This is a presentation of CMV retinitis, which is the most common manifestation of CMV disease in AIDS, the second most common being colitis. However do remember that virtually any organ can be affected by CMV and it can cause a range of conditions from encephalitis to pneumonitis. In CMV retinitis, fundoscopy will reveal areas of infarction, haemorrhage, perivascular sheathing and retinal opacification. These findings here are of chorioretinitis.
A 18 year old student from Malaysia presents with 3 days of continuously high fevers. There are also general aches and pains and a predominantly frontal headache with retro-orbital pain which gets worse on eye movement. Examination reveals hypotension, tachycardia and a generalised skin flush with warm peripheries. There is also mild thrombocytopenia, elevated LFTs and low WBC count.
1. Dengue fever 2. Leptospirosis 3. Rickettsia 4. Rubella
- Dengue fever - Dengue in endemic in over 100 countries, especially SE Asia, Western Pacific and the Americas. It is an arbovirus which is transmitted by the Aedes aegypti mosquito found in the tropical and subtropical parts of the world. Clinical features include fever, headache, myalgia/arthralgia, skin flush and leucopenia, thrombocytopenia and elevated LFTs. Viral antigen or nucleic acid detection and serology are confirmatory tests to perform.
A 35 year old homosexual man with HIV presents to his GP after a holiday in Barcelona having recently noticed the presence of painless purple skin plaques on his lower legs and some generalised rubbery lumps located over his body. Examnation reveals that there is also a purple coloured mass on his hard palate.
1. Seborrhoeic dermatitis 2. Human papillomavirus 3. Photodermatitis 4. Kaposi sarcoma
- Kaposi sarcoma - This is Kaposi sarcoma which is a low-grade neoplasm caused by human herpesvirus-8 (or KSHV). It is associated with the acquired immunodeficiency associated with HIV infection. Oral KS can frequently affect the hard palate, gums and dorsum of the tongue. Cutaneous lesions are purple in colour and usually painless and non-pruritic. Histopathology of these vascular lesions will show characteristic atypical spindle-shape cells.
A 64 year old smoker is referred to the doctors by his dentist, who noticed a white coloured plaque on the lateral tongue margin and the floor of the mouth. It has a thickened, white and leathery appearance on examination. The surrounding mucosa is clinically normal. He has recently has a kidney transplant. In situ hybridisation confirms the diagnosis.
1. Squamous cell carcinoma 2. Syphilitic leukoplakia 3. Oral hairy leukoplakia 4. Candidiasis
- Oral hairy leukoplakia - This is oral hairy leukoplakia which presents as a painless white plaque found along the lateral tongue borders. There is history here which suggests immunosuppression. In situ hybridisation here has demonstrated the presence of EBV in the tissue.
A sexually active female student presents having noticed pearly umbilicated papules on her thigh which feel smooth to the touch. She tells you that these are itchy. Examination reveals local erythema around these lesions.
1. Lymphogranuloma venereum 2. Genital warts 3. Molluscum contagiosum 4. Pelvic inflammatory disease
- Molluscum contagiosum - This is molluscum contagiosum which is sexually transmitted in adulthood. Lesions appear as the umbilicated pearly and smooth papules mentioned. About a third of patients will also develop symptoms of local redness, swelling or pruritis. Adults should be treated for this STD.
A 22 year old man notices a painless penile ulcer. He has recently started his first sexually active relationship. His partner has no symptoms and he is also otherwise well. Examination reveals an indurated ulcer with rubbery and moderate inguinal lymphadenopathy.
1. Primary HIV infection 2. Chancroid 3. Genital herpes 4. Syphilis
- Syphilis - The chance of primary syphilis is usually a solitary, painless and clean ulcer with an indurated base. This is a common sexually transmitted infection caused by the spirochete Treponema pallidum. Treatment is with penicillin – untreated infection can cause considerable morbidity and facilitates HIV transmission.
A 35 year old woman who loves birds presents with a 10 day history of a low grade fever and a recent 2 day history of a cough which is non-productive. Examination reveals diffuse crackles on chest examination and mild hepatomegaly which is tender on palpation.
- Legionella
- Q fever
- Psittacosis
- Mycoplasma
- Psittacosis - Chlamydia psittaci causes a community-acquired atypical pneumonia. It is often acquired from domesticated or commercially raised birds or exotic imported birds. The presentation can be similar to Mycoplasma and Chlamydophila pneumoniae. Tetracyclines are the preferred treatment. Hepatomegaly can occur in this condition with pain on palpation but is uncommon, as can splenomegaly. Both organs, if enlarged, are diffusely so.
A 3 year old girl presents with a week history of pain in the abdomen and watery diarrhoea which became bloody after the first 8 hours. Three days before, she had distinctly recalled consuming a burger which may have been undercooked. Investigations show a mild anaemia and thrombocytopenia with blood smear demonstrating multiple schistocytes. Creatinine is also raised.
1. Antiphospholipid syndrome 2. Disseminated intravascular coagulation 3. HELLP syndrome 4. Haemolytic uraemic syndrome
- Haemolytic uraemic syndrome - This is haemolytic uraemic syndrome which is characterised by MAHA, thrombocytopenia and nephropathy. Most cases are in children and related to gastroenteritis caused by verotoxin producing E. coli (O157:H7). The presence of schistocytes establishes the presence of thrombotic microangiopathy. The anaemia, thrombocytopenia and renal dysfunction characterised by creatinine rise mark this diagnosis.
A 12 year old boy came back from summer camp and was taken to A&E feeling hot with vomiting and a sore throat. Laboratory tests and CXR is unremarkable and he is discharged. He returns later the same day with paraesthesias of the right arm and scalp, dysphagia and ataxia. When presented with water, he goes into laryngeal spasm and feels he is choking. Placement of nasal cannulae to give oxygen has the same effect.
1. Rabies 2. Attention seeking behaviour 3. Guillain-Barre syndrome 4. West Nile virus encephalitis
- Rabies - This is rabies, which can be difficult to diagnose if there is the absence of identifiable rabies exposure. There is non-specific prodrome of fever, chills, malaise, sore throat, vomiting, headaches and paraesthesias. The most specific signs of the disease are displayed here with hydrophobia and aerophobia (elicited by the placement of nasal cannulae). This patient is going to die and palliative care is needed. Rabies is considered 100% fatal.
A 13 year old presents with fever and sore joints. She has had a sore throat about 3 weeks ago but did not see a doctor about it. While waiting in A&E she develops choreiform movements of the whole body and head with facial expressions that resemble grimaces.
1. Streptococcus pyogenes 2. Septic arthritis 3. Borrelia burgdorferi 4. Clostridium tetani
- Streptococcus pyogenes - This girl has rheumatic fever which is caused by an autoimmune process following infection with group A streptococci. The 5 major manifestations of acute rheumatic fever is something you need to be aware of. They are carditis, polyarthritis, chorea, erythema marginatum and SC nodules. Various other signs can also be seen such as spooning sign and pronator sign.
A man from Tanzania presents with occassional diarrhoea accompanied by frank blood. Examination reveals mild hepatomegaly and a palpable spleen. On further questioning, he tells you he has been swimming in the local lake.
1. Leishmaniasis 2. Schistosomiasis 3. Salmonella 4. Toxoplasmosis
This is caused by a fluke which is acquired through exposure of the skin to contaminated freshwater (it is a snail-borne parasite). Patients can present acutely with bloody diarrhoea and hepatosplenomegaly is a common finding too. Diagnosis is made on microscopic visualisation of eggs in stools or urine. The preferred treatment for schistosome infections is with a drug called praziquantel.
A 73 year old cut his hand while gardening. He presents with lock jaw which results in a grimace. There are also intermittent tonic contractions of his muscles which are painful and last for minutes.These are sometimes triggered by noise.
1. Tetanus 2. Diphtheria 3. Hypocalcaemia 4. Generalised seizure
- Tetanus - This is caused by the exotoxin of Clostridium tetani and there is trismus here (lock jaw) which has resulted in risus sardonicus. The intermittent tonic contractions are also characteristic and the spasms can be triggered by both external and internal stimuli.
A 16 year old woman presents with a headache, fever and photophobia. Her mother became seriously concerned when she noticed a rash. O/E the girl grimaces with pain upon forced extension of the knee whilst her hip is flexed. You also demonstrate neck stiffness.
A. Pseudomonas aeroginosa B. Mycobacterium leprae C. Escherchia coliform D. Legionella pneumophila E. Campylobacter jejuni F. Neisseria meningitidis G. Mycobacterium tuberculosis H. Salmonella typhi
F. Neisseria meningitidis - This patient has meningitis. Schools and universities are common sites of outbreaks due to crowding. Commonly there will be a headache, fever and nuchal rigidity. There may also be an altered mental status, confusion, photophobia and vomiting. Kernig’s sign is uncommon but is positive when attempts to extend the leg are met with resistance when the patient is supine with the thigh flexed to 90 degrees. Another uncommon sign is Brudzinski’s sign and a petechial/purpuric rash, typically associated with meningococcal meningitis.
CT head should be considered before LP if there is any evidence of raised ICP. An LP will confirm the diagnosis with bacterial meningitis showing a low CSF glucose, elevated CSF protein and positive CSF culture/gram stain or meningococcal antigen.
Miss A who recently moved to the UK from Ethiopia where she had undergone two months of antibiotic therapy for a cough, night sweats and blood stained sputum arrives at clinic with lower back pain and kyphosis. There is also hip pain.
A. Pseudomonas aeroginosa B. Mycobacterium leprae C. Escherchia coliform D. Legionella pneumophila E. Campylobacter jejuni F. Neisseria meningitidis G. Mycobacterium tuberculosis H. Salmonella typhi
G. Mycobacterium tuberculosis - This patient has had pulmonary TB in Ethiopa but has not undergone a full course of anti-TB treatment. Subsequently, she has now developed extrapulmonary involvement. There is skeletal TB here with pain of the lower back. The onset of pain is usually gradual occuring over weeks to months and diagnosis is therefore frequently delayed. Local swelling and limitation of movement may be present, along with cold abscesses which may form if this is untreated. They are called cold abscesses as they are neither tender nor erythematous. These are also more common in HIV positive patients. MRI or a CT scan of the involved area is indicated here. Vertebral disease usually starts in the subchondral cancellous bone where it then spreads into the cortex and on to the disc. Destruction is more extensive on the ventral aspect than the dorsal aspect which leads to anterior wedging. As a result patients may present with kyphosis. There may also still be abnormalities on CXR consistent with TB. Note that normally TB initial therapy is a 4 drug regimen of isoniazid, rifampicin, pyrazinamide and ethambutol lasting for at least 6 months.
Three weeks following an illness which caused crampy abdominal pains, vomiting and diarrhoea a 26 year old presented with progressive bilateral leg weakness. Knee jerks and ankle jerks were both reduced on examination.
A. Pseudomonas aeroginosa B. Mycobacterium leprae C. Escherchia coliform D. Legionella pneumophila E. Campylobacter jejuni F. Neisseria meningitidis G. Mycobacterium tuberculosis H. Salmonella typhi
H. Campylobacter jejuni - This patient has Guillain-Barre syndrome. This condition is a demyelinating polyneuropathy. Classic neurology is a progressive symmetrical muscle weakness affecting lower extremities before upper extremities, and proximal muscles before distal muscles, accompanied by paraesthesias in the hands and feet which often precedes onset of weakness. The paralysis is typically flaccid with areflexia and progresses acutely over days, with an ensuing plateau phase followed by recovery. Two thirds of patients have a history of either prior influenza-like illness or gastroenteritis. This patient gives a history of gastroenteritis, the cause of which is likely Campylobacter. Studies have shown that 60-70% of acute cases are preceded by Campylobacter jejuni infection. Additionally, Campylobacter-associated GBS appears to have a worse prognosis with slower recovery and higher residual neurological disability. A study in Sweden has shown that the risk of developing GBS after Campylobacter jejuni infection is roughly 100 fold higher than after other infections.
Other weak risks include immunisation, cancer and lymphoma, older age, HIV infection and male gender. Up to 30% will develop respiratory muscle weakness requiring ventilation so spirometry should be carried out at 6 hour intervals initially (and may show reduced vital capacity). AST and ALT may be elevated though the cause is unclear. LP is useful and the classic finding is of elevated CSF protein with normal cell count (known as albuminocytological dissociation). Treatment is with supportive and disease modifying treatment (plasma exchange or high dose Ig).
A 42 year old previously healthy plumber is brought to hospital very confused by his wife with a fever, bradycardia and SOB. Investigations reveal elevated WBC count and Na 127mmol/l, K 4.2mmol/l, urea 6.5mmol/l. The doctor orders a urine sample.
A. Pseudomonas aeroginosa B. Mycobacterium leprae C. Escherchia coliform D. Legionella pneumophila E. Campylobacter jejuni F. Neisseria meningitidis G. Mycobacterium tuberculosis H. Salmonella typhi
D. Legionella pneumophila - Legionella is a gram negative rod. Legionella infecting the lungs is legionnaires’ disease or Legionella pneumonia whereas non-lung infection is known as Pontiac fever. This bacteria is found in aqueous environments such as lakes and almost all cases are from contaminated water systems, which relates to the risk factors of getting Legionella (this patient is a plumber). Smoking is also a risk factor. It can cause confusion as well as hyponatraemia, abdominal pain, diarrhoea and bradycardia. Legionella does not grow on routine culture media and diagnosis relies on urine antigen detection (hence the request for a urine sample), serology or culture on special media.
Mr D is a 17 year old man with cystic fibrosis for which he receives intensive physiotherapy. He has come in with shortness of breath and a mild fever. Sputum cultures demonstrated the growth of an organism which also produced a green pigment.
A. Pseudomonas aeroginosa B. Mycobacterium leprae C. Escherchia coliform D. Legionella pneumophila E. Campylobacter jejuni F. Neisseria meningitidis G. Mycobacterium tuberculosis H. Salmonella typhi
A. Pseudomonas aeroginosa - The green pigment here is pyoverdine which is produced by Pseudomonas. Pseudomonas has virulence factors for lung colonisation and is noted for its type III injection apparatus, which you may remember from your microbiology lectures. It is a gram negative bacilli causing hospital acquired pneumonia and UTI. It particularly affects immunocompromised hosts such as those on chemotherapy, those with cystic fibrosis (this patient), burns and wounds. Biofilms are antibiotic resistant.
Pseudomonas is also a special organism which is not sensitive to many antibiotics originally used for gram negatives. You need to prescribe a drug here which has specific anti-pseudomonal activity such as ceftazidime (a third generation cephalosporin), tazocin, ciprofloxacin, imipenem or gentamicin (usually used with one of the others). This organism acquires resistance genes very quickly so two antibiotics are given. It is, as mentioned, inherently resistant to many drug classes. Some are even resistant to all antibiotics.
A twenty-one year old girl who presents with shortness of breath on climbing stairs, her boyfriend has told her that she looks very pale and should see the doctor. Select the most APPROPRIATE first line investigation:
A. LFTs B. CRP C. LP D. CXR and sputum sample E. Blood culture F. Sputum sample G. FBC H. Pleural biopsy I. Wound swab and culture J. Urinalysis K. HIV test L. CT head
G. FBC - Pallor and exertional SOB are suggestive of anaemia here. A FBC should be done here which would expect to find a low Hb. The World Health Organisation defines anaemia as
An 18 year old history student who has just started at university for his studies develops a pounding headache and fever. The hall warden remarked that he shouted at her to turn the lights off and draw the curtains when she was called to see him. He was then reported as having a seizure. On arrival to A&E, a CT head scan is done. Select the most APPROPRIATE first line investigation:
A. LFTs B. CRP C. LP D. CXR and sputum sample E. Blood culture F. Sputum sample G. FBC H. Pleural biopsy I. Wound swab and culture J. Urinalysis K. HIV test L. CT head
C. LP - This patient has meningitis. Universities are common sites of outbreaks due to crowding. Commonly there will be a headache, fever and nuchal rigidity. There may also be an altered mental status, confusion, photophobia and vomiting. An LP will confirm the diagnosis with bacterial meningitis showing a low CSF glucose, elevated CSF protein and positive CSF culture/gram stain or meningococcal antigen. A CT head scan should be considered before LP if there is reason to suspect raised ICP such as focal neurology, seizures, papilloedema on fundoscopy or altered mental state. The seizure here would be an indication to do a CT scan before attempting LP.
Mr D is an eco warrior who has spent the last 6 months in India. He has come back very thin with a persistent cough which occasionally produces blood streaked sputum. He has never smoked cigarettes before as it is capitalist. Select the most APPROPRIATE first line investigation:
A. LFTs B. CRP C. LP D. CXR and sputum sample E. Blood culture F. Sputum sample G. FBC H. Pleural biopsy I. Wound swab and culture J. Urinalysis K. HIV test L. CT head
D. CXR and sputum sample - This sounds like pulmonary TB although it could well also be bronchial carcinoma. Remember that adenocarcinomas are usually located peripherally in the lung and are more common in non-smokers although most cases are still associated with smoking. Which ever the diagnosis is, a CXR will be useful with a sputum sample being obtained to check for TB. This patient should be placed in isolation due to suspected TB and 3 sputum samples cultured for AFB being the gold standard for diagnosis. Culture takes several weeks so sputum smears will be done before culture results are known. Interferon-gamma release assays (IGRAs) are now used by some hospitals to rapidly determine a patient’s TB status. All patients who have TB should be tested for HIV within 2 months of diagnosis.
After coming out of surgery two weeks ago Mrs J’s arm wound has started to produce pus and the whole area is inflamed and red. She has come to you as she is concerned it is not healing. Select the most APPROPRIATE first line investigation:
A. LFTs B. CRP C. LP D. CXR and sputum sample E. Blood culture F. Sputum sample G. FBC H. Pleural biopsy I. Wound swab and culture J. Urinalysis K. HIV test L. CT head
I. Wound swab and culture - This is a straightforward question. A wound swab and culture is needed to see what the infection is. This will guide treatment.
Mrs M presents with a severe headache and fever for the past 3 days. Examination reveals fever, photophobia and neck stiffness. Fundoscopy is performed which reveals bilateral papilloedema. Select the most APPROPRIATE first line investigation:
A. LFTs B. CRP C. LP D. CXR and sputum sample E. Blood culture F. Sputum sample G. FBC H. Pleural biopsy I. Wound swab and culture J. Urinalysis K. HIV test L. CT head
L. CT head - This patient obviously has meningitis. A CT head scan needs to be done here before a diagnostic LP as there is reason to suspect raised ICP with bilateral papilloedema on fundoscopy. This can exclude a brain abscess or generalised cerebral oedema. Meningitis associated complications may also be identified such as hydrocephalus and brain infarction.
A young adult with a 2 day history of left sided pleuritic chest pain, fever and cough productive of rusty coloured sputum. A CXR was obtained which showed left lower lobe shadowing suggestive of consolidation. On agar the sputum grew gram +ve cocci which demonstrated alpha-haemolysis.
A. Klebsiella pneumoniae B. Streptoccus pyogenes C. Enterococcus faecium D. Streptococcus pneumoniae E. Escherichia coli F. Chlamydophila psittaci G. Shigella H. Salmonella paratyphi I. Salmonella typhi J. Streptoccocus viridans K. Salmonella typhimurium L. Chlamydia trachomatis
F. Streptococcus pneumoniae - Classic lobar pneumonia with no signs and symptoms to suggest an atypical organism is most likely to due to pneumococcus. Streptococcus pneumoniae is, as mentioned in the question, an alpha haemolytic Gram positive cocci (also Streptococcus viridans but it does not present in this way). A CXR is the most specific and sensitive test available and antibiotics are indicated. The rusty coloured sputum is hinting at a pneumococcal pneumonia too.The patient has presented with common symptoms of fever and a productive cough. There is additionally pleuritic chest pain. Initial treatment of a CAP is empirical with antibiotics. Often diagnosis is made solely on history and examination findings. Management is guided by the patient’s CURB-65 score.
True or False, the olfactory nerve enters the skull via the cribriform plate.
True - It is the shortest cranial nerve and enters the skull at the cribriform plate, then becoming the olfactory tract.
True or False, acromegaly can be associated with a bitemporal hemianopia.
True - Pituitary tumours and craniopharyngiomas can compress the optic chiasm, sparing the outer fibres responsible for the nasal field. This causes the characteristic bitemporal hemianopia.
True or False, third nerve palsy is associated with a ‘down and out’ pupil.
True - Because the actions of the trochlear and abducens nerve are unopposed now.
True or False, the trigeminal nerve carries taste sensation from the anterior two-thirds of the tongue.
False - The facial nerve carries taste sensation and the trigeminal carries touch/pain/temperature sensation from the anterior two-thirds of the tongue.
True or False, the sixth cranial nerve adducts the eye.
False - It ABducts the eye.
True or False, the facial nerve supplies the masseter muscle.
False - Muscles of mastication (masseter, temporalis, medial pterygoid, lateral pterygoid) are supplied by the mandibular branch of the trigeminal nerve.
True or False, the eighth cranial nerve is responsible for balance.
True. The vestibular part of the nerve transmits positional and rotational information from the semicircular canals, saccule and utricle.
True or False, the ninth cranial nerve is the accessory nerve.
False - It is the glossopharyngeal nerve.
True or False - cutting the vagus nerve (vagotomy) increases acid production in the stomach.
False - The parasympathetic system generally induces gastrointestinal motility and secretions. Acetylcholine stimulates HCl production so cutting the vagus will reduce this.
True or False, the accessory nerve allows you to shrug your shoulders.
True - It supplies trapezius, which is the muscle responsible for shrugging shoulders!
True or False, hypoglossal nerve injury makes the tongue deviate away from the side of the injury.
False - The tongue will deviate towards the weaker side.
True or False, upgoing plantars suggest a LMN lesion.
False - It is really UMN sign.
A 25-year-old man was hit on the side of the head with the ball, whilst playing cricket. He recovered enough to finish off the game. He then presented to A&E 8 hours later with a severe headache and vomiting. He rapidly loses consciousness.
A. Extradural haemorrhage B. Cluster headache C. Trigeminal neuralgia D. Subarachnoid haemorrhage E. Migraine F. Tension headache
A. Extradural haemorrhage - Here we have the ‘lucid interval’ classically associated with an extradural haematoma. There is blood buildup this time between the dura mater and the skull. Compressive signs may also be present such as the down and out pupil due to CN III compression. The bleed here is usually from arteries, under high pressure, causing raised intracranial pressure. In this case, there is a chance the brain stem has been compressed causing his LOC. The bleed here is possibly from the anterior branch of the middle meningeal artery which runs beneath the pterion and is vulnerable at this point where the skull is thin. Remember that the pterion marks the junction of four bones: the parietal bone, temporal bone, sphenoid bone and frontal bone.
A 20-year-old medical student complains of a throbbing pain bilaterally across her forehead, which is present for some hours each time. She is busy catching up on online EMQs and lectures for her exams next week. Physical examination is normal.
A. Extradural haemorrhage B. Cluster headache C. Trigeminal neuralgia D. Subarachnoid haemorrhage E. Migraine F. Tension headache
F. Tension headache - A tension headache is commonly triggered by stress and mental tension (also, fatigure and missing meals), hence the name. It is more common in females and those in middle age, and there is a link with lower socioeconomic status, although this does not necessarily represent causation. Symptoms include a dull, non-pulsatile and constricting bilateral pain, which is often described as a band across the patient’s head. It is not severe or disabling but classically worsens as the day progresses. This headache normally responds well to simple analgesics.
A 20-year-old student complains of a throbbing headache. It lasts for a few hours, and is associated with nausea. She can usually predict the headache half an hour before, when she sees flashing lights.
A. Extradural haemorrhage B. Cluster headache C. Trigeminal neuralgia D. Subarachnoid haemorrhage E. Migraine F. Tension headache
E. Migranes - Migraine is a chronic condition, with genetic determinants, which usually presents in early to mid life. The typical migraine aura this patient describes (which can be visual, sensory or speech symptoms) which can occur during or before the headache, is pathognomic, but is not seen in the majority of patients. The aura can be positive phenomena (for example this patient seeing flashing lights) or negative phenomena (for example visual loss). Nausea, photophobia and disability (the headache gets in the way with the patient’s ability to function) accompanying a headache also suggest a migraine diagnosis. The headache of a migraine tends to be prolonged if untreated, and tends to be unilateral and pounding (but does not have to be). Tests aim to rule out other differentials, although if the history is compatible and neurological examination is unremarkable, further testing is not needed.
A 34-year-old man has attacks of sudden severe pain waking him up for the last fortnight. The pain is on the right side of his face and makes his right eye water.
A. Extradural haemorrhage B. Cluster headache C. Trigeminal neuralgia D. Subarachnoid haemorrhage E. Migraine F. Tension headache
B. Cluster headache - Cluster headache is characterised by attacks of severe pain localised to the unilateral orbital, supraorbital and/or temporal areas which lasts from 15 minutes to 3 hours, and occurs with a frequency ranging from once every other day to 8 times a day. These attacks can occur at the same time period of many weeks (known as the cluster period) accompanied by ipsilateral autonomic signs. The cause is hypothalamic activation with secondary trigeminal and autonomic activation (for instance, lacrimation, rhinorrhoea, nasal congestion, conjunctival injection and partial Horner’s i.e. ptosis and miosis). Cluster period attacks can be triggered by things like alcohol. Greater occipital nerve blockade often provides immediate relief until preventative medications take effect.
A 56-year-old man has a sudden pain that ‘shoots’ to his chin whilst shaving. It disappears after a few seconds.
A. Extradural haemorrhage B. Cluster headache C. Trigeminal neuralgia D. Subarachnoid haemorrhage E. Migraine F. Tension headache
C. Trigeminal neuralgia - Trigeminal neuralgia occurs as episodes of severe unilateral pain in the distribution of CNV, usually lasting seconds, with no pain occuring between these episodes. Examination is often unremarkable. The pain is described as sharp, intense, stabbing or burning. It can be triggered commonly by actions such as eating, tooth brushing, cold and touch. Shaving and eating seem to be common in EMQs. Most people are asymptomatic between attacks although the severity of the pain makes these patients live in constant fear. TN is more common in MS and incidence increases with age. Post-herpetic TN is also possible. The mainstay of treatment is medical, with antiepileptics such as carbamazepine (which is the only medicine which is proven in RCTs and is therefore typically first line). If medical treatment fails, surgical options do exist such as microvascular decompression.
A 40-year-old man presents with an instantaneous onset of a severe headache, followed by drowsiness & vomiting.
A. Extradural haemorrhage B. Cluster headache C. Trigeminal neuralgia D. Subarachnoid haemorrhage E. Migraine F. Tension headache
D. Subarachnoid haemorrhage - SAH (bleeding into the subarachnoid space) presents with sudden severe headache patients will often describe as the worst headache of their life, and can often be so bad that they feel like they’ve been kicked in the back of the back. Half of all patients lose consciousness and eye pain with exposure to light can also be seen. Altered mental status is common. SAH occurs most commonly in the 50-55 age group and affects women and black people more than men and white people.
The most common cause of non-traumatic SAH is an aneurysm which ruptures. Conditions which predispose to aneurysm formation and SAH include adult PKD, Marfan’s, NF1 and Ehlers-Danlos. Cerebral aneurysms arise around the circle of Willis. A CT scan is indicated, and if unrevealing, this should be followed by an LP. Cerebral angiography can confirm the presence of aneurysms. The patient should be stabilised and this followed by surgical clipping or endovascular coil embolisation, the choice is subject to much current controversy sparked by relatively recent research. Complications can commonly occur and include rebleeding, hydrocephalus and vasospasm.
A 52 year old fund manager with a history of previous heart attacks, feels some palpitations and collapses. A witness said that he went very pale as he collapsed but then became flushed and regained consciousness after 30 seconds.
A. Hypoglycaemia B. Anaemia C. Stokes-Adams attack D. Opioid overdose E. Postural hypotension
C. Stokes-Adams attack - Stokes-Adams attacks are episodes of transient LOC due to sudden decreased cardiac output. The previous heart attacks and later palpitations towards an arrhythmia such as heart bock, which caused the attack. Pallor prior to the attack and facial flushing due to reactive hyperemia after the attack is characteristic of a Stokes-Adams attack. Definitive treatment is with surgical insertion of a pacemaker.
A 24-year-old patient with diabetes starts a new preparation of insulin. After going to the gym he feels sweaty, nauseous and dizzy. A few minutes later he is found unconscious on the floor.
A. Hypoglycaemia B. Anaemia C. Stokes-Adams attack D. Opioid overdose E. Postural hypotension
A. Hypoglycaemia - In DM, hypoglycaemia is usually secondary to insulin or oral hypoglycaemics. Symptoms of hypoglycaemia are present when glucose drops
An 80 year old man fainted with a 2 week history of abdominal pain and coughing up a black coffee-ground like substance. He has been feeling irritable, tired and sleepy.
A. Hypoglycaemia B. Anaemia C. Stokes-Adams attack D. Opioid overdose E. Postural hypotension
B. Anaemia - This man is anaemic and as a result he has fainted. This is IDA from a UGI bleed. His faint can also be attributed to hypovolaemia from his blood loss, from a presumed peptic ulcer (which accounts for his abdominal pain and coffee ground vomit). Whilst he may also display postural hypotension due to his hypovolaemia, this is not what this question is looking for.
A 75-year-old man is found on his bedroom floor by his wife and is now conscious. He got out of bed in the middle of the night to go to the toilet and felt dizzy and fell to the ground. He is on treatment for hypertension and has no other medical problems.
A. Hypoglycaemia B. Anaemia C. Stokes-Adams attack D. Opioid overdose E. Postural hypotension
E. Postural hypotension - This patient has postural hypotension which is demonstrated by a fall of >20 in systolic blood pressure and >10 diastolic within 3 minutes of standing upright. It is a side effect of anti-hypertensives and is a common problem in the elderly. A good history should be enough to diagnose this.
A 38-year-old homeless man is found unconscious on the street. On examination he has pinpoint pupils, a respiratory rate of 6 breaths per minute and needle marks on both arms.
A. Hypoglycaemia B. Anaemia C. Stokes-Adams attack D. Opioid overdose E. Postural hypotension
D. Opioid overdose - Opiate OD signs include CNS depression, miosis and apnoea. Finding small constricted pupils in someone who is unconscious is highly indicative. Naloxone is indicated both therapeutically and diagnostically. If there is a response, then it is diagnostic. Another diagnosis should be sought if the patient is unresponsive. IV is the preferred route of administration although naloxone can be given IM or SC if IV access cannot be established. Ventilatory support is key with 100% oxygen. You can check out Toxbase for a full database on poisons and treatments.
A 20-year-old feverish student developed a headache and vomiting during his sociology lecture. The only other student who attended, reports that he was breathing very quickly. On arrival at the surgery 30 minutes later he was semi-conscious, & breathing irregularly. His upper limbs were also jerking.
A. Right-sided stroke B. Myasthenia gravis C. Transient ischaemic attack D. Meningitis E. Pontine haemorrhage F. Hepatic encephalopathy G. Huntington’s disease H. PCA aneurysm I. Partial seizure J. Parkinson's disease K. Multiple sclerosis
D. Meningitis - This 20 year old student has meningitis. Meningitis can present with a host of symptoms and commonly presents with seizures (children who have seizures are more likely to be infected by Streptococcus pneumoniae and Haemophilus influenza than meningococcus). Universities are common sites of outbreaks due to crowding. Commonly there will be a headache, fever and nuchal rigidity. There may also be an altered mental status, confusion, photophobia and vomiting. Kernig’s sign is uncommon but is positive when attempts to extend the leg are met with resistance when the patient is supine with the thigh flexed to 90 degrees. Another uncommon sign is Brudzinski’s sign and a petechial/purpuric rash, typically associated with meningococcal meningitis.
CT head should be considered before LP if there is any evidence of raised ICP. An LP will confirm the diagnosis with bacterial meningitis showing a low CSF glucose, elevated CSF protein and positive CSF culture/gram stain or meningococcal antigen.
A 30-year-old woman experienced a strange feeling in her stomach, followed by stiffness & jerking in the left arm. Afterwards, she felt drowsy but remembers everything.
A. Right-sided stroke B. Myasthenia gravis C. Transient ischaemic attack D. Meningitis E. Pontine haemorrhage F. Hepatic encephalopathy G. Huntington’s disease H. PCA aneurysm I. Partial seizure J. Parkinson's disease K. Multiple sclerosis
I. Partial seizure - The feeling this woman felt in her stomach before (aura), the description of the arm, incontinence and feeling “worn out” afterwards make this a seizure. Seizures can happen in anyone but epilepsy is the tendency to have repeated, unprovoked seizures (but this is still the best answer option from the available list). Epileptic seizures can be split into focal (partial/localised) and generalised (involving both hemispheres). Generalised seizures can be further divided into abscence (which can be typical or atypical), myoclonic, clonic, tonic, tonic-clonic and atonic types. Be sure to exclude other causes, such as a space occupying lesion with a CT or MRI scan. There may also be tongue biting. Always think of “before, during and after” when taking the history. History taking is the most important aspect of diagnosis. Antiepileptic monotherapy is the preferred treatment.
An 80-year-old woman who is a smoker was brought into A&E from a residential home where her carers noticed that she had difficulty swallowing and that she also had difficulty moving her left arm and leg for the past few days.
A. Right-sided stroke B. Myasthenia gravis C. Transient ischaemic attack D. Meningitis E. Pontine haemorrhage F. Hepatic encephalopathy G. Huntington’s disease H. PCA aneurysm I. Partial seizure J. Parkinson's disease K. Multiple sclerosis
A. Right sided stroke - Weakness on one side and the difficulty swallowing makes this likely to be a stroke. If you have a think about the motor pathways you will realise that this is a right sided stroke. It is important is perform a CT head to exclude a haemorrhagic aetiology and consider thrombolysis with tPA if within the 4.5 hour window and there are no contraindications. Thrombolysis is done with alteplase at 10% bolus, 90% infusion at a dose of 0.9 mg/kg. Presentation after the 4.5 hour window is managed with aspirin. The Bamford/Oxford Stroke Classification subtypes ischaemic stroke according to vascular territory of infarction. After initial management, stroke care involves the ethos of an MDT environment with rehabilitation.
A 25-year-old pregnant lady presents with increasing muscle weakness. She also complains of double vision & drooping eye lids.
A. Right-sided stroke B. Myasthenia gravis C. Transient ischaemic attack D. Meningitis E. Pontine haemorrhage F. Hepatic encephalopathy G. Huntington’s disease H. PCA aneurysm I. Partial seizure J. Parkinson's disease K. Multiple sclerosis
B. Myasthenia gravis - This lady has myasthenia gravis, which is an autoimmune condition with antibodies affecting the NMJ, mostly the nAChR at the post-synaptic muscle membrane. Although some have antibodies against MuSK, and there are other proteins involved. MG is characterised by muscle weakness which increases with exercise (fatigue, unlike Lambert-Eaton myasthenic syndrome). Commonly, presentations include diplopia and drooping eyelids like this patient, and there may also be SOB, proximal limb weakness, facial paresis and oropharyngeal weakness. MG is associated with thymic hyperplasia in 70% or thymoma in 10%, and these associations can also crop up in EMQs. There will be elevated serum AChR receptor antibody titres or MuSK antibodies. Electrophysiology will demonstrate a decremental response on repetitive nerve stimulation. Treatment includes anticholinesterases (pyridostigmine, and immunotherapy. Patients may also require a thymectomy. Some 15-20% may experience a myasthenic crisis (which needs mechanical ventilation). Do you know what the Tensilon test is and why edrophonium is given in this test?
A 79-year-old man complains of difficulty walking. On examination you also notice he has a resting tremor & his limbs oppose movement.
A. Right-sided stroke B. Myasthenia gravis C. Transient ischaemic attack D. Meningitis E. Pontine haemorrhage F. Hepatic encephalopathy G. Huntington’s disease H. PCA aneurysm I. Partial seizure J. Parkinson's disease K. Multiple sclerosis
J. Parkinsons disease - Parkinson’s is characterised by a resting tremor, rigidity, bradykinesia and postural instability. The resting tremor occurs at 4-6 Hz at rest which dissipates with the use of limbs, with generally asymmetrical onset. There is often also cogwheeling, especially if there is a superimposed tremor. The patient may have other signs like a mask like face due to the loss of spontaneous facial movement, hypophonia and micrographia, and may walk around in a shuffling gait with a stooped posture. The diagnosis is clinical. Treatment is symptomatic in an MDT setting. Medical therapy includes MAO-B inhibitors and DA agonists, for example rasagiline and carbidopa/levodopa. There are other medical therapies depending on the specific symptoms the patient presents with.
A 30-year-old lady presented with pain in her left eye and numbness & weakness of her right leg. Two months earlier she had an episode of double vision in the left eye.
A. Right-sided stroke B. Myasthenia gravis C. Transient ischaemic attack D. Meningitis E. Pontine haemorrhage F. Hepatic encephalopathy G. Huntington’s disease H. PCA aneurysm I. Partial seizure J. Parkinson's disease K. Multiple sclerosis
K. Multiple Sclerosis - Multiple sclerosis is a demyelinating CNS condition which is characterised by 2 or more episodes of neurological dysfunction which are separated in both time and space. MS classically presents in white women aged 20-40 with temporary visual/sensory loss although any presentation can occur. MRI is a sensitive test but less specific than spinal MRI, however, spinal MRI is abnormal in fewer cases. Treatment aims at treating the attack, preventing future attacks and symptomatic treatment of problems like bladder dysfunction, pain and fatigue.
A 65-year-old hypertensive man has complained of losing vision twice in one eye, which lasted for a few hours and then went back to normal. He says it’s like a ‘a black sheet falling over the front of my eye’.
A. Right-sided stroke B. Myasthenia gravis C. Transient ischaemic attack D. Meningitis E. Pontine haemorrhage F. Hepatic encephalopathy G. Huntington’s disease H. PCA aneurysm I. Partial seizure J. Parkinson's disease K. Multiple sclerosis
C. Transient Ischaemic Attack - A TIA is colloquially called a ‘mini stroke’ with symptoms typically lasting under an hour. Here there is a description of focal neurological deficit unilaterally in the form of amaurosis fugax. There may also be other signs like unilateral weakness or sensory loss, hemianopsia, aphasia and cranial nerve defects, to name but a few. An antiplatelet drug such as aspirin is effective secondary prevention if the patient is not already anticoagulated. The patient will be anticoagulated if they have a likely or known cardioembolic source such as AF. Clopidogrel is an alternative in those who do not tolerate aspirin.
A 72-year-old man with a HbA1C of 12.2% has presented with ulcers in his right foot & bilateral loss of sensation below knee.
A. Diabetic neuropathy
B. Carpal tunnel syndrome
C. Charcot-Marie-Tooth disease
A. Diabetic Neuropathy - Diabetic neuropathy has lead to the painless ulcer most likely developing over pressure points in the foot such as on the ball of the foot. This may have resulted from an object becoming lodged in the shoe and eroding through the skin with walking. This is why it is important for diabetics to check their feet regularly and to wear specialised footwear. Diabetic neuropathy is a microvascular complication of DM and is characterised by peripheral nerve dysfunction. Pain is a common complaint such as the burning sensation this patient describes. Patient’s may also describe the pain as prickling or sticking.
Complications range from the painless neuropathic ulcer described, at areas of the foot where there is weight loading (particularly the metatarsal heads), to the Charcot foot with severe architectural destruction of the foot. Foot ulceration is a common precusor to amputation. Foot care is crucial in DM. Examination should include peripheral pulses, reflexes and sensation to light touch with a 10g monofilament, vibration (128Hz tuning fork), pinprick and proprioception. The pain may be treated with medications like pregabalin and gabapentin.
A 40-year-old typist presents with pain & tingling in the lateral side of the left hand, worse at night when she had to get out of bed to shake the hand for relief.
A. Diabetic neuropathy
B. Carpal tunnel syndrome
C. Charcot-Marie-Tooth disease
B. Carpal tunnel syndrome - Carpal tunnel syndrome is the most common nerve entrapment and women just past middle age are at the highest risk. Symptoms include numbness/tingling of the thumb and radial fingers, an aching wrist and clumsiness (especially with fine motor tasks). The symptoms are of gradual onset and often wake the patient up at night, and is relieved by shaking the wrist. Numbness is normally on the palmar aspect of the thumb, index and middle fingers (but not the little finger). When the patient wakes up, there may be difficulty flexing or extending fingers. Symptoms in the day tend to be associated with activity. The most sensitive and specific test for diagnosis is EMG and can confirm damage to the median nerve in the carpal tunnel and categorise the severity of the damage. There are specific tests for CTS such as Tinel’s test and Phalen’s test, though clinically these are not particularly useful due to sensitivity and specificity.
CTS is caused by anything that causes a reduction in the size of the carpal tunnel – from inflammation, arthritis and tenosynovitis to old fractures. In CTS there is preserved sensation of the palm as the palmar cutaneous branch comes off a few cm above the carpal tunnel.
A 10-year-old boy has wasting of his lower leg, which look like ‘inverted champagne bottles’. His father had the same condition.
A. Diabetic neuropathy
B. Carpal tunnel syndrome
C. Charcot-Marie-Tooth disease
C. Charcot-Marie-Tooth disease - Charcot-Marie-Tooth disease comprises of a group of hereditary peripheral neuropathies with different genetic abnormalities, hence the FH here (most patients have a FHx). Pes cavus (high foot arches), hammer toes and distal atrophy of the hands and legs are characteristic (resulting in this inverted champagne bottle appearance). Pes cavus can be associated with areflexia. Patients may have to lift their legs up excessively to clear the toes. Rehabilitative and orthotic treatments can help to keep patients functional. There are no other known risk factors for this condition aside from FH. The risk of passing the condition on to the next generation varies depending on the subtype of CMT, which have different inheritance patterns. Most CMT1 and CMT2 are autosomal dominant and CMT 4 is autosomal recessive. There are also X-linked forms.
A 75 year old lady is brought to A&E by her daughter who found her on the floor. She appears confused which you confirm with 4/10 on the AMTS. O/E she is febrile and complains of a burning pain in her lower abdomen.
A. UTI B. Faecal impaction C. Hypoxia D. Liver failure E. Post-operative F. Hypoglycaemia. G. Thiamine/B1 deficiency H. U&E imbalance I. Drug effect J. MI K. CVA
A. UTI - Confusion and a fever is not an uncommon presentation of UTI in the elderly where the presentation is often non-specific. The lower abdominal suprapubic pain here is another clue. 10 percent of women over the age of 70 have UTIs. Post-menopause (absence of oestrogen) is a strong risk factor for UTI in women. In women of this age group, sexual activity is less strongly associated with UTI. Treatment can be started on the symptoms here as well as dipstick result, but urine culture and sensitivity confirms the diagnosis and guides the appropriate use of antibiotics. A positive nitrite and leucocyte esterase in the urine indicates a likely UTI diagnosis. A midstream clean-catch urine specimen should be sent for culture (looking for uropathogenic organisms) if symptoms are atypical or do not respond to treatment.
The AMTS is used to rapidly assess mental function and a score of 6 or less suggests either delirium or dementia (the former is acute decline).
A 42 year old man who lives alone presents with confusion and ataxia. Both his hands have a Dupytrens contracture.
A. UTI B. Faecal impaction C. Hypoxia D. Liver failure E. Post-operative F. Hypoglycaemia. G. Thiamine/B1 deficiency H. U&E imbalance I. Drug effect J. MI K. CVA
G. Thiamine/B1 deficiency - This patient has Wernicke’s encephalopathy. Wernicke’s is due to acute thiamine deficiency, which is a problem in alcoholics (Dupytren’s contracture here is a clue to alcohol use). Others at risk include those with AIDS, cancer and treatment with chemotherapy, malnutrition and GIT surgery, especially bariatric procedures. It is a clinically under-diagnosed condition. The classic EMQ triad is of mental change, ophthalmoplegia and gait dysfunction, which is actually only seen in 10% of cases. In reality, the manifestations are varied and a high index of suspicion is needed. Despite there, the manifestiations typically include altered consciousness, gait disorders and eye movement abnormalities. This is an emergency and treatment is with parenteral replacement of thiamine. This avoids permanent neurological damage including later development of Korsakoff’s psychosis, which is irreversible. Note that thiamine should be given before dextrose! Magnesium deficiency also needs to be corrected as it is a co-factor in the functioning of thiamine dependent enzymes.
A 16 year old boy dressed in sports kit, presents to A&E confused, sweating and with a tremor. His speech starts to slur and he begins to lose conciousness.
A. UTI B. Faecal impaction C. Hypoxia D. Liver failure E. Post-operative F. Hypoglycaemia. G. Thiamine/B1 deficiency H. U&E imbalance I. Drug effect J. MI K. CVA
F. Hypoglycaemia - This patient has symptoms of hypoglycaemia, present when glucose drops
An 80 year old man on your ward becomes confused. He has no signs of an infection. You notice his mouth and lips are dry.
A. UTI B. Faecal impaction C. Hypoxia D. Liver failure E. Post-operative F. Hypoglycaemia. G. Thiamine/B1 deficiency H. U&E imbalance I. Drug effect J. MI K. CVA
H. U&E imbalance - This patient has become confused and there is an acute change in mental status. Dehydration (volume depletion) can be a relatively common cause. (The most common causes of volume depletion are haemorrhage, vomiting, diarrhoea, diuresis or third space losses). FBC in this case may reveal a raised haematocrit and high Hb. Serum electrolyte analysis may reveal derangements such as hyper or hypokalaemia and hyponatraemia. Urine specific gravity can be expected to be high and serum urea to creatinine ratio >20. Examination of this dehydrated patient may reveal signs like dry mucosal membranes, postural hypotension, postural tachycardia and possibly even shock depending on severity. The patient may complain of symptoms such as thirst, tiredness, muscle cramps, pain in the chest and/or abdomen as well as confusion. The confusion may result from U&E abnormalities (the best option from this list) but is non-specific and may indeed reflect poor cerebral perfusion or uraemia in the setting of impaired renal function, which are equally if not more likely in this patient.
A 77 year old lady with longstanding AF presents with confusion. She collapsed suddenly at home. O/E she has an extensor plantar response.
A. UTI B. Faecal impaction C. Hypoxia D. Liver failure E. Post-operative F. Hypoglycaemia. G. Thiamine/B1 deficiency H. U&E imbalance I. Drug effect J. MI K. CVA
K. CVA - Atrial fibrillation is strongly implicated in the risk of cardioembolic stroke. Confusion is common, especially in the elderly who have had past strokes and in those with cognitive dysfunction. An extensor plantar response is known as Babinski’s sign and is a sign of an UMN lesion. The normal response is flexion where the big toe moves downwards. It is important is perform a CT head exclude a haemorrhagic aetiology and consider thrombolysis with tPA if within the 4.5 hour window and there are no contraindications. Thrombolysis is done with alteplase at 10% bolus, 90% infusion at a dose of 0.9 mg/kg. Presentation after the 4.5 hour window is managed with aspirin. The Bamford/Oxford Stroke Classification subtypes ischaemic stroke according to vascular territory of infarction. After initial management, stroke care involves the ethos of an MDT environment with rehabilitation.
A 65 year old man with a history of an MI 2 years ago. He lost conciousness and presents to you 36 hours later with reduced power in his left arm and leg.
A. Anaemia B. Vasovagal syncope C. TIA D. Cardiac arrhythmia E. Stroke F. Postural hypotension G. Myxoedema coma H. Carotid sinus sensitivity I. Hypoglycaemia J. Aortic stenosis K. Epilepsy L. Pulmonary stenosis
E. Stroke - If you have a think about the motor pathways you will realise that this is a right sided stroke. Cardiac conditions are potential risks for cardioembolism. The previous MI could, for instance, lead to regional wall motion abnormalities or decreased LV ejection fraction which can be risks. Other cardiac conditions include valve disease, PFO, mitral prolapse, prosthetic heart valve and cardiomyopathy. Other major risk factors include AF, DM, smoking, hypertension and FH. It is important is perform a CT head exclude a haemorrhagic aetiology and consider thrombolysis with tPA if within the 4.5 hour window and there are no contraindications. Obviously this patient has presented to late for this but thrombolysis is usually done with alteplase at 10% bolus, 90% infusion at a dose of 0.9 mg/kg. Presentation after the 4.5 hour window will be managed with aspirin.
The Bamford/Oxford Stroke Classification subtypes ischaemic stroke according to vascular territory of infarction. After initial management, stroke care involves the ethos of an MDT environment with rehabilitation. This is not a TIA which resolves within 24 hours of symptom onset, although in practice this is often difficult to distinguish due to interventional measures which are rapidly enacted.
A 56 year old lady collapses whilst running for the bus. O/E there is a thrusting apex beat and an ejection systolic murmur. This is best heard on expiration and radiates to the carotids
A. Anaemia B. Vasovagal syncope C. TIA D. Cardiac arrhythmia E. Stroke F. Postural hypotension G. Myxoedema coma H. Carotid sinus sensitivity I. Hypoglycaemia J. Aortic stenosis K. Epilepsy L. Pulmonary stenosis
J. Aortic Stenosis - Aortic stenosis can present with chest pain, dyspnoea and syncope. It is characterised by a harsh ejection systolic murmur heard loudest at the right upper sternal edge at end expiration, which radiates up towards the carotids. The pulse pressure is narrow and there may be an associated slow-rising and plateau pulse. Doppler echo is vital for diagnosis and shows a pressure gradient across the narrowed valve orifice. 20% of cases are due to a congenital bicuspid valve. The most common cause of aortic stenosis in adults is calcification of normal trileaflet valves. Clinically stable patients may be considered for surgical repair or TAVR.
A previously well 14 year old girl collapses after being in a crowd for 2 hours at a pop concert.
A. Anaemia B. Vasovagal syncope C. TIA D. Cardiac arrhythmia E. Stroke F. Postural hypotension G. Myxoedema coma H. Carotid sinus sensitivity I. Hypoglycaemia J. Aortic stenosis K. Epilepsy L. Pulmonary stenosis
B. Vasovagal syncope - This is vasovagal syncope which is also known as the ‘common faint’. It has many manifestations and many triggers such as emotional upset, unpleasant sights, fear and pain. These tend to recur and patients needs to be warned of this and that they are almost never life-threatening. The cause of the faint is a transient insufficiency of cerebral flow. LOC inevitably leads to loss of postural tone and the patient falling over.
A 54 year old man collapses suddenly as he is walking across the living room. His daughter who witnessed the collapse says he dropped suddenly became very pale and started to twitch for a few seconds. After she woke him up (with some difficulty) he became flushed. O/E his nervous system is normal.
A. Anaemia B. Vasovagal syncope C. TIA D. Cardiac arrhythmia E. Stroke F. Postural hypotension G. Myxoedema coma H. Carotid sinus sensitivity I. Hypoglycaemia J. Aortic stenosis K. Epilepsy L. Pulmonary stenosis
D. Cardiac arrhythmias - Stokes-Adams attacks are episodes of transient LOC due to sudden decreased cardiac output. Pallor prior to the attack and facial flushing due to reactive hyperemia after the attack is characteristic of a Stokes-Adams attack. The underlying cause is a cardiac arrhythmia such as complete heart block.
A 70 year old lady with a history of well-controlled diabetes, collapses when she gets out of bed in the morning. She regained conciousness and pulled the emegency cord in her bedroom to call for help. She is on medication for hypertension and her ECG is unremarkable
A. Anaemia B. Vasovagal syncope C. TIA D. Cardiac arrhythmia E. Stroke F. Postural hypotension G. Myxoedema coma H. Carotid sinus sensitivity I. Hypoglycaemia J. Aortic stenosis K. Epilepsy L. Pulmonary stenosis
F. Postural hypotension - Postural hypotension is a side effect of anti-hypertensives and is a common problem in the elderly. Diabetic autonomic neuropathy may also be a cause here. A good history should be enough to diagnose this.
A 71 year old man with long standing hypertension lives on his own. He is found by his son with marked right upper limb weakness and difficulities with speech. He is now incontinent of urine and has some personality changes.
A. Subdural haemorrhage B. Space-occupying lesion C. Depression D. Stroke E. Migraine F. Myasthenia gravis G. Extradural haemorrhage H. Arterial dissection I. Multiple sclerosis J. Peripheral neuropathy K. Polio
D. Stroke - This is a left sided stroke which may have some frontal lobe involvement consistent with his personality changes. It is important is perform a CT head exclude a haemorrhagic aetiology and consider thrombolysis with tPA if within the 4.5 hour window and there are no contraindications. Thrombolysis is done with alteplase at 10% bolus, 90% infusion at a dose of 0.9 mg/kg. Presentation after the 4.5 hour window is managed with aspirin. The Bamford/Oxford Stroke Classification subtypes ischaemic stroke according to vascular territory of infarction. After initial management, stroke care involves the ethos of an MDT environment with rehabilitation.
A 22-year-old female student recently lost her mother. Now she presents with a couple of weeks Hx of increasing weakness in her arm and legs, but also while chewing or talking. Her vision seemed blurred recently.
A. Subdural haemorrhage B. Space-occupying lesion C. Depression D. Stroke E. Migraine F. Myasthenia gravis G. Extradural haemorrhage H. Arterial dissection I. Multiple sclerosis J. Peripheral neuropathy K. Polio
F. Myasthenia gravis - Myasthenia gravis is an autoimmune condition with antibodies affecting the NMJ, mostly the nAChR at the post-synaptic muscle membrane. Although some have antibodies against MuSK, and there are other proteins involved. MG is characterised by muscle weakness which increases with exercise (fatigue, unlike Lambert-Eaton myasthenic syndrome). Commonly, presentations include diplopia and drooping eyelids like this patient, and there may also be SOB, proximal limb weakness, facial paresis and oropharyngeal weakness. MG is associated with thymic hyperplasia in 70% or thymoma in 10%, and these associations can also crop up in EMQs. There will be elevated serum AChR receptor antibody titres or MuSK antibodies. Electrophysiology will demonstrate a decremental response on repetitive nerve stimulation. Treatment includes anticholinesterases (pyridostigmine, and immunotherapy. Patients may also require a thymectomy. Some 15-20% may experience a myasthenic crisis (which needs mechanical ventilation). Do you know what the Tensilon test is and why edrophonium is given in this test?
A mother of a 16-year old is concerned as he dropped a few things from his hands in the past two weaks. He says his arm went weak. He also complained of progressive headaches, associated with nausea and vomitting. He also had some visual disturbances.
A. Subdural haemorrhage B. Space-occupying lesion C. Depression D. Stroke E. Migraine F. Myasthenia gravis G. Extradural haemorrhage H. Arterial dissection I. Multiple sclerosis J. Peripheral neuropathy K. Polio
B. Space occupying lesion - Intracranial space-occupying lesions are mostly caused by primary and metastatic tumours. The progressive headache associated with N&V, visual disturbances and focal neurology indicate a slowly growing space-occupying mass. This could be a brain tumour, which may be primary (originates within the cranium) or secondary (from a metastatic tumour found elsewhere). The signs of raised intracranial pressure are seen here, and include headache, altered mental status, nausear and/or vomiting and gait abnormality. Tumours may be benign or malignant and range from meningiomas and medulloblastomas to craniopharyngiomas and astrocytic brain tumours. Further investigation is warranted in this case.
A 56-year-old lady with T2DM presents with gradual onset of weakness of the left hand and right foot. On examination, you also notice reduced sensation in the lower legs. She tells you that she also suffers a burning and shooting pain in her feet.
A. Subdural haemorrhage B. Space-occupying lesion C. Depression D. Stroke E. Migraine F. Myasthenia gravis G. Extradural haemorrhage H. Arterial dissection I. Multiple sclerosis J. Peripheral neuropathy K. Polio
J. Peripheral neuropathy - This is by definition peripheral neuropathy. This is a case of diabetic neuropathy with sensory and motor deficits. This is a microvascular complication of DM and is characterised by peripheral nerve dysfunction. Pain is a common complaint such as the burning and shooting sensation this patient describes. Patient’s may also describe the pain as prickling or sticking. Examination should include peripheral pulses, reflexes and sensation to light touch with a 10g monofilament, vibration (128Hz tuning fork), pinprick and proprioception. The pain may be treated with medications like pregabalin (VGCC modulator) and duloxetine (serotonin-NA re-uptake inhibitor).
A 41-year-old lady feels unsteady on her feet. She also has visual problems for the past 6 weeks. She has been previously well and has no other medical problems.
A. Subdural haemorrhage B. Space-occupying lesion C. Depression D. Stroke E. Migraine F. Myasthenia gravis G. Extradural haemorrhage H. Arterial dissection I. Multiple sclerosis J. Peripheral neuropathy K. Polio
I. Multiple sclerosis - Multiple sclerosis is a demyelinating CNS condition which is characterised by 2 or more episodes of neurological dysfunction which are separated in both time and space. MS classically presents in white women aged 20-40 with temporary visual/sensory loss although any presentation can occur. MRI is a sensitive test but less specific than spinal MRI, however, spinal MRI is abnormal in fewer cases. Treatment aims at treating the attack, preventing future attacks and symptomatic treatment of problems like bladder dysfunction, pain and fatigue.
A 87-year-old lady had a seizure at home. Her husband says she has been drowsy for the last couple of days. She is otherwise in good health but she has recently started antihpertensive drugs. On examination, her skin turgor is reduced.
- Absence seizure
- Meningitis
- Jacksonian seizure
- Encephalitis
- Hypercalcaemia
- Hyponatraemia
- Hypocalcaemia
- Simple partial seizure
- Atonic seizure
- Tonic-clonic seizure
- Hyponatraemia - Hyponatraemia is defined by serum sodium
A 66-year-old was getting increasing confused over the last couple of days. She was admitted after a seizure. She appears distressed and is pyrexial with mild meningism but no rash. A CT head scan shows changes in the left temporal lobe and cerebral oedema.
- Absence seizure
- Meningitis
- Jacksonian seizure
- Encephalitis
- Hypercalcaemia
- Hyponatraemia
- Hypocalcaemia
- Simple partial seizure
- Atonic seizure
- Tonic-clonic seizure
- Encaphalitis - Encephalitis (brain parenchyma inflammation) is not to be confused with meningitis where the meninges is inflammed (although meningoencephalitis can exist). In encephalitis, there is an altered state of consciousness (this patient is confused) and may there also be focal neurology. There is also a fever (exceptions, however, are subacute sclerosing panencephalitis, VZV and HCV). Other typical features include headache and seizures. It is a medical emergency and people are the extremes of age are more at risk. The list of possible causes is massive. HSV encephalitis is characterised by temporal lobe changes. Hypodense lesions and mass effect can be seen in the temporal lobes on CT. An MRI is a better test for encephalitis although it is less available when compared to CT.
If you suspect this diagnosis, empiral IV acyclovir should be started before the results of any investigations are known. A large proportion are caused by HSV and empirical therapy is backed up by RCTs which show improved mortality. You can change the treatment if the cause is known, for instance, ganciclovir for CMV. You should look for a rash which may give a clue to the aetiology. For example, vesicular patterns are seen in VZV, HSV and enteroviruses, EBV causes a maculopapular pattern after ampicillin, Lyme disease pathognomically gives erythema migrans whereas erythema nodosum might make you think of TB. The patient may also have animal/insect bites and a careful and thorough history is important.
A 42-year-old lady has a seizure the day after her thyroidectomy surgery. She is previous well and taking no medication. She has never had a seizure before.
- Absence seizure
- Meningitis
- Jacksonian seizure
- Encephalitis
- Hypercalcaemia
- Hyponatraemia
- Hypocalcaemia
- Simple partial seizure
- Atonic seizure
- Tonic-clonic seizure
- Hypocalcaemia - Hypocalcaemia can develop as a complication of thyroid surgery due to the loss of parathyroid glands which produce PTH. This is the cause of this lady’s seizure. Trousseau’s sign can be seen which is carpal spasm when a blood pressure cuff is used for several minutes. Carpopedal spasm that occurs with hypocalcaemia is a painful spasm and could also be the presenting sign. Chvostek’s sign is twitching of the perioral muscles in response to tapping over the facial nerve at the ear. If urgent replacement is necessary, calcium gluconate can be given IV. It is preferred over calcium chloride as it causes less tissue necrosis if it leaks out. It is worth noting that digoxin may be ineffective until serum calcium is restored to normal.
A 21-year-old man is walking down the street to visit his friends while suddenly he falls to the ground unconscious. His body goes stiff and then he begins to jerk his arms. He becomes incontinent of urine.
- Absence seizure
- Meningitis
- Jacksonian seizure
- Encephalitis
- Hypercalcaemia
- Hyponatraemia
- Hypocalcaemia
- Simple partial seizure
- Atonic seizure
- Tonic-clonic seizure
- Tonic-clonic seizures - This is a tonic-clonic, generalised seizure. It is characterised by LOC and widespread motor tonic contractions followed by clonic jerking movements. There will characteristically be a suppressed level of arousal following the event. This may either reflect a primary generalised episode or a focal seizure with secondary generalisation. The main aim of acute treatment is to terminate the seizure and to protect the airway. Management always starts with basic life-support (like every acute emergency) and your ABCs. IV access needs to be established (bloods sent to the lab too and serum glucose measured to test for reversable causes of seizure activity – thiamine should also be given to the patient if there is any concern about deficiency and hypoglycaemia, for instance in alcohol abuse). The following are needed: ECG, pulse oximetry, ABG. IV lorazepam is the preferred initial therapy, though rectal diazepam can be used if there is no IV access. If BZDs fail to stop the seizure then phenytoin or fosphenytoin can be tried.
After the episode, MRI and EEG are essential in diagnosing an epilepsy syndrome. During the episode of generalised tonic-clonic activity, the EEG will show bilateral synchrony in the epileptiform activity. If this is a one-off seizure in which a provoking factor, such as electrolyte disturbance or hypoglycaemia, has been identified then there is no need for therapy for epilepsy. In unprovoked cases, this depends on history, examination, EEG and MRI. Treatment may not be needed the first time but after a second unprovoked instance, therapy is generally recommended.
A 10-year-old girl is not doing well at school, her teacher says she doesn’t concentrate and shows no interest. Her parents also noticed the girl has moments (around 10secs) when she just stares blankly, blinking and then returns to normal. It happens up to several times an hour.
- Absence seizure
- Meningitis
- Jacksonian seizure
- Encephalitis
- Hypercalcaemia
- Hyponatraemia
- Hypocalcaemia
- Simple partial seizure
- Atonic seizure
- Tonic-clonic seizure
- Abscence seizures - A typical absence seizure is characterised by behavioural arrest or staring, lasting 5-10 seconds, interrupting otherwise normal activity. It can be induced by hyperventilation. Absence seizures can also be atypical, where it is less clear when it begins and ends and is not usually precipitated by hyperventilation. The definitive test here is to do an EEG to determine the exact nature of the seizure. This will ensure appropriate treatment. Most are medically responsive and childhood absence epilepsy (CAE) tends to remit by adulthood. First line treatment is with ethosuximide, valproate or lamotrigine. Atypical seizures though, tend to be refractory to medical treatment and associated with mental retardation.
A 72-year-old arteriopath is brought into the A&E collapsed. After admission you realise he is blind although he denies it.
A. Normal pressure hydrocephalus B. Occipital Stroke C. Multiple Sclerosis D. Hypothyroidism E. Vascular dementia F. Lewy body dementia G. Parkinson's disease H. Pick's disease I. Azheimer's dementia
B. Occipital stroke - This is cortical blindness, which is the loss of vision in a normal eye which results from damage to the visual area in the occipital cortex. The lack of insight that they have lost vision is a phenomenon known as Anton’s syndrome. They are ‘cortically blind’ but claim, often confabulate and adamantly despite obvious evidence, claim that they are capable of seeing. The pupillary reflex to light is intact as it does not involve the cortex and fundscopy, if done, would also be normal.
A 74-year-old man feels unsteady on his feet. He is on no medication and has no PMH. O/E you notice his gait is slow with small steps. He has a resting tremor of his right hand, and you note the cogwheel rigidity of his upper limbs.
A. Normal pressure hydrocephalus B. Occipital Stroke C. Multiple Sclerosis D. Hypothyroidism E. Vascular dementia F. Lewy body dementia G. Parkinson's disease H. Pick's disease I. Azheimer's dementia
G. Parkinson’s disease - Parkinson’s is characterised by a resting tremor, rigidity, bradykinesia and postural instability. This patient has difficulty walking, a resting tremor (4-6 Hz at rest which dissipates with the use of limbs, with generally asymmetrical onset) and limbs oppose movement. This last point demonstrates rigidity, which shows as resistance to passive movement about a joint. There is often also cogwheeling, especially if there is a superimposed tremor. The patient may have other signs like a mask like face due to the loss of spontaneous facial movement, hypophonia and micrographia, and may walk around in a shuffling gait with a stooped posture. The diagnosis is clinical. Treatment is symptomatic in an MDT setting. Medical therapy includes MAO-B inhibitors and DA agonists, for example rasagiline and carbidopa/levodopa. There are other medical therapies depending on the specific symptoms the patient presents with.
A 75-year-old lady is brought to see her GP by her husband. He says that in the past couple of months she needs more and more assistance with everyday life as she is very forgetful, gets lost easily and cannot name objects at times. He is very upset as he says she is not the way she used to be. Examination and routine investigations are unremarkable although her MMSE is 19/30.
A. Normal pressure hydrocephalus B. Occipital Stroke C. Multiple Sclerosis D. Hypothyroidism E. Vascular dementia F. Lewy body dementia G. Parkinson's disease H. Pick's disease I. Azheimer's dementia
I. Alzheimers dementia - This is Alzheimer’s dementia which is a progressive irreversible disorder characterised by memory loss, loss of social function and dimished executive function. The MMSE is the most widely used screening test for cognitive function and a score
A 80-year-old gentleman became agitated recently, he has behavioural changes including sexual disinhibition. He is unable to take care of himself, being unable to plan or make judgements on even simplest matters. His MMSE is 25/30.
A. Normal pressure hydrocephalus B. Occipital Stroke C. Multiple Sclerosis D. Hypothyroidism E. Vascular dementia F. Lewy body dementia G. Parkinson's disease H. Pick's disease I. Azheimer's dementia
H. Picks disease - Pick’s disease (not to be confused with Niemann-Pick disease) is one cause of frontotemporal degeneration. If you are not aware of this then this question would have been difficult. In any case, you should have realised that this is frontotemporal dementia. This type of dementia presents primarily with disruption in personality and social conduct, or as a primary language disorder. Essentially, the patient will present with components of the dysexecutive syndrome. Almost 50% will also display parkinsonism and a subset also have MND. Treatment is supportive combining medications ranging from BZDs to antidepressants depending on symptoms, with community services and carer guidance.
A 72-year-old smoker, a known arteriopath is suffering from increasing behavioural problems and forgetfulness. His family is concerned as his state is deteriorating. This is confirmed by the decline of his MMSE from 25/30 six months ago to 18/30 today.
A. Normal pressure hydrocephalus B. Occipital Stroke C. Multiple Sclerosis D. Hypothyroidism E. Vascular dementia F. Lewy body dementia G. Parkinson's disease H. Pick's disease I. Azheimer's dementia
E. Vascular dementia - Vascular dementia is a chronic and progressive dementia with loss of brain parenchyma mainly due to causes such as infarction and small vessel changes. It is classically assumed to be a stepwise progression in symptoms, although a gradual course can also be seen. This patient is an arteriopath. It is the second most common cause of dementia in older people and there is a large overlap with Alzheimer’s with many patients having a mixed form. Treatment is of limited use and the best course of action is to target vascular risk factors at as early a stage as possible.
What does the ‘f’ in ‘fMRI’ stand for?
1. Functional 2. Fast 3. Ferrous 4. Frequency
- Functional - Functional MRI’s can highlight an area of the brain that is being used in certain tasks.
Status epilepticus is uncontrolled or serial seizure activity lasting more than 30 minutes.
Selected Answer:
True
False
True - It is a medical emergency, treated with IV or PR diazepam initially.
Which type of seizure is associated with impaired consciousness and feelings of unreality, deja vu, or depersonalisation?
Selected Answer:
- Tonic-clonic
- Complex partial
- Simple partial
- Absence
- Complex partial - The above are all features of a temporal lobe seizure, which is often complex but may be simple (ie. no impairment of consciousness).
Parkinson’s disease is characterised by a loss of dopaminergic neurons in which area below?
- Caudate nucleus
- Substantia nigra
- Thalamus
- Corpus callosum
- Substantia nigra - The substantia nigra is the main area affected in Parkinson’s disease.
Which of these is not a feature of Parkinson’s disease?
- Resting tremor
- Rigidity of limbs
- Hemiplegia
- Mask-like face
- Hemiplegia - Hemiplegia is seen in cerebrovascular disease or lesions affecting the motor cortex and pyramidal tracts.
Dopamine tablets can be given to patients to treat Parkinson’s disease.
True
False
False - Dopamine does not cross the blood-brain barrier. Therefore levodopa (L-dopa), its precursor is given, because this can cross the barrier. It is then processed to make dopamine.
MS affects peripheral nerves.
True
False
False - MS only affects the CNS.
MS is a demyelinating disease.
True
False
True - MS is characterised by inflammation and demyelination.
Which is the best type of scan to help confirm MS?
- MRI
- CT
- Skull X-ray
- Angiogram
- MRI - MRI scans of the brain or spine are the most useful imaging investigations.
A 70 year old man is found by his wife to have difficulty speaking and comprehending spoken language. There is also an inability to raise his right arm. He was fine an hour ago when his sister spoke to him on the phone.
- Right anterior circulation stroke
- Left anterior circulation stroke
- Left posterior circulation stroke
- Lacunar stroke
- Left anterior circulation stroke - Anterior circulation strokes are commonly associated with aphasia and loss of strength in the face and upper and/or lower extremities. The inability to raise the right arm points to a left sided lesion. Remember that UMNs innervate contralateral LMNs, which innervate ipsilateral muscles.
A 30 year old man with a history of alcohol abuse is found by the police lying on the side of the motorway. The man has severe ataxia on walking into A&E. On examination, he is clearly confused. There is horizontal gaze palsy with markedly impaired vestibulo-ocular reflexes.
- Alcohol withdrawal
- Viral encephalitis
- Miller-Fisher syndrome
- Wernicke’s encephalopathy
- Wernicke’s encephalopathy - Wernicke’s encephalopathy is caused by the acute deficiency of thiamine in a susceptible host. It is clinically underdiagnosed. The classic triad is of mental status changes, ophthalmoplegia and gait dysfunction, though this is present in only 10% of cases.
An 8 year old boy with no significant PMH comes into clinic with worried parents who report frequent unusual episodes over the past few months. The parents tell you their child will suddenly stop activity for 10 to 20 seconds around 5 times a day and will be found staring with minimal eyelid flutter. During this time, he is unresponsive to voice.
- Daydreaming
- Frontal epilepsy
- Partial seizure
- Absence seizure
- Absence seizure - This description is of a typical absence seizure which interrupts otherwise normal activity. This can be hyperventilation-induced. EEG is the definitive test and most cases are medically responsive.
A 28 year old woman presents with severe weakness in the legs and distal arms. She began to have foot drop in her pre-school years which has progressed through her teens. She finds walking up stairs difficult and has been wheelchair bound since the age of 20. You notice the lower leg and foot are atrophied and there is pes cavus.
- Diabetic neuropathy
- Charcot-Marie-Tooth disease
- Hereditary spastic paraplegia
- Motor neurone disease
- Charcot-Marie-Tooth disease - This history here points to Charcot-Marie-Tooth disease. There is pes cavus and distal atrophy of the legs here which are characteristic features. The progressive history from childhood suggests this genetic condition as the cause.
A 27 year old medical student complains of a throbbing pain on the left side of her head. This is accompanied by nausea and vomiting as well as photophobia. It lasts about 6 hours and beforehand, she claims she felt a tingling in her limbs.
- Subarachnoid haemorrhage
- Migraine
- Tension headache
- Meningitis
- Migrane - This is a classic migraine history with an aura of a tingling sensation, which is pathognomic of migraine. The pain tends to be described as throbbing or pulsing and tends to be disabling, and can be associated with N&V and photophobia.
A 54 year old man presents complaining of dizziness. He describes it as a sudden and severe spinning feeling which is precipitated by rolling over in bed onto his left side. Symptoms last under a minute and has occured every night for a month.
- Benign positional paroxysmal vertigo
- Meniere’s disease
- Vestibular neuritis
- Labyrinthitis
- Benign positional paroxysmal vertigo - The history of vertigo on rollong over in bed which lasts for a few seconds is suggestive here. Dix-Hallpike test would be diagnostic in this case and will demonstrate nystagmus and symptoms delayed by about 15s, peak in 20-30s and then decay with complete resolution of the episode of vertigo.
A hypertensive old man with a history of atrial fibrillation is brought into A&E by his carer who is worried that he is getting confused. The carer tells you that he has fallen over frequently over the last few months and is unstable on his feet. There is right-sided pronator drift and some right sided weakness.
- Brain tumour
- Subdural haematoma
- Concussive syndrome
- Benign intracranial hypertension
- Subdural haematoma - There is a history of falls here and confusion. A subdural haematoma occurs due to extra-axial blood collection between the dura and arachnoid layers surrounding the brain. Surgical treatment will likely be necessary in this case with neurological deficit.
An 18 month old child from rural India presents to the local clinic where you are based for your elective. He has weakness in his left leg. Approximately 2 weeks ago, he has had fever and diarrhoea which resolved without treatment. The boy’s mother only believes in herbal remedies. Examination reveals the left leg has decreased tone, power and reflexes.
- Guillan-Barre syndrome
- Poliovirus infection
- Transverse myelitis
- Mycobacterium tuberculosis
- Poliovirus infection - This is paralytic poliovirus infection. The mother here only believes in herbal remedies so this child has not been immunised. The gastrointestinal prodrome is common although non-specific. Examination findings here are consistent. While GBS may be preceded by gastroenteritis, the limb weakness is usually symmetrical and there may be autonomic dysfunction. There is no cure for poliovirus infection.
A 22 year old female presents with recurrent slurred speech which gets worse as she continues to talk. Her friends think she is retarded. She also has difficulty swallowing which worsens as she continues to eat. There is also some double vision.
- Myasthenia gravis
- Multiple sclerosis
- Chronic subdural haematoma
- Embolic stroke
- Myasthenia gravis - MG is characterised by muscle weakness that increases with exercise and improves on rest. It is a chronic autoimmune disorder of the post-synaptic membrane at the NMJ in skeletal muscle.
A 35 year old engineer with a 10 pack year history presents with cloudy vision and pain on movement of the left eye. He also has some problems discriminating colours. 3 weeks ago she had unilateral parathesias during exam time and also occasionally has some tingling in her arms and legs when she is stressed.
- Fibromyalgia
- Migraine
- Pituitary tumour
- Multiple sclerosis
- Multiple sclerosis - MS is a demyelinating CNS condition clinically defined by 2 episodes of neurological dysfunction which are separated in space and time. Smoking is a weak risk factor.
A 54 year old postman has had lower back pain for 2 years now. Over the past year he has experienced bilateral leg pain and heavy legs when walking. This pain is relieved by sitting down. He has now assumed a stooped posture when walking which makes the pain better.
- Spinal stenosis
- Lumbosacral disc herniation
- Peripheral vascular disease
- Spinal compression fracture
- Spinal stenosis - This is neurogenic claudication due to spinal stenosis. This is relieved by sitting or bending over, which widens the size of the spinal canal. This is the reason for the stooped posture when walking, which is classically described as the ‘shopping cart sign’ where patients lean forwards like onto a shopping trolley to flex the spine.
48 yr old man presents with central chest pain on unusual exertion. Resting ECG is normal and there are no obvious risk factors. He would prefer not to take medication until a definitive diagnosis is made.
A. Coronary angiography B. Exercise ECG C. Beta blockers D. Thallium scan E. CABG F. Long acting nitrates G. Angioplasty H. Ace inhibitors I. Nifedipine
B. Exercise ECG - This patient has presented with stable angina. Resting ECG is often normal however during exercise stress ECG (most often the Bruce Protocol) there will be ST segment depression during exercise indicative of ischaemia. Those unable to exercise to an adequate level may need stress myocardial perfusion imaging or stress echocardiography. 1st line treatment involves lifestyle changes and antiplatelet therapy with aspirin. Anti-anginal theray will also be given, first line being beta-blockade. Statin therapy, blood sugar control in diabetics and BP control with antihypertensives may also be necessary. Those with LMS disease, 3 vessel disease or a reduced EF may benefit from CABG. Single vessel disease may benefit from PCI.
55 yr old man is taking increasing doses of sublingual GTN for established stable angina. He also has COPD with a reduced PEFR. Coronary angiography has shown diffuse disease but he has refused intervention.
A. Coronary angiography B. Exercise ECG C. Beta blockers D. Thallium scan E. CABG F. Long acting nitrates G. Angioplasty H. Ace inhibitors I. Nifedipine
I. Nifedipine - First line anti-anginal therapy for stable angina is a beta blocker such as metoprolol. However, this patient has COPD and beta blockers are relatively contraindicated due to bronchospasm (even those considered to be cardioselective). 2nd line treatment is with a CCB such as nifedipine. Long acting nitrates can be used as additional therapy or in patients where beta blockers and CCBs are contraindicated.
60 yr old man with stable angina is awaiting surgery. He is on the highest tolerated dose of beta blocker and CCB but is still symptomatic. BP is 170/95 mmHg.
A. Coronary angiography B. Exercise ECG C. Beta blockers D. Thallium scan E. CABG F. Long acting nitrates G. Angioplasty H. Ace inhibitors I. Nifedipine
F. Long acting nitrates - Long acting nitrates such as isosorbide mononitrate or transdermal GTN is indicated as the patient is still symptomatic on beta blockers and CCBs. Appropriate nitrate-free periods will be needed to avoid tolerance. Severe hypotension may occur if combined with a phosphodiesterase-5 inhibitor.
50 yr old man presents with typical history of exertional angina with ischaemic changes on resting ECG. Coronary angiography shows 70% stenosis of the left anterior descending artery with no significant lesions elsewhere.
A. Coronary angiography B. Exercise ECG C. Beta blockers D. Thallium scan E. CABG F. Long acting nitrates G. Angioplasty H. Ace inhibitors I. Nifedipine
G. Angioplasty - Those with LMS disease, 3 vessel disease or a reduced EF may benefit from CABG. Most single vessel disease can be adequately managed with PCI.
For each of the following patients choose one of the options above as the single most appropriate means of reducing cardiovascular risk
A 62 year old man, 3 months after an MI, taking asprin, atenolol and simvistatin, whose echocardigram shows worsening left ventricular function
A. Weight reduction and increased physical activity
B. Angiotensin converting enzyme inhibitor therapy
C. Weight reduction and metformin therapy
D. Cholesterol lowering therapy with a statin
E. Reduced alcohol intake
F. Antihypertensive drugs
G. Aspirin therapy
H. Smoking cessation
B. Angiotensin converting enzyme inhibitor therapy - This patient has worsening LV function in line with heart failure. First line treatment is with an ACE inhibitor which reduces morbidity and mortality associated with the condition. All patients with LV dysfunction should receive ACE inhibitors, whether symptomatic or not. Caution should be taken if the patient has renal impairment, cardiogenic shock or hyperkalaemia. All patients with chronic heart failure will also receive a beta blocker such as carvedilol.
For each of the following patients choose one of the options above as the single most appropriate means of reducing cardiovascular risk
A 46 year old woman, normal blood pressure, cholesterol and blood sugar, body mass index 32
A. Weight reduction and increased physical activity
B. Angiotensin converting enzyme inhibitor therapy
C. Weight reduction and metformin therapy
D. Cholesterol lowering therapy with a statin
E. Reduced alcohol intake
F. Antihypertensive drugs
G. Aspirin therapy
H. Smoking cessation
A. Weight reduction and increased physical activity - This woman is obese (BMI greater than or equal to 30) and needs to lose weight. Central obesity has a greater correlation with co-morbidities than peripheral obesity so arguably waist circumference is a better indicator of risk than body mass index. The mainstay of treatment is with diet and exercise. This patient is obese and drug therapy can be considered as an adjunct. This is primarily with orlistat which inhibits fat absorption by inhibiting lipases. For weight loss, the recommended intake is 1200-1500 kcal/day for men and 1000-1200kcal/day for women, producing a 500-1000kcal/day deficit.
For each of the following patients choose one of the options above as the single most appropriate means of reducing cardiovascular risk
A 77 year old man, normal blood pressure, not diabetic, who has had 3 episodes of transient left sided weakness in the last month
A. Weight reduction and increased physical activity
B. Angiotensin converting enzyme inhibitor therapy
C. Weight reduction and metformin therapy
D. Cholesterol lowering therapy with a statin
E. Reduced alcohol intake
F. Antihypertensive drugs
G. Aspirin therapy
H. Smoking cessation
G. Aspirin therapy - A TIA is colloquially called a ‘mini stroke’ with symptoms typically lasting under an hour. An antiplatelet drug such as aspirin is effective secondary prevention if the patient is not already anticoagulated. The patient will be anticoagulated if they have a likely or known cardioembolic source such as AF. Clopidogrel is an alternative in those who do not tolerate aspirin.
For each of the following patients choose one of the options above as the single most appropriate means of reducing cardiovascular risk
A 54 year old man, normal blood pressure, normal lipid profile, body mass index 28, random blood sugar 15 mmol/L, fasting blood sugar 8.5mmol/L
A. Weight reduction and increased physical activity
B. Angiotensin converting enzyme inhibitor therapy
C. Weight reduction and metformin therapy
D. Cholesterol lowering therapy with a statin
E. Reduced alcohol intake
F. Antihypertensive drugs
G. Aspirin therapy
H. Smoking cessation
C. Weight reduction and metformin therapy - This patient has DM. Symptomatic patients need a single random blood glucose of >11.1 or single fasting glucose of >7. Asymptomatic patients need two separate elevated readings for a diagnosis. Alternatively if there are borderline results, an OGTT can be conducted to see if plasma glucose is raised >11.1 two hours after an oral glucose load of 75g. A patient is said to have impaired fasting glucose if fasting glucose falls between 6.1-6.9. Impaired glucose tolerance is present if plasma glucose 2 hours after oral glucose load in OGTT falls between 7.8-11.0. First line intervention in this situation is diet and lifestyle advice and changes. Metformin will be added if there is no adequate response. In terms of this question, this option will reduce cardiovascular risk the most for this patient. Metformin is a biguanide and suppresses hepatic glucose production.
For each situation choose the single most likely diagnosis from the options. Each option may be used once, more than once or not at all.
30 yr old man attends for a routine pre employment medical. On examination of the CVS system the doctor finds a soft ejection systolic murmur at the apex. He has no previous cardiac or respiratory problems, and has normal pulse and BP
A. Aortic regurgitation B. Mitral incompetence C. Mixed mitral and ahortic valve disease D. Mitral stenosis - rheumatic E. Infective endocarditis F. Innocent murmur G.Aortic stenosis H. Hypertrophic obstructive cardiomyopathy I. Mixed aortic valve disease J. Mixed mitral valve disease K. Mitral regurgitation- rheumatic
F. Innocent Murmur - This is a functional murmur which is not caused by a structural cardiac defect. Functional murmurs tend to be systolic, occuring in an otherwise healthy individual with no symptoms. They are also characteristically position dependent and soft in nature. Ones that occur in children tend to disappear as the child grows. Benign paediatric murmurs include Still’s murmur.
For each situation choose the single most likely diagnosis from the options. Each option may be used once, more than once or not at all.
60 yr old Irish woman comes to see you with a progressive one year history of shortness of breath and recent onset of paroxysmal nocturnal dyspnoea. She has been previously well apart from Sydenham’s chorea as a child. She had six normal pregnancies. Examination= plethoric cheeks, pulse is 110 bpm irregular and small volume. BP 128/80mmHg. The JVP is normal. The apex is in the 5th i.c.s. and m.c.l and tapping in nature. The 1st heart sound is loud and P2 accentuated. A low pitched mid-diastolic murmur is heard in the apex.
A. Aortic regurgitation B. Mitral incompetence C. Mixed mitral and ahortic valve disease D. Mitral stenosis - rheumatic E. Infective endocarditis F. Innocent murmur G.Aortic stenosis H. Hypertrophic obstructive cardiomyopathy I. Mixed aortic valve disease J. Mixed mitral valve disease K. Mitral regurgitation- rheumatic
D. Mitral stenosis - rheumatic - Sydenham’s chorea (St Vitus Dance) are dancelike movements seen in rheumatic fever. The major criteria for rheumatic fever can be remember by CASES: carditis, arthritis, Sydenham’s chorea, erythema marginatum and subcutaneous nodules. Practically every single case of mitral stenosis is caused by rheumatic heart disease. The process tends to also cause regurgitation. This is characteristically a grade 1-2 low pitch murmur heard in mid-diastole which has a rumbling nature and there is no radiation. There can be an associated malar flush, tapping apex beat and a diastolic thrill palpable at the apex, in the 5th intercostal space in the MCL. The first heart sound is also characteristically loud and often this is the most striking feature on ascultation. It is a difficult murmur to pick up so if you are ever asked at this stage to spot this murmur, it will most likely be based on the loud S1.
For each situation choose the single most likely diagnosis from the options. Each option may be used once, more than once or not at all.
50 yr old man attends A&E with SOB, fever and hyperdynamic regular pulse of 100. BP 160/60 mmHg. He has a murmur at the left sternal edge. On further enquiry it is found he attended for a routine dental procedure 2 months ago.
A. Aortic regurgitation B. Mitral incompetence C. Mixed mitral and ahortic valve disease D. Mitral stenosis - rheumatic E. Infective endocarditis F. Innocent murmur G.Aortic stenosis H. Hypertrophic obstructive cardiomyopathy I. Mixed aortic valve disease J. Mixed mitral valve disease K. Mitral regurgitation- rheumatic
E. Infective endocarditis - Any patient presenting with fever and a new murmur should always make you think of bacterial endocarditis. The classic new or worsening murmur is actually rare. As are splinter haemorrhages, which this patient has. Other uncommon signs you may find include Janeway lesions (painless macular haemorrhagic plaques on the palms and soles) and Osler nodes (painful nodules on the pads of the fingers and toes). Roth spots may also be seen on fundoscopy. Three sets of bood cultures are required and this patient will have to go for an echocardiogram.The Duke criteria is used for diagnosis.
For each situation choose the single most likely diagnosis from the options. Each option may be used once, more than once or not at all.
80 yr old woman presents with recent onset of effort related chest pain. On examination= loud ejection systolic murmur and a low pulse pressure with a slow rising pulse.
A. Aortic regurgitation B. Mitral incompetence C. Mixed mitral and ahortic valve disease D. Mitral stenosis - rheumatic E. Infective endocarditis F. Innocent murmur G.Aortic stenosis H. Hypertrophic obstructive cardiomyopathy I. Mixed aortic valve disease J. Mixed mitral valve disease K. Mitral regurgitation- rheumatic
G. Aortic stenosis - Aortic stenosis can present with chest pain, dyspnoea and syncope. It is characterised by a harsh ejection systolic murmur heard loudest at the right upper sternal edge at end expiration, which radiates up towards the carotids. The pulse pressure is narrow and there may be an associated slow-rising and plateau pulse. Doppler echo is vital for diagnosis and shows a pressure gradient across the narrowed valve orifice. 20% of cases are due to a congenital bicuspid valve. The most common cause of aortic stenosis in adults is calcification of normal trileaflet valves. Clinically stable patients may be considered for surgical repair or TAVR.
For each situation choose the single most likely diagnosis from the options. Each option may be used once, more than once or not at all.
65 yr old man had an inferior MI 10 days ago. His initial course was uncomplicated. He suddenly deteriorates with LVF. On examination pulse is regular 100bpm and normal volume and character. BP 110/160mmHg. Apex beat is dynamic. There is a loud grade III apical pan-systolic murmur radiating to the axilla.
A. Aortic regurgitation B. Mitral incompetence C. Mixed mitral and ahortic valve disease D. Mitral stenosis - rheumatic E. Infective endocarditis F. Innocent murmur G.Aortic stenosis H. Hypertrophic obstructive cardiomyopathy I. Mixed aortic valve disease J. Mixed mitral valve disease K. Mitral regurgitation- rheumatic
B. Mitral incompetence - MR is loudest at the apex and radiates to the axilla and tends to be around grade 4. It is associated with a systolic thrill at the apex. TTE is the investigation of choice for diagnosis. Chronic MR is associated with a laterally displaced apex beat with LV dilatation. Mitral valve prolapse is a strong risk factor for development of MR.
Match the cause of hypotension to the following case histories. Each option may be used once, more than once or not at all.
33 year old woman complains of giddiness on standing & can no longer cross a road on her own as she is worried that she may pass out. She developed diabetes when age 12 & has had treatment to her eyes 2 years ago.
A. Addison’s disease B. Arrhythmia C. Drug induced D. Volume depletion E. Autonomic neuropathy F. Pulmonary embolus G. Blood loss H. Septicaemia I. Cardiogenic shock
E. Autonomic neuropathy - Autonomic neuropathy is a complication of diabetic neuropathy. Symptoms of autonomic neuropathy include… resting tachycardia (late findings due to vagal impairment), impaired HR variation, erectile dysfunction (affects many diabetic men though is not solely due to autonomic neuropathy), decreased libido and dyspareunia, orthostatic hypotension (measure BP supine and then standing after 1, 2, 3 and sometimes 5 minutes – an abnormal drop when standing is indicative) and urinary symptoms of frequency, urgency, incontinence, nocturia, weak stream and retention. Other symptoms include constipation, faecal incontinence and sweating dysfunction. Fludrocortisone may be helpful in this woman.
Match the cause of hypotension to the following case histories. Each option may be used once, more than once or not at all.
76 year old woman was admitted with confusion. She had been increasingly unable to care for herself. On admission, she was found to have cool peripheries & her blood pressure was 100/70. Blood results showed plasma urea 25mmol/l & plasma creatinine 120umol/l.
A. Addison’s disease B. Arrhythmia C. Drug induced D. Volume depletion E. Autonomic neuropathy F. Pulmonary embolus G. Blood loss H. Septicaemia I. Cardiogenic shock
D. Volume depletion - Volume depletion is a reduction in ECF volume due to salt and fluid losses which exceed intake. Causes include vomiting, bleeding, diarrhoea, diuresis and third space losses. Symptoms do not occur until large losses have alrady occured. Cool peripheries are a sign of peripheral shut down. Confusion may reflect poor cerebral flow or uraemia.Volume depletion has led to the low BP. Other symptoms include postural hypotension and tachycardia, weight loss and signs of shock. Serum urea and creatinine is elevated (you need to eyeball the patient when looking at creatinine – a very big body builder will have a much higher creatinine), indicating poor renal blood flow. This patient needs IV saline fluid replacement.
Match the cause of hypotension to the following case histories. Each option may be used once, more than once or not at all.
22 year old man presented with vomiting. He had not been feeling himself for some weeks. On examination, the skin creases of his hands were dark. Blood results showed plasma urea 8.5mmol/l, sodium 121mmol/l & potassium 5.1mmol/l.
A. Addison’s disease B. Arrhythmia C. Drug induced D. Volume depletion E. Autonomic neuropathy F. Pulmonary embolus G. Blood loss H. Septicaemia I. Cardiogenic shock
A. Addison’s disease - Hyperpigmentation in the palmar creases points towards Addison’s disease. Hyperpigmentation due to excess ACTH production can be mucosal or cutaneous and is more pronounced in the palms, knuckles and around scars. MSH is a byproduct of the production of ACTH from the cleavage of POMC. Sodium is low and potassium elevated. Vomiting is present in 75% of patients and nausea is a common finding. Additionally, postural hypotension may be present. The presence of other autoimmune diseases is a risk factor for the development of Addison’s.
Match the cause of hypotension to the following case histories. Each option may be used once, more than once or not at all.
45 year old man presented with severe chest pain radiating down his left arm. He was pale, cold & sweaty. Blood pressure was 80/50mmHg, pulse rate was 100 & regular. JVP was raised by 3cm & auscultation of the chest revealed basal creps. Over the next few hours, he became progressively short of breath despite being given intravenous diuretics. Chest x-ray showed signs of pulmonary congestion.
A. Addison’s disease B. Arrhythmia C. Drug induced D. Volume depletion E. Autonomic neuropathy F. Pulmonary embolus G. Blood loss H. Septicaemia I. Cardiogenic shock
I. Cardiogenic shock - Cardiogenic shock is pump dysfunction. This may occur, like in this case, after MI (shock complicates just under 10% of MIs) or may be due to cardiomyopathy, valve dysfunction or arrhythmias. This cause of shock in this patient is obviously apparent. Clinical signs of shock include stress responses of tachycardia and tachypnoea, hypotension (
Match the cause of hypotension to the following case histories. Each option may be used once, more than once or not at all.
67 year old man was observed to be very drowsy 12 hours after an aortic aneurysm repair. There had been considerable blood loss & he had been given 4 units of blood during surgery. He had been written up for pethidine 50-100mg 3 hourly postoperatively & had had 3 doses. BP had been 150/80 post-operatively & was now 100/60 with a pulse rate of 75/minute. Oxygen saturation was low at 85%.
A. Addison’s disease B. Arrhythmia C. Drug induced D. Volume depletion E. Autonomic neuropathy F. Pulmonary embolus G. Blood loss H. Septicaemia I. Cardiogenic shock
C. Drug induced - Opioid OD symptoms include CNS depression (drowsiness, sleepiness), respiratory depression and relative bradycardia. This patient needs ventilation prior to the administration of naloxone, titrated to patient response.
For each patient below, choose the SINGLE most useful investigation from the above list of options. Each option may be used once, more than once or not at all.
An 80 year old man with a history of ischaemic heart disease trips over a paving stone & fractures his hip. An ambulance takes him to A&E. 1 hour after arrival, he develops crushing central chest pain.
A. Upper GI endoscopy B. Thoracic spine x-ray C. ECG D. Coronary angiogram E. Transthoracic echo F. CT scan abdomen G. Chest x-ray H. Chest x-ray in expiration I. Barium swallow J. Chest x-ray rib views K. V/Q scan L. Exercise ECG M. CPK (creatine phosphokinase)
C. ECG - This patient’s crushing central chest pain sounds like an MI. Chest pain is classically severe and heavy in nature, located centrally with possible radiation to the left arm or jaw and lasts for >20 minutes. SOB due to pulmonary congestion and sweating due to high sympathetic output are common symptoms. Risk factors incorporate the standard set of cardiovascular risks such as smoking, high BP, DM, obesity and dyslipidaemia. An ECG is indicated. If ECG shows STEMI, new LBBB or confirmed posterior MI then PCI/thrombolysis is indicated. It is worth noting that RV infarction is present in 40% of inferior infarcts so if ST elevation is seen in II, III and aVF, right sided ECG leads should be obtained.
For each patient below, choose the SINGLE most useful investigation from the above list of options. Each option may be used once, more than once or not at all.
A 19 year old male medical student develops acute chest pain during a game of squash. On examination he is distressed there but examination is otherwise unremarkable. There is a family history of sudden death.
A. Upper GI endoscopy B. Thoracic spine x-ray C. ECG D. Coronary angiogram E. Transthoracic echo F. CT scan abdomen G. Chest x-ray H. Chest x-ray in expiration I. Barium swallow J. Chest x-ray rib views K. V/Q scan L. Exercise ECG M. CPK (creatine phosphokinase)
E. Transthoracic echo - This patient has likely HOCM. The patient’s young age makes it unlikely to be atherosclerotic coronary artery disease (unstable angina). HOCM is the most likely cardiomyopathy and the most frequent cause of sudden cardiac death in younger people. The FH of sudden death is suggestive of this diagnosis. This has a benign prognosis is most people though symptomatic patients are treated medically with beta blockade, CCBs or disopyramide. Chest pain on exertion (playing squash) is a common presentation in those who are symptomatic, as is dyspnoea on exertion, palpitations (such as due to AF) and a history of either pre-syncope or syncope (due to LV outflow obstruction). Inheritance is autosomal dominant with a variable penetrance.
Examination findings may be normal or may reveal an ejection systolic murmur which is positionally responsive and a double carotid or apex pulsation due to the transient interruption of CO. A fourth heart sound may also be heard due to hypertrophy. Echocardiography must be performed to establish a diagnosis, though ECG and CXR will also be done and may also show changes. Echo will show septal hypertrophy. There may also be MR.
For each patient below, choose the SINGLE most useful investigation from the above list of options. Each option may be used once, more than once or not at all.
A 55 year old obese female complains of a 6 month history of chest pain which radiated to the jaw and both shoulders. The pain is reported to be more severe at night. ECG and chest x-rays are normal.
A. Upper GI endoscopy B. Thoracic spine x-ray C. ECG D. Coronary angiogram E. Transthoracic echo F. CT scan abdomen G. Chest x-ray H. Chest x-ray in expiration I. Barium swallow J. Chest x-ray rib views K. V/Q scan L. Exercise ECG M. CPK (creatine phosphokinase)
A. Upper GI endoscopy - This patient has GORD characterised by heartburn and regurgitation of acid. It is more severe at night when the patient is lying flat and also when the patient is bending over. Risk factors include obesity and hiatus hernia. Diagnosis is generally clinical and can also be achieved by a diagnostic trial of a PPI. Normally an upper GI endoscopy is reserved for complications such as strictures, Barrett’s or cancer, or for atypical features. An OGD may show oesophagitis or Barrett’s (red velvety), however OGD may be normal. Manometry and pH monitoring may also be performed, but in this case, this patient will probably just have a therapeutic and diagnostic trial of a PPI instead of an OGD.
For each patient below, choose the SINGLE most useful investigation from the above list of options. Each option may be used once, more than once or not at all.
A 30 year old male alcoholic presents nausea and pain in the lower chest in a band radiating around to the back. The pain makes the patient curl up in a ball and movement worsens it. On examination there is decreased breath sounds on the left side which is stony dull to percussion at the base.
A. Upper GI endoscopy B. Thoracic spine x-ray C. ECG D. Coronary angiogram E. Transthoracic echo F. CT scan abdomen G. Chest x-ray H. Chest x-ray in expiration I. Barium swallow J. Chest x-ray rib views K. V/Q scan L. Exercise ECG M. CPK (creatine phosphokinase)
F. CT scan abdomen - This patient has acute pancreatitis. He has vomited and is describing mid-epigastric pain radiating around to the back which is relieved in the fetal position and is worse with movement. He is an alcoholic and alcoholic pancreatitis is seen more frequently in men usually after an average of 4-8 years of alcohol intake. Binge drinking also increases the risk. This patient also has nausea and may describe vomiting too, with agitation and confusion. The examination findings described here allude to a pleural effusion which is seen in half of patients with acute pancreatitis. Complicated haemorrhagic pancreatitis may exhibit Cullen’s sign, Grey-Turner’s sign and Fox’s sign. Make sure you know what these are and you are familiar with the causes of acute pancreatitis. Those caused by hypocalcaemia may display Chvostek’s sign and Trousseau’s sign.
Key to diagnosis is serum amylase or lipase levels which are massively elevated. Prognostic criteria are outlined in Ranson’s criteria applied on admission and after 48 hours, or the modified Glasgow score which you can find in your Oxford Handbook. An abdominal CT is however the most sensitive and specific study and findings may include enlargement of the pancreas with irregular contours, necrosis, pseudocysts and peripancreatic fat obliteration.
For interest, urinary trypsinogen-2 is now considered a better screening test than amylase but is not currently clinically used.
For each patient below, choose the SINGLE most useful investigation from the above list of options. Each option may be used once, more than once or not at all.
An 80 year old woman on corticosteroids develops acute chest pain while standing up. It is posteriorly sited, radiating anteriorly under the breast. The pain is worse on movement and there is tenderness on the back of the chest. Cardiovascular and respiratory examination are normal.
A. Upper GI endoscopy B. Thoracic spine x-ray C. ECG D. Coronary angiogram E. Transthoracic echo F. CT scan abdomen G. Chest x-ray H. Chest x-ray in expiration I. Barium swallow J. Chest x-ray rib views K. V/Q scan L. Exercise ECG M. CPK (creatine phosphokinase)
B. Thoracic spine x-ray - This patient likely has osteoporotic vertebral collapse/fracture which is compressing the intercostal nerve and causing her sudden pain in the back of her chest. The pain tends not to be related to traumatic activities and can occue on standing up or bending forwards, even coughing and sneezing. The history of corticosteroid use is associated with osteoporosis through multiple mechanisms. The thoracic spine plain x-ray would show loss of height in one of the thoracic vertebrae. Osteoporotic changes may also be seen such as osteopenia. Treatment depends on the extent of spinal involvement and the severity of the pain and whether it is complicated by spinal deformity such as kyphosis. For the diagnosis of osteoporosis, a DEXA scan is needed indicating a T score of less than or equal to -2.5.
A 62 year old gentleman presents with fatigue, breathlessness & anorexia. On examination his JVP is noted as being elevated, he has hepatomegaly & swollen ankles.
A. Atrial fibrillation B. Unstable angina C. Atrial flutter D. Left ventricular failure E. Congestive cardiac failure F. Myocardial infarction G. Decubitus angina H. Constrictive pericarditis I. Stable angina J. Bacterial endocarditis
E. Congestive cardiac failure - The key manifestation is dyspnoea and tiredness. CCF is a term used for patients who are breathless with oedema (signs of LVF and RVF). Elevated JVP, hepatomegaly and peripheral oedema are all signs of RVF. Initial investigations should include ECG, CXR, TTE and bloods including BNP levels. First line treatment is with an ACE inhibitor which reduces morbidity and mortality. Salt and fluid restriction is also beneficial. All patients with chronic heart failure will also receive a beta blocker such as carvedilol. Other adjuncts include spironolactone, diuretics, hydralazine and a nitrate, and digoxin. The NYHA classification criteria can be used based on symptoms to describe functional limitations and ranges from Class I to Class IV with symptoms occuring at rest. Many patients are asymptomatic for long periods of time because mild cardiac impairment is balanced by compensation.
A 55 year old gentleman with a history of systemic hypertension presents to A&E with breathlessness on exertion & orthopnoea. Examination reveals cardiomegaly & a displaced apex beat to the left.
A. Atrial fibrillation B. Unstable angina C. Atrial flutter D. Left ventricular failure E. Congestive cardiac failure F. Myocardial infarction G. Decubitus angina H. Constrictive pericarditis I. Stable angina J. Bacterial endocarditis
D. Left ventricular failure - This patient has no signs of RVF mentioned above. You should know the distinction between LVF and RVF. RVF leads to a backlog of blood and congestion of the systemic capillaries. This causes peripheral oedema and ascites and hepatomegaly may develop. Nocturia may be a symptom as fluid returns from the legs when the patient lies down flat. LVF causes congestion in the pulmonary circulation so the symptoms are respiratory. As seen in this patient, there is SOB on exertion and orthopnoea. This is why you can ask patients in a cardiac history how many pillows they sleep with. PND can also occur as well as ‘cardiac asthma’.
A diabetic, 66 year old lady presents to A&E with breathlessness, sweating, nausea & vomiting. She is feeling very distressed. She has no pain. On inspection she appears pale, sweaty & grey.
A. Atrial fibrillation B. Unstable angina C. Atrial flutter D. Left ventricular failure E. Congestive cardiac failure F. Myocardial infarction G. Decubitus angina H. Constrictive pericarditis I. Stable angina J. Bacterial endocarditis
F. Myocardial infarction - This diabetic is having a silent MI without chest pain. Silent MIs are more common in the elderly and those with DM probably due to autonomic neuropathy.
Tachycardia is a common feature of MI especially anterior wall MI. Chest pain of MI is classically severe and heavy in nature, located centrally with possible radiation to the left arm or jaw and lasts for >20 minutes. SOB due to pulmonary congestion and sweating due to high sympathetic output are common symptoms. This patient also has pallor which is due to a high sympathetic output. Risk factors incorporate the standard set of cardiovascular risks such as smoking, high BP, DM, obesity and dyslipidaemia. Patients with DM are at increased risk of CAD by a variety of mechanisms which are not fully known.
If ECG shows STEMI, new LBBB or confirmed posterior MI then PCI/thrombolysis is indicated. It is worth noting that RV infarction is present in 40% of inferior infarcts so if ST elevation is seen in II, III and aVF, right sided ECG leads should be obtained. Cardiac biomarkers include CK-MB and troponin.
A 49 year old man presents to A&E with a 2 week history of a ‘tight’ central chest pain radiating to the jaw experienced when he is lying down.
A. Atrial fibrillation B. Unstable angina C. Atrial flutter D. Left ventricular failure E. Congestive cardiac failure F. Myocardial infarction G. Decubitus angina H. Constrictive pericarditis I. Stable angina J. Bacterial endocarditis
G. Decubitus angina - Usually as a complication of heart failure. This patient has chest pain which occurs on lying down, which is decubitus angina by definition.
A 45 year old man comes to A&E with shortness of breath, giving a history of decreased exercise tolerance. On examination the patient is noted as having an irregular pulse, warm vasodilated peripheries, exopthalmos & a goitre.
A. Atrial fibrillation B. Unstable angina C. Atrial flutter D. Left ventricular failure E. Congestive cardiac failure F. Myocardial infarction G. Decubitus angina H. Constrictive pericarditis I. Stable angina J. Bacterial endocarditis
A. Atrial fibrillation - This patient has hyperthyroidism. More specifically, Graves’ disease (peripheral manifestations such as ophthalmopathy do not occur with other causes of hyperthyroidism). Treatment of Graves’ aims to normalise thyroid function and is achieved by radioactive iodine, antithyroid medications or with surgery. They are all effective and relatively safe options. Symptomatic therapy is given with beta blockers such as propranolol. This patient has AF which has occured as a result of his hyperthyroid state which affects around 10% of untreated patients. Irregular HR is the hallmark feature of AF. Have a think about what the ECG would show.
Mrs James is a 50 year old diabetic lady who complains of a tightness in her chest plus a cramping sensation in her jaw and neck after climbing 2 flights of stairs to her apartment.
A. Pericarditis B. Angina C. VSD D. Romano-Ward syndrome E. Infective endocarditis F. HOCM (hypertrophic obstructive cardiomyopathy) G. ASD H. Congestive cardiac failure I. MI J. Left ventricular failure
B. Angina - This patient has presented with stable angina. Resting ECG is often normal however during exercise stress ECG (most often the Bruce Protocol) there will be ST segment depression during exercise indicative of ischaemia. Those unable to exercise to an adequate level may need stress myocardial perfusion imaging or stress echocardiography. 1st line treatment involves lifestyle changes and antiplatelet therapy with aspirin. Anti-anginal theray will also be given, first line being beta-blockade. 2nd line is with a CCB. Long acting nitrates can be used as additional therapy or in patients where beta blockers and CCBs are contraindicated. Statin therapy, blood sugar control in diabetics and BP control with antihypertensives may also be necessary. Those with LMS disease, 3 vessel disease or a reduced EF may benefit from CABG. Single vessel disease may benefit from PCI.
A 45 year old man developed severe central chest pain, lasting for approximately 30 mins. He vomited with the pain, became acutely breathless & sweated profusely.
A. Pericarditis B. Angina C. VSD D. Romano-Ward syndrome E. Infective endocarditis F. HOCM (hypertrophic obstructive cardiomyopathy) G. ASD H. Congestive cardiac failure I. MI J. Left ventricular failure
I. MI - Chest pain of MI is classically severe and heavy in nature, located centrally with possible radiation to the left arm or jaw and lasts for >20 minutes. SOB due to pulmonary congestion and sweating due to high sympathetic output are common symptoms. N&V was believed to be associated with inferior wall infarction but this has been disputed. If ECG shows STEMI, new LBBB or confirmed posterior MI then PCI/thrombolysis is indicated. Risk factors incorporate the standard set of cardiovascular risks such as smoking, high BP, DM, obesity and dyslipidaemia.
Jean is a 35 year old lady who has recently given birth to her 3rd baby. 2 weeks ago she developed a fever, malaise & night sweats. She feels too tired to care for the baby & is losing weight. She attended her GP following the loss of control in her left arm. On examination there was weakness on the left side of her body. Her BP was normal, 120/80. Urinalysis showed small amounts of blood & auscultation revealed a loud pansystolic murmur plus bilateral basal crepitations.
A. Pericarditis B. Angina C. VSD D. Romano-Ward syndrome E. Infective endocarditis F. HOCM (hypertrophic obstructive cardiomyopathy) G. ASD H. Congestive cardiac failure I. MI J. Left ventricular failure
E. Infective endocarditis - Any patient presenting with fever and a new murmur should always make you think of bacterial endocarditis. The classic new or worsening murmur is actually rare. Other uncommon signs you may find include splinter haemorrhages, Janeway lesions (painless macular haemorrhagic plaques on the palms and soles) and Osler nodes (painful nodules on the pads of the fingers and toes). Roth spots may also be seen on fundoscopy. Septic embolic are common in IE and urinalysis may show active sediment. Three sets of bood cultures are required and this patient will have to go for an echocardiogram.The Duke criteria is used for diagnosis.
John is a 20 year old man with breathlessness and chest pain after exertion. 5 days ago he developed palpitations & fainted during a game of squash. Examination revealed a jerky pulse and a systolic murmur. His ECG showed changes of left ventricular hypertrophy.
A. Pericarditis B. Angina C. VSD D. Romano-Ward syndrome E. Infective endocarditis F. HOCM (hypertrophic obstructive cardiomyopathy) G. ASD H. Congestive cardiac failure I. MI J. Left ventricular failure
F. HOCM - This patient has HOCM. The patient’s young age makes it unlikely to be atherosclerotic coronary artery disease (unstable angina). HOCM is the most likely cardiomyopathy and the most frequent cause of sudden cardiac death in younger people. This has a benign prognosis in most people though symptomatic patients are treated medically with beta blockade, CCBs or disopyramide. Chest pain on exertion (playing squash) is a common presentation in those who are symptomatic, as is dyspnoea on exertion, palpitations (such as due to AF) and a history of either pre-syncope or syncope (due to LV outflow obstruction). Inheritance is autosomal dominant with a variable penetrance and there may be a FH of sudden death. Examination findings may be normal or may reveal an ejection systolic murmur which is positionally responsive and a double carotid or apex pulsation due to the transient interruption of CO. The arterial pulse is described as ‘jerky’. A fourth heart sound may also be heard due to hypertrophy. Echocardiography must be performed to establish a diagnosis, though ECG and CXR will also be done and may also show changes. Echocardiography will show septal hypertrophy. There may also be MR.
Mark developed a sharp sternal chest pain 10 days after a flu-like illness. The pain radiated down the arm & to the left shoulder. It was aggravated by lying flat, inspiration, coughing & swallowing. It was relieved by sitting forward.
A. Pericarditis B. Angina C. VSD D. Romano-Ward syndrome E. Infective endocarditis F. HOCM (hypertrophic obstructive cardiomyopathy) G. ASD H. Congestive cardiac failure I. MI J. Left ventricular failure
A. Pericarditis - This patient has presented with pericarditis. Symptoms include a sharp and severe chest pain retrosternally which is worse on inspiration and when supine, relieved by sitting forwards. The classical finding on examination is a friction rub which is said to sound like ‘walking on snow’. There may be diffuse ST elevations on ECG, an effusion on echocardiography and blood results suggesting inflammation. Complications include tamponade and constrictive pericarditis. The prior viral infection is a risk factor with the most common pericardial infection being viral. Bacterial purulent pericarditis also occurs. The inflammation is due either to direct viral attack or immune mediated damage. Other risk factors include male gender, post-MI (both ‘early’ and Dressler’s), post-pericardiotomy syndrome, neoplasm from local tumour invasion, uraemia and autoimmune conditions such as RA and SLE.
A 45 year old Caucasian lady presents with a 4cm chronic ulcer on the medial aspect of the lower leg. She has a history of “bursting” pain in the calf on walking. The skin around the ulcer is brown & heavily indurated.
A. RA B. Arterial ulcer C. Malignant ulcer D. Neuropathic ulcer E. Cardiac failure F. Lymphoedema G. Cellulitis H. Syphilis I. TB J. Pyoderma gangrenosum K. Venous ulcer L. DVT
K. Venous ulcer - Venous ulcers occur on a background of deep venous insufficiency. There is oedema and a brown skin discolouration due to leaching of pigments and haemosiderin deposition. In addition there may be lipodermatosclerosis and an inflammatory response, which is seen as an eczema-like thickening and hardening of the skin. The skin can also be drawn tightly around the ankle. Ulceration usually follows trauma and is usually on the medial gaiter region. The base has granulation tissue and is sloughy in nature and there is a sloping edge to the ulcer. The shape is often irregular. Look up some photos to help you remember. Once significant arterial disease is excluded (ulcers can have mixed components), the mainstay of treatment is with compression bandaging, appropriate dressings and treatment of any infection with antibiotics. Maggots can also be used and varicose veins should be treated where possible to reduce recurrence. If the ulcer is not healing, a biopsy should be considered (Marjolin’s ulcer).
A 75 year old woman with long standing hypertension has had progressive swelling of her legs over the last 3 months. She has consulted her GP because she has developed an ulcer on the anterior aspect of the right shin which weeps serous fluid profusely. She wonders what caused this ulcer?
A. RA B. Arterial ulcer C. Malignant ulcer D. Neuropathic ulcer E. Cardiac failure F. Lymphoedema G. Cellulitis H. Syphilis I. TB J. Pyoderma gangrenosum K. Venous ulcer L. DVT
E. Cardiac failure - This sounds like a venous ulcer due to right-sided heart failure. Venous ulcers are mainly caused by either venous insufficiency or heart failure. RVF will lead to peripheral oedema which may be complicated by a venous ulcer. Initial investigations should include ECG, CXR, TTE and bloods including BNP levels. First line treatment is with an ACE inhibitor which reduces morbidity and mortality. Salt and fluid restriction is also beneficial. All patients with chronic heart failure will also receive a beta blocker such as carvedilol. Other adjuncts include spironolactone, diuretics, hydralazine and a nitrate, and digoxin.
A 50 year old Asian woman, who is known to be diabetic, presents with a painless ulcer on the ball of her foot. She has been complaining of a burning feeling of the soles of her feet for the last year.
A. RA B. Arterial ulcer C. Malignant ulcer D. Neuropathic ulcer E. Cardiac failure F. Lymphoedema G. Cellulitis H. Syphilis I. TB J. Pyoderma gangrenosum K. Venous ulcer L. DVT
D. Neuropathic ulcer - This is a case of diabetic neuropathy. This is a microvascular complication of DM and is characterised by peripheral nerve dysfunction. Pain is a common complaint such as the burning sensation this patient describes. Patient’s may also describe the pain as prickling or sticking. Complications range from the painless neuropathic ulcer described, at areas of the foot where there is weight loading (particularly the metatarsal heads), to the Charcot foot with severe architectural destruction of the foot. Foot ulceration is a common precusor to amputation. Foot care is crucial in DM. Examination should include peripheral pulses, reflexes and sensation to light touch with a 10g monofilament, vibration (128Hz tuning fork), pinprick and proprioception. The pain may be treated with medications like pregabalin and gabapentin.
A 70 year old man, with ischaemic heart disease & COPD, presents with an ulcer between the great & second toes on the right foot. This is associated with pain in the whole foot at night.
A. RA B. Arterial ulcer C. Malignant ulcer D. Neuropathic ulcer E. Cardiac failure F. Lymphoedema G. Cellulitis H. Syphilis I. TB J. Pyoderma gangrenosum K. Venous ulcer L. DVT
B. Arterial ulcer - Arterial ulcers are deep and painful with a well defined edge, usually found on the shin or foot. There may be local changes such as cold peripheries, loss of hair, dusky cyanosis and toenail dystrophy. On examination, peripheral pulses may be absent or reduced. An angiogram with contrast will define the lesion and determine whether it can be improved by surgical intervention. Pain often increases when your legs are at rest and elevated. They can occur between the webs of toes so it is important to always check these in your peripheral vascular examination.
A 30 year old woman with long-standing ulcerative colitis, which is in remission, presents with 2 areas of ulceration on the right mid-thigh.
A. RA B. Arterial ulcer C. Malignant ulcer D. Neuropathic ulcer E. Cardiac failure F. Lymphoedema G. Cellulitis H. Syphilis I. TB J. Pyoderma gangrenosum K. Venous ulcer L. DVT
J. Pyoderma gangrenosum - Pyoderma gangrenosum is mainly associated with IBD (UC more so than CD), RA and the myeloid blood dyscrasias. It causes necrotic tissue leading to deep ulcers, often found on the legs. There are dark red borders.
A 60 year old male smoker complains of severe central chest pain radiating to the left arm. This is post operational following a sigmoidectomy the previous day.
A. Variant angina B. Pulmonary embolus C. MI D. Anxiety E. Congestive heart failure F. Unstable angina G. Stable angina H. GORD
C. MI - NSTEMI is a common complication of surgical procedures and is often detected as a rise in cardiac markers in the days following surgery. Chest pain of MI is classically severe and heavy in nature, located centrally with possible radiation to the left arm or jaw and lasts for >20 minutes. SOB due to pulmonary congestion and sweating due to high sympathetic output are common symptoms. If ECG shows STEMI, new LBBB or confirmed posterior MI then PCI/thrombolysis is indicated. Risk factors incorporate the standard set of cardiovascular risks such as smoking, high BP, DM, obesity and dyslipidaemia, though there are variations in risk factors attributed to STEMIs and NSTEMIs.
A 40 year old obese male presents with a burning chest pain which is worsened by lying down.
A. Variant angina B. Pulmonary embolus C. MI D. Anxiety E. Congestive heart failure F. Unstable angina G. Stable angina H. GORD
H. GORD - This is a common condition and the patient here is complaining of heartburn and acid regurgitation. The diagnosis is easily made clinically as the symptoms the patient describes are worse on lying down or bending over. Typically a patient will describe a burning sensation after meals which is not exertional. Reflux of acid into the mouth can leave a sour taste. Aside from obesity, other strong risk factors include advanced age, family history and hiatus hernia. This patient will need a trial of a PPI which will both be therapeutic and diagnostic. Complications of GORD include stricture formation, Barrett’s and oesophageal carcinoma.
A 59 year old female is admitted to A&E with chest pain. The pain is central in origin and came on while she was watching television. The patient has a BMI of 34 and is a known hypertensive. Troponin and CK-MB are not elevated.
A. Variant angina B. Pulmonary embolus C. MI D. Anxiety E. Congestive heart failure F. Unstable angina G. Stable angina H. GORD
F. Unstable angina - This is UA characterised by chest pain at rest. ECG will typically show ST depression and T wave inversion. Acute management includes antiplatelets and antithrombotics to reduce damage and complications. Long term management aims at reducing risk factors. Key risk factors include obesity, hypertension, smoking, hyperlipidaemia, FH, DM and positive FH. People with diabetes may again present with atypical symptoms. Cardiac biomarkers will not be elevated although in a patient who has had an acute MI days earlier, troponin may remain elevated (remains elevated up to 10-14 days after release). All patients with presumed cardiac chest pain should in the first instance get oxygen, morphine and GTN with antiplatelet therapy in the absence of contraindications.
A 62 year old male complains of chest pain at rest. An ECG performed in A&E shows ST elevation. A subsequent angiogram with a provocative agent showed an exaggerated spasm of the coronary arteries.
A. Variant angina B. Pulmonary embolus C. MI D. Anxiety E. Congestive heart failure F. Unstable angina G. Stable angina H. GORD
A. Variant angina - Variant angina (Prinzmetal) is angina caused by coronary artery vasospasm rather than atherosclerosis. It occurs at rest and in cycles. Many patients will also have some degree of atherosclerosis although not in proportion to the severity of the chest pain experienced. ECG changes are of ST elevation (rather than depression) when the patient is experiencing an attack and a stress ECG will be negative. Patients with Prinzmetal angina are often treated for ACS and indeed, cardiac biomarkers may be raised as vasospasm can cause damage to the myocardium. The gold standard investigation is with coronary angiography and the injection of agents to try to provoke a spasm.
A 45 year old merchant banker is referred by her GP to the Rapid Access Chest Pain clinic. She is asked to perform the treadmill test & complains of chest pain 9 minutes into the test.
A. Variant angina B. Pulmonary embolus C. MI D. Anxiety E. Congestive heart failure F. Unstable angina G. Stable angina H. GORD
G. Stable angina - This patient has presented with stable angina. Resting ECG is often normal and the patient is asymptomatic. However during exercise stress ECG (most often the Bruce Protocol) there will be ST segment depression during exercise indicative of ischaemia and the patient will complain of chest pain. Those unable to exercise to an adequate level may need stress myocardial perfusion imaging or stress echocardiography. 1st line treatment involves lifestyle changes and antiplatelet therapy with aspirin. Anti-anginal theray will also be given, first line being beta-blockade. Statin therapy, blood sugar control in diabetics and BP control with antihypertensives may also be necessary. Those with LMS disease, 3 vessel disease or a reduced EF may benefit from CABG. Single vessel disease may benefit from PCI.
A 49 year old man with recent history of long-haul travel presents with shortness of breath & haemoptysis. He also complains of chest pain & ECG shows sinus tachycardia.
A. Pericarditis B. Pulmonary embolism C. MI D. Hiatus hernia E. Anxiety F. Angina G. Coronary artery disease H. Oesophageal spasm I. Tietze’s syndrome J. Dissecting aortic aneurysm
B. Pulmonary embolism - Patients with a high clinical suspicion of PE should be anticoagulated while waiting a definitive diagnosis unless contraindicated. The underlying pathophysiology is based on Virchow’s triad. SOB and chest pain are common symptoms and there may also be haemoptysis. This patient has recent air travel, which is actually a weak risk factor but seems to crop up a lot on EMQs. Strong risk factors include DVT, obesity, surgery in the past 2 months, prolonged bed rest, malignancy, previous VTE, pregnancy and the thrombophilias such as factor V Leiden. ECG may be normal, or may show tachycardia, new RAD, new RBBB or the classical S wave in I, Q wave with T inversion in III. Various clinical probability scores exist for PE and D-dimer can be used to exclude PE as a diagnosis.
A 53 year old lady complains of central “crushing” chest pain, sudden onset & spontaneous remission, with no attributable cause. She has no history of hypertension, current BP is 116/76.
A. Pericarditis B. Pulmonary embolism C. MI D. Hiatus hernia E. Anxiety F. Angina G. Coronary artery disease H. Oesophageal spasm I. Tietze’s syndrome J. Dissecting aortic aneurysm
E. Anxiety - The presence of anxiety does not exclude a cardiac cause and appropriate investigations are required even if the patient obviously has anxiety.The absence of an attributable cause and the sudden onset and spontaneous remission with no cardiac risk factors make this likely to be due to anxiety.
A 73 year old gentleman presents to A&E with sudden “tearing” chest pain, radiating to the back. The house officer on duty notices unequal arm pulses & BP.
A. Pericarditis B. Pulmonary embolism C. MI D. Hiatus hernia E. Anxiety F. Angina G. Coronary artery disease H. Oesophageal spasm I. Tietze’s syndrome J. Dissecting aortic aneurysm
J. Dissecting aortic aneurysm - The tearing chest pain suggests aortic dissection. There may also be interscapular pain with dissection of the descending aorta. Dissecting aneurysms are either type A, which involves the ascending aorta, or type B. Type A dissections require urgent surgery whereas type B can be managed medically if it is not complicated by end organ ischaemia. BP differential between the 2 arms is a hallmark feature. Pulse differences may also be present in the lower limbs. There may also be the diastolic murmur of AR in proximal dissections.
A 63 year old gentleman develops acute central chest pain, radiating down the left arm. He appears very pale & sweaty, & has a strong family history of ischaemic heart disease.
A. Pericarditis B. Pulmonary embolism C. MI D. Hiatus hernia E. Anxiety F. Angina G. Coronary artery disease H. Oesophageal spasm I. Tietze’s syndrome J. Dissecting aortic aneurysm
C. MI - Chest pain of MI is classically severe and heavy in nature, located centrally with possible radiation to the left arm or jaw and lasts for >20 minutes. SOB due to pulmonary congestion and sweating due to high sympathetic output are common symptoms. This patient also has pallor which is due to a high sympathetic output. Risk factors incorporate the standard set of cardiovascular risks such as smoking, high BP, DM, obesity and dyslipidaemia.
If ECG shows STEMI, new LBBB or confirmed posterior MI then PCI/thrombolysis is indicated. It is worth noting that RV infarction is present in 40% of inferior infarcts so if ST elevation is seen in II, III and aVF, right sided ECG leads should be obtained. Cardiac biomarkers include CK-MB and troponin. Troponins rise 4-6 hrs after onset of infarction and peak at 18-24 hours and may persist for 7-10 days.
A 67 year old man recovering from an inferior MI complains of sharp retrosternal chest pain. He comments that leaning forward provides relief of the pain. The attending medical student claims to have heard a “rub” on auscultation.
A. Pericarditis B. Pulmonary embolism C. MI D. Hiatus hernia E. Anxiety F. Angina G. Coronary artery disease H. Oesophageal spasm I. Tietze’s syndrome J. Dissecting aortic aneurysm
A. Pericarditis - This patient has presented with pericarditis. Symptoms include a sharp and severe chest pain retrosternally which is worse on inspiration and when supine, relieved by sitting forwards. The classical finding on examination is a friction rub which is said to sound like ‘walking on snow’. There may be diffuse (saddle-shaped) ST elevations on ECG, an effusion on echocardiography and blood results suggesting inflammation. Complications include tamponade and constrictive pericarditis. The prior viral infection is a risk factor with the most common pericardial infection being viral. Bacterial purulent pericarditis also occurs. The inflammation is due either to direct viral attack or immune mediated damage. Other risk factors include male gender, post-MI (both ‘early’ and Dressler’s), post-pericardiotomy syndrome, neoplasm from local tumour invasion, uraemia and autoimmune conditions such as RA and SLE.
There is a harsh pan-systolic murmur loudest at the lower left sternal edge & inaudible at the apex. The apex is not displaced.
A. Tricuspid stenosis B. Pulmonary stenosis C. Patent ductus arteriosus D. Aortic stenosis E. Atrial septal defect F. Mitral regurgitation G. Aortic regurgitation H. Tricuspid regurgitation I. Mitral valve prolapse J. HOCM K. Left ventricular aneurysm L. Aortic sclerosis M. Mitral stenosis N. Ventricular septal defect
H. Tricuspid regurgitation - The murmur of TR is a lower left parasternal systolic murmur (pansystolic, or less, depending on severity). The murmur commonly increases on inspiration (Carvallo’s sign). The apex is not displaced in TR whereas it may be displaced in MR. Key risk factors for TR include LVF, rheumatic heart disease, endocarditis and the presence of a permanent pacemaker. Patients typically present with SOB, periperal oedema and tiredness. There may also be abdominal distension. An enlarged and pulsatile liver is normally seen in severe TR and caused by a reversal in blood flow during systole. It is worth noting that mild to moderate TR is not necessarily abnormal and is present in many asymptomatic young adults.
There is a mid-systolic click and a soft late systolic murmur at the apex.
A. Tricuspid stenosis B. Pulmonary stenosis C. Patent ductus arteriosus D. Aortic stenosis E. Atrial septal defect F. Mitral regurgitation G. Aortic regurgitation H. Tricuspid regurgitation I. Mitral valve prolapse J. HOCM K. Left ventricular aneurysm L. Aortic sclerosis M. Mitral stenosis N. Ventricular septal defect
I. Mitral valve prolapse - This is mitral valve prolapse characterised by the mid-systolic click and the late systolic murmur, the combination of which is specific for mitral prolapse but not sensitive enough for diagnosis. It occurs when one or more leaflets of the mitral valve prolapse into the LA in systole. Dynamic manoevres such as Valsalva or squatting change the timing of the murmur and click. Patients also commonly suffer from palpitations. Diagnosis can be confirmed with an echocardiogram. Connective tissue diseases such as Marfan’s are key risk factors. The exact cause of MVP is unknown but genetic links have been identified and there is histological myxomatous degeneration in MVP. MR is a complication of MVP.
The pulse is slow rising. There is an ejection systolic murmur loudest at the right upper sternal border radiating to the carotids.
A. Tricuspid stenosis B. Pulmonary stenosis C. Patent ductus arteriosus D. Aortic stenosis E. Atrial septal defect F. Mitral regurgitation G. Aortic regurgitation H. Tricuspid regurgitation I. Mitral valve prolapse J. HOCM K. Left ventricular aneurysm L. Aortic sclerosis M. Mitral stenosis N. Ventricular septal defect
D. Aortic stenosis - Aortic stenosis can present with chest pain, dyspnoea and syncope. It is characterised by a harsh ejection systolic murmur heard loudest at the apex at end expiration, which radiates up towards the carotids. The pulse pressure is narrow and there may be an associated slow-rising and plateau pulse. Carotid parvus et tardus may also be present. Doppler echo is vital for diagnosis and shows a pressure gradient across the narrowed valve orifice. 20% of cases are due to a congenital bicuspid valve which experience abnormal shear stress. Bicuspid valves are more common in those with aortic co-arctation and Turner’s. The most common cause of aortic stenosis in adults is calcification of normal trileaflet valves. Clinically stable patients may be considered for surgical repair or TAVR. The murmur of aortic sclerosis is usually less than grade 2 and describes leaflet thickening without obstruction. The pulse character is normal in sclerosis as blood flow is normal. It is the beginning of a spectrum of calcific aortic disease with severe stenosis at the end of the spectrum.
The pulse is irregularly irregular and jerky in character. There is an ejection systolic murmur lessened by squatting loudest at the lower left sternal edge. There is a double apical impulse felt. The apex beat is not displaced.
A. Tricuspid stenosis B. Pulmonary stenosis C. Patent ductus arteriosus D. Aortic stenosis E. Atrial septal defect F. Mitral regurgitation G. Aortic regurgitation H. Tricuspid regurgitation I. Mitral valve prolapse J. HOCM K. Left ventricular aneurysm L. Aortic sclerosis M. Mitral stenosis N. Ventricular septal defect
J. HOCM - This patient has HOCM which is the most frequent cause of sudden cardiac death in younger people. Examination findings may be normal or may reveal an ejection systolic murmur which is positionally responsive and a double carotid or apex pulsation due to the transient interruption of CO. A fourth heart sound may also be heard due to hypertrophy. There may also be MR. The arterial pulse is described as ‘jerky’ and this patient is also in AF which warrants anticoagulation and anti-arrhythmics. Echocardiography must be performed to establish a diagnosis, though ECG and CXR will also be done and may also show changes. Echo will show septal hypertrophy. This has a benign prognosis in most people though symptomatic patients are treated medically with beta blockade, CCBs or disopyramide. Chest pain on exertion is a common presentation in those who are symptomatic, as is dyspnoea on exertion, palpitations (such as due to AF) and a history of either pre-syncope or syncope (due to LV outflow obstruction). Inheritance is autosomal dominant with a variable penetrance. It is worth noting that ventricular hypertrophy causes concentric hypertrophy i.e. the wall of the ventricle gets thicker inwards. Hence the apex beat is not displaced unlike in DCM.
There is a constant ‘machinery-like’ murmur throughout systole & diastole.
A. Tricuspid stenosis B. Pulmonary stenosis C. Patent ductus arteriosus D. Aortic stenosis E. Atrial septal defect F. Mitral regurgitation G. Aortic regurgitation H. Tricuspid regurgitation I. Mitral valve prolapse J. HOCM K. Left ventricular aneurysm L. Aortic sclerosis M. Mitral stenosis N. Ventricular septal defect
C. Patent ductus arteriosus - The ductus arteriosus is a fetal structure which normally closes within 2 days of birth. Persistence can result in heart failure and increased pressures in the pulmonary vasculature as blood is shunted from the aorta into the pulmonary artery. The classic murmur is known as a Gibson murmur or machinery murmur and is best heard in the left infraclavicular area, usually peaking in late systole and continuing into diastole. Maternal rubella infection in the first trimester is a predisposing risk factor for PDA. The definitive diagnostic test is an echocardiogram.
A 78 year old male with an ejection systolic murmur loudest at the aortic area and radiating to the neck.
A. Atrial septal defect B. Chemotherapy C. Aortic stenosis D. Mitral stenosis E. Aortic regurgitation F. Systemic hypertension G. Pulmonary hypertension H. Mitral valve prolapse I. Alcohol
C. Aortic stenosis - Aortic stenosis is characterised by a harsh ejection systolic murmur heard loudest at the right upper sternal border at end expiration, which radiates up towards the carotids. The pulse pressure is narrow and there may be an associated slow-rising and plateau pulse. Doppler echo is vital for diagnosis and shows a pressure gradient across the narrowed valve orifice. 20% of cases are due to a congenital bicuspid valve. The most common cause of AS in adults is calcification of normal trileaflet valves. Clinically stable patients may be considered for surgical repair or TAVR. Presentation includes chest pain, dyspnoea and syncope.
56 year old male with ankylosing spondylitis. Collapsing pulse was noted on peripheral pulse examination.
A. Atrial septal defect B. Chemotherapy C. Aortic stenosis D. Mitral stenosis E. Aortic regurgitation F. Systemic hypertension G. Pulmonary hypertension H. Mitral valve prolapse I. Alcohol
E. Aortic regurgitation - Aortic regurgitation is the leakage of blood back into the LV in diastole. The collapsing pulse is also known as a water hammer or Corrigan’s pulse and describes the rapid rise and quick ‘collapse’ of the arterial pulse resulting in a wide pulse pressure. The murmur in AR is early diastolic in mild cases and increases to pansystolic in severe cases. Risk factors include a bicuspid valve, rheumatic fever, endocarditis, anklylosing spondylitis and Marfan’s. Other commonly seen signs in EMQs, although uncommon in clinical practice include Traube’s, Quincke’s, Duroziez’s and de Musset’s sign. Occasionally although uncommon, an Austin Flint murmur may be heard which is a rumbling mid-diastolic murmur best heard at the apex, produced by the regurgitant jet hitting the LV endocardium. Its presence indicates severe AR and the absence of a loud S1 or an opening snap distinguishes this from the murmur of mitral stenosis.
65 year old female with a mid-systolic click and a late systolic murmur.
A. Atrial septal defect B. Chemotherapy C. Aortic stenosis D. Mitral stenosis E. Aortic regurgitation F. Systemic hypertension G. Pulmonary hypertension H. Mitral valve prolapse I. Alcohol
H. Mitral valve prolapse - This is mitral valve prolapse characterised by the mid-systolic click and the late systolic murmur, the combination of which is specific for mitral prolapse but not sensitive enough for diagnosis. It occurs when one or more leaflets of the mitral valve prolapse into the LA in systole. Dynamic manoevres such as Valsalva or squatting change the timing of the murmur and click. Patients also commonly suffer from palpitations. Diagnosis can be confirmed with an echocardiogram. Connective tissue diseases such as Marfan’s are key risk factors. The exact cause of MVP is unknown but genetic links have been identified and there is histological myxomatous degeneration in MVP.
44 year old diabetic with renal impairment. Fundoscopy revealed AV nipping, silver wiring and small haemorrhages.
A. Atrial septal defect B. Chemotherapy C. Aortic stenosis D. Mitral stenosis E. Aortic regurgitation F. Systemic hypertension G. Pulmonary hypertension H. Mitral valve prolapse I. Alcohol
F. Systemic hypertension - Fundoscopy clearly demonstrates changes associated with hypertensive retinopathy. There are 4 grades:
Grade 1: ‘Silver wiring’ and tortuous vessels,
Grade 2: Plus ‘AV nipping’,
Grade 3: Plus cotton wool spots (previously called soft exudates but they are not exudates) and flame haemorrhages,
Grade 4: Plus papilloedema
34 year old male complaining of headaches, anxiety attacks, recurrent sweating and postural dizziness.
A. Addisons Disease B. Doxazosin C. Renal artery stenosis D. Phenelzine E. Conns syndrome F. Coarctation of the aorta G. Patent ductus arteriosus H. Cushings disease I. Hyperthyroidism J. Phaeochromocytoma
J. Phaeochromocytoma - Phaeochromocytomas presents with paroxysmal episodes of palpitations, anxiety, excessive sweating, pallor and hypertension. The postural dizziness is thought to be due to a reduction in volume secondary to alpha stimulation. Episodic panic attacks are seen commonly in adrenaline producing phaeochromocytomas. Headaches occur in up to 90% of those symptomatic. It can be inherited in MEN2, von Hippel-Lindau syndrome and NF1. Diagnosis is based on raised urinary and serum catecholamines, metanephrines and normetanephrines. 24 hour urinary VMA will be elevated. CT is used to localise the tumour. Treatment includes medical with the use of phenoxybenzamine, phentolamine and surgical options. Surgical excision is carried out under alpha and beta blockade to protect against the release of catecholamines into circulation when the tumour is being manipulated. The 10% rule is often quoted: 10% are bilateral, 10% malignant, 10% extraadrenal and 10% hereditary.
23 year old female with a complaint of progressive weight loss, palpitations and frequent loose motions.
A. Addisons Disease B. Doxazosin C. Renal artery stenosis D. Phenelzine E. Conns syndrome F. Coarctation of the aorta G. Patent ductus arteriosus H. Cushings disease I. Hyperthyroidism J. Phaeochromocytoma
I. Hyperthyroidism - This woman has hyperthyroidism. Symptoms include those mentioned and heat intolerance, sweating, tremor and tachycardia. In countries where sufficient iodine intake is not an issue, Graves’ disease is the most common cause of hyperthyroidism. Treatment aims to normalise thyroid function and is achieved by radioactive iodine, antithyroid medications or with surgery. Symptomatic therapy is given with beta blockers such as propranolol.
54 year old asymptomatic male. A left paraumbilical bruit was noted on examination.
A. Addisons Disease B. Doxazosin C. Renal artery stenosis D. Phenelzine E. Conns syndrome F. Coarctation of the aorta G. Patent ductus arteriosus H. Cushings disease I. Hyperthyroidism J. Phaeochromocytoma
C. Renal artery stenosis - This is a renal bruit which is best heard in the MCL at the costal margin. The finding of a bruit in the abdomen should make you suspicious of renal artery stenosis. RAS is due, typically, to atherosclerosis and often presents with hypertension and worsening renal function. A form of imaging is required for diagnosis.
17 year old male with radiological apperance of rib notching on chest radiograph.
A. Addisons Disease B. Doxazosin C. Renal artery stenosis D. Phenelzine E. Conns syndrome F. Coarctation of the aorta G. Patent ductus arteriosus H. Cushings disease I. Hyperthyroidism J. Phaeochromocytoma
F. Coarctation of the aorta - Aortic coarctation is characterised by a BP difference between the upper and lower extremities. Posterior rib notching is due to enlargement of collateral vessels due to aortic narrowing. Diagnosis is made on demonstrating narrowing of the aortic arch, typically shown by echocardiography. Treatment may involve surgical repair such as the placement of a stent. This condition is typically congenital with a male predominance. It is commonly detected in the first decade and is associated with Turner’s and DiGeorge. An ejection systolic murmur is also common present over the LSB and back.
18 year old female with progressive weight gain and gradual development of bitemporal hemianopia.
A. Addisons Disease B. Doxazosin C. Renal artery stenosis D. Phenelzine E. Conns syndrome F. Coarctation of the aorta G. Patent ductus arteriosus H. Cushings disease I. Hyperthyroidism J. Phaeochromocytoma
H. Cushings disease - There is weight gain in Cushing’s due to hypercorticolism. Gradual bitemporal hemianopia occurs due to an enlarging pituitary adenoma. Cushing’s disease is due to an ACTH secreting pituitary adenoma and is responsible for most cases of Cushing’s syndrome. A low dose 1mg overnight dexamethasone suppresion test can be done, or a 24 hour urinary free cortisol collection to diagnose Cushing’s syndrome. Plasma ACTH should guide further investigation. If ACTH is suppressed, the problem is likely to be with the adrenals. If it not suppressed, pituitary or ectopic disease is more likely. The treatment of choice in this case is surgical resection of the adenoma.
A 50 year old male smoker describes episodes of dull central chest pain on exertion lasting 10 minutes & relieved by rest.
A. Thallium perfusion scan B. Chest x-ray C. Ventilation/perfusion scan D. Coronary angiogram E. Abdominal ultrasound F. CT chest G. Sputum culture H. Upper GI endoscopy I. ECG J. Liver function tests K. Lower limb venogram L. Exercise ECG M. Arterial blood gases N. Creatine kinase
L. Exercise ECG - This patient has presented with stable angina. Resting ECG is often normal however during exercise stress ECG (most often the Bruce Protocol) there will be ST segment depression during exercise indicative of ischaemia. Those unable to exercise to an adequate level may need stress myocardial perfusion imaging or stress echocardiography. 1st line treatment involves lifestyle changes and antiplatelet therapy with aspirin. Anti-anginal theray will also be given, first line being beta-blockade. Statin therapy, blood sugar control in diabetics and BP control with antihypertensives may also be necessary. Those with LMS disease, 3 vessel disease or a reduced EF may benefit from CABG. Single vessel disease may benefit from PCI.
A 60 year old man who is waiting to have a knee replacement, describes daily episodes of central chest pain when he gets up in the morning. The pain lasts 15 minutes & settles with rest.
A. Thallium perfusion scan B. Chest x-ray C. Ventilation/perfusion scan D. Coronary angiogram E. Abdominal ultrasound F. CT chest G. Sputum culture H. Upper GI endoscopy I. ECG J. Liver function tests K. Lower limb venogram L. Exercise ECG M. Arterial blood gases N. Creatine kinase
D. Coronary angiogram - Variant angina (Prinzmetal) is angina caused by coronary artery vasospasm rather than atherosclerosis. It occurs at rest and in cycles. Many patients will also have some degree of atherosclerosis although not in proportion to the severity of the chest pain experienced. ECG changes are of ST elevation (rather than depression) when the patient is experiencing an attack and a stress ECG will be negative. Patients with Prinzmetal angina are often treated for ACS and indeed, cardiac biomarkers may be raised as vasospasm can cause damage to the myocardium. The gold standard investigation is with coronary angiography and the injection of agents to try to provoke a spasm.
A 25 year old pregnant woman, who has returned from Australia, developed sudden severe pleuritic pain and mild breathlessness. Her left leg was swollen yesterday, but not today.
A. Thallium perfusion scan B. Chest x-ray C. Ventilation/perfusion scan D. Coronary angiogram E. Abdominal ultrasound F. CT chest G. Sputum culture H. Upper GI endoscopy I. ECG J. Liver function tests K. Lower limb venogram L. Exercise ECG M. Arterial blood gases N. Creatine kinase
C. Ventilation/perfusion scan - This patient has a PE. The study of choice is a CTPA with direct visualisation of the thrombus. If there is a contraindication to a CT scan such as contrast allergy or pregnancy, then a V/Q scan is indicated. If a V/Q scan is not possible, alternatives such as MRA can be requested. It is worth noting that in patients with cardiopulmonary disease, these tests may not be accurate. A TTE can also be used to detect RV strain seen with PE.
A 30 year old man has had a 12 hour history of central chest pain, relieved by sitting forwards. He recently had a sore throat.
A. Thallium perfusion scan B. Chest x-ray C. Ventilation/perfusion scan D. Coronary angiogram E. Abdominal ultrasound F. CT chest G. Sputum culture H. Upper GI endoscopy I. ECG J. Liver function tests K. Lower limb venogram L. Exercise ECG M. Arterial blood gases N. Creatine kinase
I. ECG - This patient has presented with pericarditis. Symptoms include a sharp and severe chest pain retrosternally which is worse on inspiration and when supine, relieved by sitting forwards. The classical finding on examination is a friction rub which is said to sound like ‘walking on snow’. ECG is the investigation of choice with changes seen in 90%. There may be diffuse (saddle-shaped) ST elevations on ECG. Also, an effusion may be seen on echocardiography and blood results suggesting inflammation. Complications include tamponade and constrictive pericarditis. Risk factors include viral/bacterial infection (this patient’s sore throat), male gender, post-MI (both ‘early’ and Dressler’s), post-pericardiotomy syndrome, neoplasm from local tumour invasion, uraemia and autoimmune conditions such as RA and SLE.
82 year old man with hypertension for many years. He presents with increasing shortness of breath particularly when lying flat, & ankle swelling. On examination JVP is raised, BP 140/60, pulse 120/minute in atrial fibrillation.
A. Arterial doppler studies B. Lymphangiogram C. Chest x-ray D. Venous doppler studies E. 24 hour urine protein F. Pelvic ultrasound G. Liver function tests H. Plasma creatinine I. Coagulation screen J. Full blood count
C. Chest x-ray - The signs and symptoms this patient has points to CCF (congestive cardiac failure). This patient has a history of hypertension and is elderly. Other key cardiovascular risk factors include MI, DM and dyslipiaemia. SOB with orthopnoea due to the sudden increase in pre-load, indicates LV failure. Neck vein distension is also present, which is a major Framingham criteria for diagnosis. Tachycardia and ankle oedema are both minor criteria for diagnosis. Other major criteria for diagnosis include S3 gallop, cardiomegaly and hepatojugular reflux. For all patients, initial investigations should include ECG, CXR, TTE and bloods including BNP levels.
CXR may reveal pulmonary vascular redistribution to the upper zones, Kerley B lines, an increased CTR (cardiomegaly) and pleural effusion.
54 year old Asian woman with type 2 diabetes for 15 years. She comes to the clinic complaining of ankle swelling. On examination, BP 170/95, JVP not raised & bilateral oedema to the knees. Albumin is low.
A. Arterial doppler studies B. Lymphangiogram C. Chest x-ray D. Venous doppler studies E. 24 hour urine protein F. Pelvic ultrasound G. Liver function tests H. Plasma creatinine I. Coagulation screen
E. 24hr urine protein - The most common cause of nephrotic syndrome in adults with long standing diabetes is diabetic nephropathy. However, non-diabetic renal disease cannot be excluded. Nephrotic syndrome is defined by the presence of proteinuria (>3.5g/24h), oedema and hypoalbuminaemia. Some definitions add hyperlipidaemia. Do not confuse this with nephritic syndrome. Diagnosis is made by quantification of proteinuria with a 24 hour urine collection, although now it is common to do a spot urine protein-to-creatinine ratio for practical reasons.
65 year old woman with weight loss, malaise & ankle swelling. She smokes 20 cigarettes/day. There is a past history of irritable bowel syndrome. On examination; pulse 80/min irregularly irregular, JVP not seen, BP 135/85, clear chest, bilateral oedema & large mass in pelvis. Urine testing reveals protein +.
A. Arterial doppler studies B. Lymphangiogram C. Chest x-ray D. Venous doppler studies E. 24 hour urine protein F. Pelvic ultrasound G. Liver function tests H. Plasma creatinine I. Coagulation screen
F. Pelvic ultrasound - There is a large mass in the pelvis which is most likely malignant given the history. This needs to be investigated by pelvic ultrasound.
73 year old man was reviewed in the diabetic clinic. He was complaining of increasing tiredness & loss of appetite. His ankles had become more swollen over the last few weeks.
A. Arterial doppler studies B. Lymphangiogram C. Chest x-ray D. Venous doppler studies E. 24 hour urine protein F. Pelvic ultrasound G. Liver function tests H. Plasma creatinine I. Coagulation screen
H. Plasma creatinine - Diabetic patients are at risk of diabetic nephropathy and need to have their plasma creatinine regularly checked to monitor renal function. Tiredness, loss of appetite, confusion and pruritis can all be subtle signs of worsening renal function.
66 year old man presents with swelling of his right leg to the knee. He had had a right hip replacement 5 weeks previously.
A. Arterial doppler studies B. Lymphangiogram C. Chest x-ray D. Venous doppler studies E. 24 hour urine protein F. Pelvic ultrasound G. Liver function tests H. Plasma creatinine I. Coagulation screen
D. Venous doppler studies - Recent surgery, especially orthopaedic surgery, is a strong risk factor for developing a DVT. Other strong risks include active malignancy, pregnancy, obesity and coagulopathies such as factor V Leiden. A Wells score is determined in all patients with a suspected DVT with the condition being likely if the score is 2 or more. If the Wells score is
A man who works in the city suffers from burning, retrosternal discomfort radiating from epigastrium to jaw & throat. Worse on lying down.
A. BNP level B. ultrasound scan C. CTPA D. Exercise ECG E. Upper GI endoscopy F. MRI scan G. CT scan H. V/Q scan I. Chest X-ray
E. Upper GI endoscopy - This patient has GORD characterised by heartburn and regurgitation of acid. It is more severe at night when the patient is lying flat and also when the patient is bending over. Risk factors include obesity and hiatus hernia. Diagnosis is generally clinical and can also be achieved by a diagnostic trial of a PPI. Normally an upper GI endoscopy is reserved for complications such as strictures, Barrett’s or cancer, or for atypical features. An OGD may show oesophagitis or Barrett’s (red velvety), however OGD may be normal. Manometry and pH monitoring may also be performed, but in this case, this patient will probably just have a therapeutic and diagnostic trial of a PPI instead of an OGD.
A gentleman suffers from intermittent episodes of nausea, sweating, central crushing pain, radiating to jaw, lasting a few minutes, which is made worse by exercise.
A. BNP level B. ultrasound scan C. CTPA D. Exercise ECG E. Upper GI endoscopy F. MRI scan G. CT scan H. V/Q scan I. Chest X-ray
D. Exercise ECG - During exercise stress ECG (most often the Bruce Protocol) there will be ST segment depression during exercise indicative of ischaemia. Those unable to exercise to an adequate level may need stress myocardial perfusion imaging or stress echocardiography. 1st line angina treatment involves lifestyle changes and antiplatelet therapy with aspirin. Anti-anginal theray will also be given, first line being beta-blockade. Statin therapy, blood sugar control in diabetics and BP control with antihypertensives may also be necessary. Those with LMS disease, 3 vessel disease or a reduced EF may benefit from CABG. Single vessel disease may benefit from PCI.
A lady suffers from dyspnoea, coughing up blood as well which is mixed with frothy sputum and is stony dull to percuss.
A. BNP level B. ultrasound scan C. CTPA D. Exercise ECG E. Upper GI endoscopy F. MRI scan G. CT scan H. V/Q scan I. Chest X-ray
I. Chest X-ray - This patient has pulmonary oedema likely due to heart failure. There is also a pleural effusion which may be due to LVF. CXR may show pulmonary vascular redistribution to the upper zones, Kerley B lines, an increased CTR (cardiomegaly) and pleural effusion consistent with pulmonary oedema. Other investigations should include ECG, TTE and bloods including BNP levels. First line treatment for chronic heart failure is with an ACE inhibitor which reduces morbidity and mortality. Salt and fluid restriction is also beneficial. All patients with chronic heart failure will also receive a beta blocker such as carvedilol. Other adjuncts include spironolactone, diuretics, hydralazine and a nitrate, and digoxin.
30 year old women returning from holiday. Sudden onset chest pain with shortness of breath, coughed blood. She has no other lung disease.
A. BNP level B. ultrasound scan C. CTPA D. Exercise ECG E. Upper GI endoscopy F. MRI scan G. CT scan H. V/Q scan I. Chest X-ray
C. CTPA - This patient has a PE. Regardless of whether there is lung disease, guidelines state that the study of choice if there is an initial high probability of PE is a CTPA with direct visualisation of the thrombus (a filling defect is seen). If there is a contraindication to a CT scan such as contrast allergy or pregnancy, then a V/Q scan is indicated.
An older man collapses with sudden chest pain radiating to back.
A. BNP level B. ultrasound scan C. CTPA D. Exercise ECG E. Upper GI endoscopy F. MRI scan G. CT scan H. V/Q scan I. Chest X-ray
G. CT scan - A CT scan is indicated as soon as a diagnosis of aortic dissection is suspected and should be from the chest to the pelvis to see the full extent of the dissecting aneurysm. MRI is more sensitive and specific but is more difficult to obtain acutely.
Aortic dissection typically presents with tearing/ripping chest pain and classically radiates through to the back. There may be interscapular pain with dissection of the descending aorta. Dissecting aneurysms are either type A, which involves the ascending aorta, or type B. Type A dissections require urgent surgery whereas type B can be managed medically if it is not complicated by end organ ischaemia. BP differential between the 2 arms is a hallmark feature. Pulse differences may also be present in the lower limbs. There may also be the diastolic murmur of AR in proximal dissections.
70 yr old retired boiler maker presents with a 5 year history of exertional dyspnoea and a dry cough. The patient is non-smoker. Examination reveals fine crackles heard at the lung bases.
A. PE B. Chronic lymphatic leukaemia C. Pulmonary TB D. COPD E. Congestive cardiac failure F. Pulmonary fibrosis G. Acute lymphoblastic leukaemia H. Bronchopneumonia I. Iron deficiency anaemia J. Sarcoidosis K. Pernicious anaemia
F. Pulmonary fibrosis - Idiopathic pulmonary fibrosis (previously known as Cryptogenic fibrosing alveolitis) progresses over several years and is characterised by pulmonary scar tissue formation and dyspnoea. Patients complain of a non-productive cough and typically reproducible and predictable SOB on exertion. Boiler makers can come into contact with small organic or inorganic dust particles which is thought to be implicated in the cascade of events leading to IPF. While this patient does not smoke, another risk factor is cigarette smoking which significantly increases the risk of IPF. The mean age of diagnosis is 60-70. End expiratory basal crackles are found on examination. These are described as ‘Velcro-like’ in quality. IPF is also associated with clubbing.
25 yr old HIV positive man has a productive cough for the last 3 months with haemoptysis and night sweats. CXR shows hilar lymphadenopathy.
A. PE B. Chronic lymphatic leukaemia C. Pulmonary TB D. COPD E. Congestive cardiac failure F. Pulmonary fibrosis G. Acute lymphoblastic leukaemia H. Bronchopneumonia I. Iron deficiency anaemia J. Sarcoidosis K. Pernicious anaemia
C. Pulmonary TB - HIV infection is a key risk factor for pulmonary TB. It is important to have a high level of suspicion when evaluating patients with risk factors who present with suggestive symptoms. Night sweats, fever, malaise, cough, haemoptysis and erythema nodosum are all suggestive. In the first half of the 20th century, tuberculosis accounted for over 90% of cases of erythema nodosum. Other key risk factors for pulmonary TB include exposure to infection and returning from or being born in a high-risk region such as Asia, Africa and Latin America. If TB is suspected, the patient should be placed in isolation and a CXR obtained with 3 sputum samples cultured for AFB being the gold standard of diagnosis. Culture takes several weeks so sputum smears will be done before culture results are known. Interferon-gamma release assays (IGRAs) are now used by some hospitals to rapidly determine a patient’s TB status. All patients who have TB should be tested for HIV within 2 months of diagnosis.
12 yr old boy has become increasingly tired over the last month. On examination he looks pale, has a large bruise over his right thigh and a firm palpable liver and spleen.
A. PE B. Chronic lymphatic leukaemia C. Pulmonary TB D. COPD E. Congestive cardiac failure F. Pulmonary fibrosis G. Acute lymphoblastic leukaemia H. Bronchopneumonia I. Iron deficiency anaemia J. Sarcoidosis K. Pernicious anaemia
G. Acute lymphoblastic leukaemia - ALL presents in children with bone marrow involvement and the associated symptoms or in adults with an anterior mediastinal mass. Bone marrow infiltration leads to a pancytopenia leading to anaemia (reduced red blood cells), haemorrhage (reduced platelets) and infections (reduced mature white blood cells). In ALL, bone marrow is replaced by lymphoblasts. There may also be spread to CNS and testes. ALL is associated with Down’s syndrome. In contrast, CLL presents in older adults and is often asymptomatic, discovered by chance when a FBC is ordered. Smear/smudge cells are seen in peripheral blood smear. CLL is associated with a warm-type AIHA and there is peripheral blood lymphocytosis.
60 yr old publican smokes 20 a day. He has a 10 year history of having a ‘smoker’s morning cough’ when he expectorates clear sputum. This is worse in winter when it turns green and he has to go to his GP for antibiotics. Examination reveals poor air entry over both lung fields and his PEFR is 210 L/min (reduced by 60%)
A. PE B. Chronic lymphatic leukaemia C. Pulmonary TB D. COPD E. Congestive cardiac failure F. Pulmonary fibrosis G. Acute lymphoblastic leukaemia H. Bronchopneumonia I. Iron deficiency anaemia J. Sarcoidosis K. Pernicious anaemia
D. COPD - Smoking is the most important risk factor, accounting for 90% of COPD. COPD has an insidious onset and usually presents in older people with a history of cough, wheeze and SOB. This patient appears to have infective exacerbations of his COPD every winter. Patients with COPD are at a higher risk of infections and are vaccinated against influenza annually and pneumococcal pneumonia every 5 years. Spirometry is the gold standard for diagnosis, with FEV1/FVC ratio
A 58-year-old man, who smoked 30 cigarettes a day, presents with a 6-week history of cough, malaise, anorexia and weight loss. Past medical history includes hypertension for which he has taken lisinopril and bendrofluazide for 4 years.
A. Oesophageal reflux B. COPD C. Asthma D. Bronchiectasis E. Carcinoma of bronchus F. Sarcoidosis G. ACE inhibitor H. Postnasal drip I. Tuberculosis J. Foreign body
E. Carcinoma of the bronchus - The history of smoking and weight loss point to a bronchial carcinoma. Initial investigation is with a CXR. Diagnosis relies on pathological confirmation from a tissue sample, often obtained from bronchoscopy. First line treatment aims at surgical resection if possible. Small cell lung cancer is treated with chemotherapy and is associated with SIADH and ectopic ACTH. Non-small cell lung cancer is more often associated with clubbing. Squamous cell carcinoma is associated with PTHrp release and is treated with radiotherapy. Adenocarcinomas are usually located peripherally in the lung and are more common in non-smokers although most cases are still associated with smoking. The paraneoplastic syndromes may include Lambert-Eaton myasthenic syndrome.
A 45-year-old woman who smokes 25 cigarettes a day is reviewed in the diabetic clinic. She has had a dry cough for 2 months. She is on numerous tablets as her diabetes is complicated by microalbuminuria and hypertension. Her GP had given her a course of antibiotics 2 weeks previously.
A. Oesophageal reflux B. COPD C. Asthma D. Bronchiectasis E. Carcinoma of bronchus F. Sarcoidosis G. ACE inhibitor H. Postnasal drip I. Tuberculosis J. Foreign body
G. ACE inhibitor - A dry cough is a side effect of ACE inhibitors due to the build up of bradykinin which is normally degraded by ACE. ARB such as losartan will be indicated in this case. ARBs are insurmountable antagonists of AT1 receptors for angiotensin II, preventing its renal and vascular effects.
A 40-year-old Afro-Caribbean woman presents with bilateral parotid swelling, and painful nodules on the front of the shins. She has a dry cough and slight shortness of breath on exertion.
A. Oesophageal reflux B. COPD C. Asthma D. Bronchiectasis E. Carcinoma of bronchus F. Sarcoidosis G. ACE inhibitor H. Postnasal drip I. Tuberculosis J. Foreign body
F. Sarcoidosis - Sarcoidosis is a chronic multisystem disease with an unknown aetiology. The painful (mauve) nodules are erythema nodosum. Lupus pernio is another typical skin manifestation of sarcoidosis presenting with indurated plaques with discoloration on the face. Parotid enlargement is a classic feature (involvement of exocrine glands). The dry cough and SOB on exertion indicate pulmonary involvement. CXR will typically show bilateral hilar lymphadenopathy and CXR findings are used in the staging of disease. Additionally, serum calcium and ACE levels may be raised. A transbronchial biopsy is essential for diagnosis in most cases and shows the presence of non-caseating granulomas. Black people have a higher lifetime risk of sarcoidosis, as do those of Scandinavian origin. The mainstay of treatment for severe disease involves systemic corticosteroids.
An 18-year-old man presents with a night-time cough and shortness of breath while playing football. This has got progressively worse over the previous 2 months.
A. Oesophageal reflux B. COPD C. Asthma D. Bronchiectasis E. Carcinoma of bronchus F. Sarcoidosis G. ACE inhibitor H. Postnasal drip I. Tuberculosis J. Foreign body
C. Asthma - SOB and the cough, which may wake the patient from sleep combined with the patient’s age and progessive course suggest asthma. Examination can show an expiratory wheeze but may be normal and treatment is step-wise based on BTS guidelines. It is worth noting that in severe exacerbations, the chest may be silent. Night symptoms occur in more severe asthma and symptoms can be exacerbated by exercise. Diagnosis is supported by PEFR variation of at least 20% over 3 days in a week over several weeks or an increase of at least 20% to treatment. Stepwise treatment is outlined below. Look up the BTS guidelines for more information.
Step 1: SABA PRN, Step 2: Plus low-dose inhaled corticosteroids (ICS) , Step 3: Plus LABA, Step 4: Increase dose of ICS or add LTRA, SR theophylline or beta agonist tablet, Step 5: Daily steroid tablet and maintain ICS with specialist care.
A 30-year-old man, a lifelong non-smoker, presents with a history of at least 6 months of purulent sputum. He has had regular chest infections since an attack of measles at the age of 14.
A. Oesophageal reflux B. COPD C. Asthma D. Bronchiectasis E. Carcinoma of bronchus F. Sarcoidosis G. ACE inhibitor H. Postnasal drip I. Tuberculosis J. Foreign body
D. Bronchiectasis - Bronchiectasis is permanent bronchi dilatation due to bronchial wall damage and loss of elasticity. It is often as a consequence of recurrent/severe infections and most present with chronic productive mucopurulent cough. The most common identifiable cause is CF. Chest CT is the diagnostic test. Diagnosis is aided by sputum analysis. Have a think about what you would expect to hear on ascultation of the chest.
50 yr old male smoker presents with 3 month hx of cough, haemoptysis and wt loss. Chest examination = unremarkable
A. Fibrosing alveolitis B. TB C. Chronic bronchitis D. Pneumonia E. Influenza F. ACE inhibitor G. Asthma H. Extrinsic allergic alveolitis I. Right ventricular failure J. Left ventricular failure K. Bronchial carcinoma
K. Bronchial carcinoma - The history of smoking, weight loss, cough and haemoptysis point to a bronchial carcinoma. Initial investigation is with a CXR. Diagnosis relies on pathological confirmation from a tissue sample, often obtained from bronchoscopy. First line treatment aims at surgical resection if possible. Small cell lung cancer is treated with chemotherapy and is associated with SIADH and ectopic ACTH. Non-small cell lung cancer is more often associated with clubbing. Squamous cell carcinoma is associated with PTHrp release and is treated with radiotherapy. Adenocarcinomas are usually located peripherally in the lung and are more common in non-smokers although most cases are still associated with smoking. The paraneoplastic syndromes may include Lambert-Eaton myasthenic syndrome.
5yr old coughs most nights. He has frequent courses of antibiotics for a ‘bad chest’ especially in winter. He also has eczema.
A. Fibrosing alveolitis B. TB C. Chronic bronchitis D. Pneumonia E. Influenza F. ACE inhibitor G. Asthma H. Extrinsic allergic alveolitis I. Right ventricular failure J. Left ventricular failure K. Bronchial carcinoma
G. Asthma - These are infective exacerbations of asthma. The eczema is an atopic disease which is a strong risk factor for the development of asthma. A dry night time cough is commonly seen and suggestive. The hyperexpanded chest suggests persistent asthma. A Harrison’s sulcus, though rare, may also be seen. Treatment will be based on paediatric guidelines which differs from adult asthma. If you are really keen, you can look these up.
50 yr old male smoker has a productive cough with clear sputum most days, especially winter. He has not lost wt. On examination he has hyperexpanded chest and a few scattered wheezes and crackles.
A. Fibrosing alveolitis B. TB C. Chronic bronchitis D. Pneumonia E. Influenza F. ACE inhibitor G. Asthma H. Extrinsic allergic alveolitis I. Right ventricular failure J. Left ventricular failure K. Bronchial carcinoma
C. Chronic bronchitis - This patient has COPD, which is a progressive disease characterised by not fully reversible airflow limitation. COPD encompasses both emphysema and chronic bronchitis. Cigarette smoking is the most important risk factor. The hyperexpanded chest implies trapping of air due to incomplete expiration. Wheezes and coarse crackles are commonly seen in exacerbations. The cough is often the first symptom a patient complains of and is usually a morning event which is normally productive. The sputum can change quality with exacerbations/infection. Treatment aims at stopping smoking and vaccinating the patient against influence and pneumococcus with options such as bronchodilators or ICS. LTOT improves survival in those with severe COPD with a low PaO2. Lung function tests are key in diagnosis with an obstructive FEV1/FVC ratio
40 yr old Asian male has a 2 month hx of cough, haemoptysis, wt loss and night sweats. He has swollen cervical lymph nodes and his trachea deviated to the left.
A. Fibrosing alveolitis B. TB C. Chronic bronchitis D. Pneumonia E. Influenza F. ACE inhibitor G. Asthma H. Extrinsic allergic alveolitis I. Right ventricular failure J. Left ventricular failure K. Bronchial carcinoma
B. TB - It is important to have a high level of suspicion when evaluating patients with risk factors who present with suggestive symptoms. Night sweats, fever, malaise, weight loss, cough, haemoptysis and erythema nodosum are all suggestive. This patient’s tracheal deviation may be due to apical fibrosis or a cavitating lesion. The swollen lymph nodes in this patient’s neck may well represent a scrofula. Other key risk factors for pulmonary TB include exposure to infection and returning from or being born in a high-risk region such as Asia, Africa and Latin America. If TB is suspected, the patient should be placed in isolation and a CXR obtained with 3 sputum samples cultured for AFB being the gold standard of diagnosis. Culture takes several weeks so sputum smears will be done before culture results are known. Interferon-gamma release assays (IGRAs) are now used by some hospitals to rapidly determine a patient’s TB status. All patients who have TB should be tested for HIV within 2 months of diagnosis.
40 yr old man has a hx of hypertension. His anti-hypertensive meds were recently changed due to ankle swelling. He has now developed a dry cough.
A. Fibrosing alveolitis B. TB C. Chronic bronchitis D. Pneumonia E. Influenza F. ACE inhibitor G. Asthma H. Extrinsic allergic alveolitis I. Right ventricular failure J. Left ventricular failure K. Bronchial carcinoma
F. ACE inhibitor - A dry cough is a side effect of ACE inhibitors due to the build up of bradykinin which is normally degraded by ACE. ARB such as losartan will be indicated in this case. ARBs are insurmountable antagonists of AT1 receptors for angiotensin II, preventing its renal and vascular effects.
A 62 year old man presents with progressive breathlessness over many years. He worked in power stations. He has finger clubbing and his chest xray shows a honeycomb apperance.
A. Bronchiectasis B. Sarcoidosis C. Sinusitis D. Wegeners Granulomatosis E. Silicosis F. Tuberculosis G. Asbestosis H. Idiopathic pulmonary fibrosis I. Asthma J. Streptococcal pneumonia K. Mycoplasma pneumonia L. Cystic fibrosis
H. Idiopathic pulmonary fibrosis - Idiopathic pulmonary fibrosis (previously known as Cryptogenic fibrosing alveolitis) progresses over several years and is characterised by pulmonary scar tissue formation and dyspnoea. Patients complain of a non-productive cough and typically reproducible and predictable SOB on exertion. Work in power stations can involve contact with small organic or inorganic dust particles which is thought to be implicated in the cascade of events leading to IPF. Another risk factor is cigarette smoking which significantly increases the risk of IPF. The mean age of diagnosis is 60-70. End expiratory basal crackles are found on examination. These are described as ‘Velcro-like’ in quality. IPF is also associated with clubbing. A CXR in most will show reticulonodular shadowing consistent with fibrosis. This can be described as a ‘honeycomb’ pattern.
A 35 year old lady presents with a rash over her face and raised levels of ACE.
A. Bronchiectasis B. Sarcoidosis C. Sinusitis D. Wegeners Granulomatosis E. Silicosis F. Tuberculosis G. Asbestosis H. Idiopathic pulmonary fibrosis I. Asthma J. Streptococcal pneumonia K. Mycoplasma pneumonia L. Cystic fibrosis
B. Sarcoidosis - Sarcoidosis is a chronic multisystem disease with an unknown aetiology. The lesion described is lupus pernio which is a typical skin manifestation of sarcoidosis presenting with indurated plaques with discoloration on the face. Erythema nodosum is another dermatological manifestation. Additionally, serum calcium and ACE levels may be raised. CXR will typically show bilateral hilar lymphadenopathy and CXR findings are used in the staging of disease. A transbronchial biopsy is essential for diagnosis in most cases and shows the presence of non-caseating granulomas. Black people have a higher lifetime risk of sarcoidosis, as do those of Scandinavian origin. The mainstay of treatment for severe disease involves systemic corticosteroids.
A 28 year old homeless man presents with tiredness and cough for over 3 months. His CXR showed patchy shadows in the upper zones. His sputum grew positive culture on Lowenstein-jensen medium after 4 weeks.
A. Bronchiectasis B. Sarcoidosis C. Sinusitis D. Wegeners Granulomatosis E. Silicosis F. Tuberculosis G. Asbestosis H. Idiopathic pulmonary fibrosis I. Asthma J. Streptococcal pneumonia K. Mycoplasma pneumonia L. Cystic fibrosis
F. Tuberculosis - This patient should be placed in isolation. His sputum culture results suggest TB which is an AFB growing on Lowenstein-Jensen medium. A sputum culture is the most sensitive and specific test for TB and whilst growth takes a long time, the positive culture is diagnostic of TB. Sputum cultures will be repeated during treatment until 2 consecutive negative cultures. Treatment consists of anti-TB medication. Ethambutol should be stopped if the AFB is sensitive to isoniazid and rifampicin. Pyridoxine should be given with isoniazid to help prevent neuropathy. Pyrazinamide is not recommended in acute gout or pregnancy. Treatment of MDR TB involves additional therapy with agents such as amikacin and ethionamide.
A 19 year old presents with headache and malaise for 1 week. His chest was clear on auscultation. He subsequently developed a cough and was given erythomycin for 1 week.
A. Bronchiectasis B. Sarcoidosis C. Sinusitis D. Wegeners Granulomatosis E. Silicosis F. Tuberculosis G. Asbestosis H. Idiopathic pulmonary fibrosis I. Asthma J. Streptococcal pneumonia K. Mycoplasma pneumonia L. Cystic fibrosis
K. Mycoplasma pneumonia - The atypical presentation (a week of headache and malaise before the cough) and the prescription of erythromycin, a macrolide antibiotic, point to infection with an atypical pneumonia. However, depending on local prescribing policies, first line therapy for a CAP like pneumococcus may also be with a macrolide. Young people who ‘live together’ are commonly affected. The cough often does not resolve and is dry in nature. Symptoms tend to be prolonged and a low-grade fever is a common finding. Mycoplasma is the only atypical pneumonia on the list although there is no reason why this cannot be Legionella or Chlamydia. All can be treated with macrolides although in EMQs Chlamydia tends to be treated with doxycyline. All 3 atypicals are to some extent sensitive to fluoroquinolones and tetracylines too although these cannot be used in pregnancy. Whether they are first or second line therapy depends on the organism.
A newborn child presents with small bowel obstruction due to meconium ileus.
A. Bronchiectasis B. Sarcoidosis C. Sinusitis D. Wegeners Granulomatosis E. Silicosis F. Tuberculosis G. Asbestosis H. Idiopathic pulmonary fibrosis I. Asthma J. Streptococcal pneumonia K. Mycoplasma pneumonia L. Cystic fibrosis
L. Cystic fibrosis - CF can present in newborns with a failure to pass meconium (early stools) which can even result in bowel obstruction as in this patient. The bowel may even perforate if the patient is very unlucky resulting in a meconium peritonitis. The most conclusive diagnostic test is the sweat test which is pisitive if sweat chloride is >60mmol/L. Serum IRT from a heel prick blood spot allows screening of newborns. CF is a genetic condition with abnormal salt and water transport due to mutations in the CFTR (an apical anion channel). Heterozygotes generally do not demonstrate disease.
A 42 year old man has a 3 year history of progressive SOB and joint pains. His bloods show positive rheumatoid factor
A. Bronchiectasis B. Sarcoidosis C. Sinusitis D. Wegeners Granulomatosis E. Silicosis F. Tuberculosis G. Asbestosis H. Idiopathic pulmonary fibrosis I. Asthma J. Streptococcal pneumonia K. Mycoplasma pneumonia L. Cystic fibrosis
D. Wegeners Granulomatosis - Wegener’s is a systemic vasculitis affecting the small and medium vessels and presents with the classic triad of upper respiratory tract involvement, lower respiratory tract involvement and GN. It is a multisystem disease which can manifest with symptoms ranging from cutaneous, musculoskeletal, ocular and neurological features. Joint swelling and tenderness may all be present. It is associated with a positive cANCA and rhematoid factor is positive in around half.
A 38 year old woman with a history of multiple allergies is given intravenous contrast medium for a urogram. Within a couple of minutes, she has become breathless, with wheeze & stridor, & her blood pressure is 80/40 mmHg.
A. Acute anxiety B. Epiglotitis C. Exacerbation of COPD D. Inhaled foreign body E. Pneumothorax F. Anaphylaxis G. Left ventricular failure H. Viral pneumonia I. Asthma J. Pulmonary embolus
F. Anaphylaxis - There is a sudden onset of both respiratory and cardiovascular complaints with the recently given IV contrast agent. This patient is having an anaphylactic reaction and the airway needs to be promptly secured with prompt treatment with adrenaline. IM adrenaline must not be delayed and the anterolateral thigh is the preferred location, with repeated doses as necessary every 10 minutes or so. A 1:1000 solution is used of 0.3-0.5mg adrenaline. Fluid replacement with IV saline is also indicated to correct the intravascular volume redistribution.
A 69 year old man with a history of hypertension & stable angina wakes up severely short of breath. He is slightly more comfortable sitting upright. On examination, his blood pressure is 195/115 mmHg & there are crepitations at the bases of both lungs.
A. Acute anxiety B. Epiglotitis C. Exacerbation of COPD D. Inhaled foreign body E. Pneumothorax F. Anaphylaxis G. Left ventricular failure H. Viral pneumonia I. Asthma J. Pulmonary embolus
G. Left ventricular failure - This patient has LVF and pulmonary oedema. This accounts for the basal crepitations heard on ascultation and the SOB which is better on sitting upright. Patients need to be sat upright for this reason and IV access needs to be established. Oxygen, morphone, diuretics (frusemide or another loop diuretic) and nitrates will be given. Once stable, medical treatment of heart failure should be started which involves in the first instance, an ACE inhibitor followed by beta blockade. Ongoing diuretics are necessary if the patient has persistent symptoms of fluid overload.
A 73 year old woman is recovering in hospital 3 days after an operation to replace her right hip. On going to the toilet she suddenly becomes extremely short of breath & rapidly loses consciousness. Blood pressure is 60/20 mmHg.
A. Acute anxiety B. Epiglotitis C. Exacerbation of COPD D. Inhaled foreign body E. Pneumothorax F. Anaphylaxis G. Left ventricular failure H. Viral pneumonia I. Asthma J. Pulmonary embolus
J. Pulmonary embolus - This patient should be thrombolysed immediately if not contraindicated due to her haemodynamically unstable and critical state. Treatment should not be delayed in this obvious PE. This patient is now at a serious risk of cardiac arrest. Anticoagulation should also be started. The underlying pathophysiology is based on Virchow’s triad. SOB is a common symptom and there may also be chest pain and haemoptysis. This patient has had recent surgery, particularly orthopaedic surgery, which is a strong risk factor for PE. Other strong risk factors include DVT, obesity, prolonged bed rest, malignancy, previous VTE, pregnancy and the thrombophilias such as factor V Leiden. ECG may be normal, or may show tachycardia, new RAD, new RBBB or the classical S wave in I, Q wave with T inversion in III. Various clinical probability scores exist for PE and D-dimer can be used to exclude PE as a diagnosis. The study of choice is a CTPA with direct visualisation of the thrombus. If there is a contraindication to a CT scan such as contrast allergy or pregnancy, then a V/Q scan is indicated. If a V/Q scan is not possible, alternatives such as MRA can be requested. It is worth noting that in patients with cardiopulmonary disease, these tests may not be accurate. A TTE can also be used to detect RV strain seen with PE.
A 77 year old former coal miner has 30 year history of cough, mostly productive of sputum. He suddenly becomes breathless after a bout of coughing & complains of right sided chest pain. On examination, he is cyanosed, the trachea is deviated to the left & no breath sounds are audible over part of the right side of the chest.
A. Acute anxiety B. Epiglotitis C. Exacerbation of COPD D. Inhaled foreign body E. Pneumothorax F. Anaphylaxis G. Left ventricular failure H. Viral pneumonia I. Asthma J. Pulmonary embolus
E. Pneumothorax - This patient has developed a right sided tension pneumothorax and will need emergency intervention in the form of the insertion of a large bore cannula into the 2nd intercostal space in the MCL of the affected side. This will need to be followed by the insertion of a chest drain.
Otherwise if this was not a tension pneumothorax, for secondary spontaneous pneumothoraces, if large enough for a chest drain or the patient is clinically unstable, chest drain insertion is indicated. Simple aspiration success rate is reduced in secondary spontaneous pneumothoraces. Primary pneumothoraces occur in young people without known lung conditions. This patient has pulmonary fibrosis. Those who suffer recurrent pneumothoraces may have to undergo pleurodesis to stick the parietal and visceral pleural together by an inflammatory reaction.
A 25 year old man has a 3 day history of shivering, general malaise & productive cough. The x-ray shows right lower lobe consolidation.
A. Bacteroides fragilis B. Mycobacterium tuberculosis C. E coli D. Haemophilus influenzae E. Mixed growth of organisms F. Mycoplasma pneumoniae G. Staphylococcus aureus H. Pneumocystis jirovecii I. Legionella pneumophila J. Coxiella burnetii K. Streptococcus pneumoniae
K. Streptococcus pneumoniae - Classic lobar pneumonia with no signs and symptoms to suggest an atypical organism is most likely to due to pneumococcus. Symptoms include chills, fever, cough, SOB and pleuritic chest pain. A CXR is the most specific and sensitive test available and antibiotics are indicated.
A 26 year old man presents with severe shortness of breath and a dry cough which he has had for several weeks. He is an IV drug user. There are purple patches on the arms and in the mouth. CXR shows reticular perihilar opacities. Chest examination is unremarkable.
A. Bacteroides fragilis B. Mycobacterium tuberculosis C. E coli D. Haemophilus influenzae E. Mixed growth of organisms F. Mycoplasma pneumoniae G. Staphylococcus aureus H. Pneumocystis jirovecii I. Legionella pneumophila J. Coxiella burnetii K. Streptococcus pneumoniae
H. Pneumocystus jirovecii - PCP is caused by Pneumocystis jirovecii, previously called Pneumocystis carinii. It is a fungal organism and an AIDS defining illness. Signs and symptoms occur in a patient who is immunosuppressed especially HIV with a CD4 count
A 35 year old previously healthy man returned from a conference in the USA 5 days ago. He travels frequently and gives a 30 pack year history. He presents with mild confusion, a productive cough, diarrhoea and is pyrexic. His chest examination is normal. CXR shows infiltrates in the RUL.
A. Bacteroides fragilis B. Mycobacterium tuberculosis C. E coli D. Haemophilus influenzae E. Mixed growth of organisms F. Mycoplasma pneumoniae G. Staphylococcus aureus H. Pneumocystis jirovecii I. Legionella pneumophila J. Coxiella burnetii K. Streptococcus pneumoniae
I. Legionella pneumophila - Legionella is a gram negative rod. Legionella infecting the lungs is legionnaires’ disease or Legionella pneumonia whereas non-lung infection is known as Pontiac fever. This bacteria is found in aqueous environments such as lakes and almost all cases are from contaminated water systems, which relates to the risk factors of getting Legionella (recent water exposure like a hot tub). Smoking is also a risk factor. It can cause confusion as well as hyponatraemia, abdominal pain, diarrhoea and bradycardia. Legionella does not grow on routine culture media and diagnosis relies on urine antigen detection, serology or culture on special media.
A 20 year old previously healthy woman presents with general malaise, severe cough & breathlessness which has not improved with a 7 day course of amoxicillin. There is nothing significant to find on examination. The x-ray shows patchy shadowing throughout the lung fields. The blood film shows clumping of red cells with suggestion of cold agglutinins.
A. Bacteroides fragilis B. Mycobacterium tuberculosis C. E coli D. Haemophilus influenzae E. Mixed growth of organisms F. Mycoplasma pneumoniae G. Staphylococcus aureus H. Pneumocystis jirovecii I. Legionella pneumophila J. Coxiella burnetii K. Streptococcus pneumoniae
F. Mycoplasma pneumoniae - The cold agglutinins is what gives this question away. Mycoplasma is associated with with cold type agglutinins and a cold AIHA. Humans are thought to be the only host for Mycoplasma. The most commonly affected are young adults living in close proximity to each other. PCR can be used in diagnosis.
A 63 year old tramp presents to the A&E department with a 4 day history of haemoptysis. He has felt unwell for about 2 months with a cough, loss of weight & generalised weakness. He attributes his diplopia, which started a fortnight ago, to excessive alcohol consumption. On examination he has bilateral ptosis & proximal weakness in the limbs which improves on repeated testing.
A. Pulmonary metastases B. Pulmonary embolism C. Pulmonary abscess D. Small cell carcinoma E. Streptococcal pneumonia F. Squamous cell carcinoma G. Microscopic polyarteritis H. Tuberculosis I. Goodpasture’s disease J. Myaesthenia gravis K. Mesothelioma
D. Small cell carcinoma - First line treatment aims at surgical resection if possible. Small cell lung cancer is treated with chemotherapy and is also associated with SIADH and ectopic ACTH. Non-small cell lung cancer is more often associated with clubbing. Squamous cell carcinoma is associated with PTHrp release and is treated with radiotherapy. Adenocarcinomas are usually located peripherally in the lung and are more common in non-smokers although most cases are still associated with smoking. The paraneoplastic syndromes may include Lambert-Eaton myasthenic syndrome which this patient has (though weakness of the eye muscles is uncommon in Lambert-Eaton and is more prominent in myasthenia gravis). This classically presents with weakness which improves on repeated testing (in contrast to myaesthenia gravis) and is more commonly associated with small cell lung cancer than other lung cancers. It is for this reason that the most likely diagnosis is small cell and not squamous cell lung cancer.
A 48 year old woman with ovarian carcinoma presents to the A&E department with a 12 hour history of haemoptysis associated with dyspnoea & pleuritic pains. On examination she is apyrexial & has a right sided pleural rub. The chest x-ray shows a wedge shaped infarct peripherally on the right but is otherwise normal.
A. Pulmonary metastases B. Pulmonary embolism C. Pulmonary abscess D. Small cell carcinoma E. Streptococcal pneumonia F. Squamous cell carcinoma G. Microscopic polyarteritis H. Tuberculosis I. Goodpasture’s disease J. Myaesthenia gravis K. Mesothelioma
B. Pulmonary embolism - This CXR finding is Hampton’s hump seen in about a third of PE. Additional CXR signs include Westermark’s sign and Fleischner’s sign. CXR is however not diagnostic and may well be normal. Patients with a high clinical suspicion of PE should be anticoagulated while waiting a definitive diagnosis unless contraindicated. The underlying pathophysiology is based on Virchow’s triad. SOB and chest pain are common symptoms and there may also be haemoptysis. Strong risk factors include DVT, obesity, surgery in the past 2 months, prolonged bed rest, malignancy (which this patient has), previous VTE, pregnancy and the thrombophilias such as factor V Leiden. ECG may be normal, or may show tachycardia, new RAD, new RBBB or the classical S wave in I, Q wave with T inversion in III. Various clinical probability scores exist for PE and D-dimer can be used to exclude PE as a diagnosis. The study of choice is a CTPA with direct visualisation of the thrombus. If there is a contraindication to a CT scan such as contrast allergy or pregnancy, then a V/Q scan is indicated. If a V/Q scan is not possible, alternatives such as MRA can be requested. It is worth noting that in patients with cardiopulmonary disease, these tests may not be accurate. A TTE can also be used to detect RV strain seen with PE.
A 51 year old social worker presents to her GP with haemoptysis. On further questioning she admits to having a productive cough for 6 months & to losing 2 stones in weight over the same time. Chest x-ray shows patchy consolidation & scarring in both apices.
A. Pulmonary metastases B. Pulmonary embolism C. Pulmonary abscess D. Small cell carcinoma E. Streptococcal pneumonia F. Squamous cell carcinoma G. Microscopic polyarteritis H. Tuberculosis I. Goodpasture’s disease J. Myaesthenia gravis K. Mesothelioma
H. Tuberculosis - It is important to have a high level of suspicion when evaluating patients with risk factors who present with suggestive symptoms. Night sweats, fever, malaise, cough, haemoptysis and erythema nodosum are all suggestive. Other key risk factors for pulmonary TB include exposure to infection and returning from or being born in a high-risk region such as Asia, Africa and Latin America. If TB is suspected, the patient should be placed in isolation and a CXR obtained with 3 sputum samples cultured for AFB being the gold standard of diagnosis. Culture takes several weeks so sputum smears will be done before culture results are known. Interferon-gamma release assays (IGRAs) are now used by some hospitals to rapidly determine a patient’s TB status. All patients who have TB should be tested for HIV within 2 months of diagnosis. CXR is the first line test to order. Classically, in primary disease there are middle and lower zone infiltrates. Post-primary TB usually involves apical changes with or without cavitation. However, recent students have indicated that both presentations are seen in both primary and post-primary TB. HIV positive patients tend to have a more atypical CXR including effusion, lower zone involvement and a miliary pattern.
A 34 year old man presents to the A&E department with a short history of haemoptysis. He has had a cough for a fortnight & noticed his ankles beginning to swell 5 days ago. Initial blood tests show a creatinine of 400mol/l. An autoantibody screen is positive for p-ANCA & anti-glomerular basement membrane antibodies.
A. Pulmonary metastases B. Pulmonary embolism C. Pulmonary abscess D. Small cell carcinoma E. Streptococcal pneumonia F. Squamous cell carcinoma G. Microscopic polyarteritis H. Tuberculosis I. Goodpasture’s disease J. Myaesthenia gravis K. Mesothelioma
I. Goodpasture’s disease - Goodpasture’s is associated with anti-GBM antibodies, and of those who are positive, some will have a positive ANCA too, although this is more suggestive of diagnoses such as Churg-Strauss and microscopic polyarteritis. Definitive diagnosis is by renal biopsy showing crescentic GN and linear IgG staining on immunofluorescence. It is one of the few causes of pulmonary renal syndrome. Aggressive treatment is often needed and plasma exchange can also be performed to remove preformed antibodies.
Trachea deviated to the right. Hyper-resonant percussion on the left side with reduced breath sounds.
A. Pleurisy B. Emphysema C. Normal variant D. Pulmonary oedema E. Pleural effusion F. Lobar collapse G. Idiopathic pulmonary fibrosis H. Hyperventilation I. Pneumothorax J. Lobar pneumonia K. Chronic bronchitis
I. Pneumothorax - A tension pneumothorax shifts the trachea AWAY with hyper-resonance on affected side and reduced/absent breath sounds. A tracheal shift means this is a tension pneumothorax which needs immediate relief with a wide bore cannula inserted into the second intercostal space in the MCL.
Trachea deviated to left. Dull to percussion & reduced breath sounds at left base.
A. Pleurisy B. Emphysema C. Normal variant D. Pulmonary oedema E. Pleural effusion F. Lobar collapse G. Idiopathic pulmonary fibrosis H. Hyperventilation I. Pneumothorax J. Lobar pneumonia K. Chronic bronchitis
F. Lobar collapse - Collapse pulls the trachea TOWARDS the affected side. There is dullness and reduced/absent breath sounds due to a lack of air filled lung in this space. Do you know how to identify which lobe has collapsed on a CXR? The findings in this examination are consistent with LLL collapse. A ‘sail sign’ will classically be seen behind the cardiac shadow on CXR.
Reduced chest movements bilaterally. Using accessory muscles of respiration. Breath sounds generally quiet.
A. Pleurisy B. Emphysema C. Normal variant D. Pulmonary oedema E. Pleural effusion F. Lobar collapse G. Idiopathic pulmonary fibrosis H. Hyperventilation I. Pneumothorax J. Lobar pneumonia K. Chronic bronchitis
B Emphysema - This patient is in obvious respiratory distress and is using accessory muscles of respiration. Chest movements are reduced too. There is hyperexpansion of the lung fields due to emphysema, which has caused the respiratory distress, combined with diminished breath sounds.
Bilateral fine basal crepitations. There are no signs of CCF.
A. Pleurisy B. Emphysema C. Normal variant D. Pulmonary oedema E. Pleural effusion F. Lobar collapse G. Idiopathic pulmonary fibrosis H. Hyperventilation I. Pneumothorax J. Lobar pneumonia K. Chronic bronchitis
G. Idiopathic pulmonary fibrosis - Bibasal inspiratory crackles without signs of CCF are seen in idiopathic pulmonary fibrosis. These are described as ‘Velcro-like’ in quality. IPF is also associated with clubbing. Idiopathic pulmonary fibrosis (previously known as Cryptogenic fibrosing alveolitis) progresses over several years and is characterised by pulmonary scar tissue formation and dyspnoea. Patients complain of a non-productive cough and typically reproducible and predictable SOB on exertion
Trachea central. Reduced chest movement on right. Dull to percussion on right. Bronchial breathing at right base.
A. Pleurisy B. Emphysema C. Normal variant D. Pulmonary oedema E. Pleural effusion F. Lobar collapse G. Idiopathic pulmonary fibrosis H. Hyperventilation I. Pneumothorax J. Lobar pneumonia K. Chronic bronchitis
J. Lobar pneumonia - These are the classic signs of pneumonia. Also expect to find increased vocal resonance and tactile vocal fremitus over areas of consolidation. In reality, it can be confusing if the pneumonia causes a lobar collapse as you can also find signs of collapse on examination. Always consider the history as well as examination and investigation findings. On a CXR with pneumonia, you can expect to see air space shadowing with air bronchograms. Always remember to auscultate at the right axilla when doing a respiratory examination or a RML pneumonia may be missed.
A 50 year old man became suddenly breathless whilst eating. He has marked stridor & is choking & drooling.
A. Heimlich manoeuvre B. Forced alkaline diuresis C. Intravenous furosemide D. Rapid infusion of saline E. Nebulised salbutamol F. Intravenous aminophylline G. Re-breathing into paper bag H. Pleural aspiration I. Chest drain J. Intravenous adrenaline K. Heparin L. Intravenous insulin
A. Heimlich manoeuvre - This patient has choked on some food. The patient should be encouraged to cough if they are conscious. Otherwise, external manoevres can be performed such as abdominal thrusts (Heimlich) or back blows. These actions increase intrathoracic pressure and help to dislodge the foreign body. If it still isn’t removed, a flexible bronchoscopy may be necessary. Most cases occur in very young children.
A 60 year old male presents with acute breathlessness & a cough productive of frothy, pink sputum. He cannot lie flat. On examination, he has crackles to both midzones & a few scattered wheezes.
A. Heimlich manoeuvre B. Forced alkaline diuresis C. Intravenous furosemide D. Rapid infusion of saline E. Nebulised salbutamol F. Intravenous aminophylline G. Re-breathing into paper bag H. Pleural aspiration I. Chest drain J. Intravenous adrenaline K. Heparin L. Intravenous insulin
C. Intravenous furosemide - This patient has pulmonary oedema. CXR may show pulmonary vascular redistribution to the upper zones, Kerley B lines, an increased CTR (cardiomegaly) and pleural effusion.
A 20 year old woman is too breathless to speak. Her pulse is 120/min, respiratory rate 30 per min & peak expiratory flow is 100l/min. Examination reveals a very quiet chest & chest x-ray is normal.
A. Heimlich manoeuvre B. Forced alkaline diuresis C. Intravenous furosemide D. Rapid infusion of saline E. Nebulised salbutamol F. Intravenous aminophylline G. Re-breathing into paper bag H. Pleural aspiration I. Chest drain J. Intravenous adrenaline K. Heparin L. Intravenous insulin
E. Nebulised salbutamol - This patient is having an asthma attack. This patient is too breathless to speak and has a quiet chest so this is severe and ICU admission is indicated. Initial treatment is with repeated administration of an inhaled SABA with early systemic corticosteroids and supplemental oxygen, monitoring the patient’s status regularly.
A 25 year old woman has just returned from a holiday in Kenya. She suddenly became breathless & is cyanosed. Her pulse is 120/min, BP 110/70, peak expiratory flow 400l/min. Chest x-ray is normal.
A. Heimlich manoeuvre B. Forced alkaline diuresis C. Intravenous furosemide D. Rapid infusion of saline E. Nebulised salbutamol F. Intravenous aminophylline G. Re-breathing into paper bag H. Pleural aspiration I. Chest drain J. Intravenous adrenaline K. Heparin L. Intravenous insulin
K. Heparin - Patients with a high clinical suspicion of PE should be anticoagulated while waiting a definitive diagnosis unless contraindicated. As UFH is used in this case, a weight based dosing normogram will need to be used to establish a therapeutic APTT within the first 24 hours. Alternatives include LMWH and fondaparinux.
A 50 year old with pulmonary fibrosis develops sudden left-sided pleuritic pain & dyspnoea. He has reduced air entry in the left side of the chest & percussion is hyper-resonant. Oxygen saturation is 80%.
A. Heimlich manoeuvre B. Forced alkaline diuresis C. Intravenous furosemide D. Rapid infusion of saline E. Nebulised salbutamol F. Intravenous aminophylline G. Re-breathing into paper bag H. Pleural aspiration I. Chest drain J. Intravenous adrenaline K. Heparin L. Intravenous insulin
I. Chest drain - This patient has developed a pneumothorax. This is a secondary spontaneous pneumothorax, and if large enough for a chest tube or the patient is clinically unstable, chest drain insertion is indicated. Simple aspiration success rate is reduced in secondary spontaneous pneumothoraces. Primary pneumothoraces occur in young people without known lung conditions. This patient has pulmonary fibrosis. Those who suffer recurrent pneumothoraces may have to undergo pleurodesis using a chemical such as doxycycline to stick the perietal and visceral pleural together by an inflammatory reaction.
A 36 year old popstar presents with fever, a cough & an itchy vesicular rash. Chest x-ray shows mottling through both lung fields.
A. Mycoplasma pneumonia B. Streptococcus pneumonia C. Varicella zoster D. Adenovirus E. Pneumocystis jirovecii F. Influenzae A G. Haemophilus influenza H. Group A streptococci I. Staphylococcus aureus J. Corynebacterium diphtheriae K. Legionella pneumophila L. Escherichia coli M. Aspergillus fumigatus N. Clamydia pneumoniae
C. Varicella zoster - The pruritic vesicular rash (the classic description of a ‘dewdrop on a rose petal’) makes you think of VZV. The rash typically occurs on the patient’s torso and face and pneumonia is a complication occuring more commonly in those with immunosuppression. The lesions are often crusted over by 7-10 days. The diagnosis is based on clinical findings.
A 60 year old woman presented to the casualty with chest pain on coughing, fever, shortness of breath for the last 2 days. Her chest x-ray showed right lower lobe consolidation & pleural effusion.
A. Mycoplasma pneumonia B. Streptococcus pneumonia C. Varicella zoster D. Adenovirus E. Pneumocystis jirovecii F. Influenzae A G. Haemophilus influenza H. Group A streptococci I. Staphylococcus aureus J. Corynebacterium diphtheriae K. Legionella pneumophila L. Escherichia coli M. Aspergillus fumigatus N. Clamydia pneumoniae
B. Streptococcus pneumoniae - This is classic CAP. Classic lobar pneumonia with no signs and symptoms to suggest an atypical organism is most likely to due to pneumococcus. Symptoms include chills, fever, cough, SOB and pleuritic chest pain. A CXR is the most specific and sensitive test available and antibiotics are indicated.
30 year old intravenous drug abuser presented to the GP with shortness of breath. The GP sent him for a blood count, HIV testing & a chest x-ray. He was found to be HIV positive. Chest x-ray showed bilateral fluffy opacities.
A. Mycoplasma pneumonia B. Streptococcus pneumonia C. Varicella zoster D. Adenovirus E. Pneumocystis jirovecii F. Influenzae A G. Haemophilus influenza H. Group A streptococci I. Staphylococcus aureus J. Corynebacterium diphtheriae K. Legionella pneumophila L. Escherichia coli M. Aspergillus fumigatus N. Clamydia pneumoniae
E. Pneumocystis jirovecii - PCP is caused by Pneumocystis jirovecii, previously called Pneumocystis carinii. It is a fungal organism and an AIDS defining illness. Signs and symptoms occur in a patient who is immunosuppressed, especially HIV with a CD4 count
A plumber renovating old properties presented to casualty with fever & loss of consciousness. On examination he had bilateral consolidation. Plasma sodium was low. The doctor sent for urinary antigen & serology. On the results he was treated with azithromycin & ciprofloxacin & improved.
A. Mycoplasma pneumonia B. Streptococcus pneumonia C. Varicella zoster D. Adenovirus E. Pneumocystis jirovecii F. Influenzae A G. Haemophilus influenza H. Group A streptococci I. Staphylococcus aureus J. Corynebacterium diphtheriae K. Legionella pneumophila L. Escherichia coli M. Aspergillus fumigatus N. Clamydia pneumoniae
K. Legionella pneumphila - Legionella is a gram negative rod. Legionella infecting the lungs is legionnaires’ disease or Legionella pneumonia whereas non-lung infection is known as Pontiac fever. This bacteria is found in aqueous environments such as lakes and almost all cases are from contaminated water systems, which relates to the risk factors of getting Legionella (recent water exposure like a hot tub). Smoking is also a risk factor. It can cause confusion as well as hyponatraemia, abdominal pain, diarrhoea and bradycardia. Legionella does not grow on routine culture media and diagnosis relies on urine antigen detection, serology or culture on special media. Treatment is with fluoroquinolones or macrolides. Azithromycin is considered a safer option than erythromycin. For severe cases, a combination of both is given although this is potentially toxic and can cause long QT and torsades de pointes.
A holiday worker had a severe chest infection abroad & was diagnosed to have influenza A infection. He was improving but suddenly deteriorated with the last 24 hours becoming breathless, febrile & septic. X-ray chest showed circular opacities some with a fluid level. Gram stain of sputum showed Gram positive cocci in clusters.
A. Mycoplasma pneumonia B. Streptococcus pneumonia C. Varicella zoster D. Adenovirus E. Pneumocystis jirovecii F. Influenzae A G. Haemophilus influenza H. Group A streptococci I. Staphylococcus aureus J. Corynebacterium diphtheriae K. Legionella pneumophila L. Escherichia coli M. Aspergillus fumigatus N. Clamydia pneumoniae
I. Staphylococcus aureus - Think Staphylococcus aureus for post-influenza pneumonia. It causes a cavitating pneumonia which explains the CXR findings (some abscesses are also seen) and Gram stain of culture yields grape like clusters of Gram positive cocci which is consistent with staphylococcus. Treatment of staphyloccocal infection is with flucoxacillin or vancomycin if MRSA.
A 30 year old company executive became unwell whilst on a business trip. He developed a high fever, muscle pains, nausea & vomiting, abdominal pain. He admitted to the hotel doctor that his cough has worsened over the past 7 days & he had coughed up blood on a couple of occasions. Chest x-ray showed consolidation in both lungs.
A. Atypical pneumonia B. Bronchial carcinoma C. Pleural effusion D. Sarcoidosis E. Fibrosing alveolitis F. Pneumothorax G. Lung abscess H. Bronchiectasis I. Bronchial asthma J. COPD K. Cystic fibrosis
A. Atypical pneumonia - Given this is the only pneumonia option on the list, this is an easy question, although the presentation is in line with an atypical organism. The most common atypical pneumonias are Mycoplasma, Legionella and Chlamydophila (Chlamydia). All can be treated with macrolides and are to some extent sensitive to fluoroquinolones and tetracyclines too, although these are contraindicated in pregnancy. Check local prescribing policies first.
A 40 year old alcoholic man who was treated for a chest infection one week previously, developed a productive cough, & a fever. He felt unwell. He lost weight during this period of illness & also coughed up blood several times. A chest x-ray showed a fluid level in the right lung.
A. Atypical pneumonia B. Bronchial carcinoma C. Pleural effusion D. Sarcoidosis E. Fibrosing alveolitis F. Pneumothorax G. Lung abscess H. Bronchiectasis I. Bronchial asthma J. COPD K. Cystic fibrosis
G. Lung abscess - A lung abscess is diagnosed on CXR with a cavitation with an air-fluid level in it. Preceding pneumonia which this patient gives a history of is a risk factor. Fever and a productive cough are common symptoms and treatment involves antibacterials and drainage/resection if unresponsive. Lung abscesses are commonly caused by aspiration of gastric contents.
A 20 year old man with recurrent episodes of chest infection & diarrhoea, which is difficult to flush away in the toilet. He developed a persistent cough with the production of sputum & blood. On examination his fingers are clubbed & in his chest there are low pitched inspiratory & expiratory crackles, plus some wheeze. He recalls being small for his age despite having a healthy appetite.
A. Atypical pneumonia B. Bronchial carcinoma C. Pleural effusion D. Sarcoidosis E. Fibrosing alveolitis F. Pneumothorax G. Lung abscess H. Bronchiectasis I. Bronchial asthma J. COPD K. Cystic fibrosis
K. Cystic fibrosis - CF is autosomal recessive and the mean age of death is around 40. There is currently no cure for this condition. The reccurent chest infections and greasy stools (fat malabsorption due to pancreatic insufficiency) should make you think of CF. A persistent cough which is productive should also raise suspicions. Examination findings here which raise your suspicion include clubbing and crackles on auscultation. Additionally, you may find nasal polyps and hepatomegaly and/or splenomegaly and a congenital absence of the vas deferens in males. There is also some failure to thrive with the patient being small for his age. The most conclusive diagnostic test is the sweat test which is positive if sweat chloride is >60mmol/L. Serum IRT from a heel prick blood spot allows screening of newborns. CF is a genetic condition with abnormal salt and water transport due to mutations in the CFTR (an apical anion channel). Heterozygotes generally do not demonstrate disease.
Mr Jones is a 60 year old man who smoked heavily over the past 40 years (up to 40 cigarettes per day). He gives a 4 week history of a cough with the production of sputum & blood, breathlessness chest pain & weight loss. On examination his fingers were clubbed & he had a ptosis in the left eye.
A. Atypical pneumonia B. Bronchial carcinoma C. Pleural effusion D. Sarcoidosis E. Fibrosing alveolitis F. Pneumothorax G. Lung abscess H. Bronchiectasis I. Bronchial asthma J. COPD K. Cystic fibrosis
B. Bronchial carcinoma - This patient is a heavy smoker with a history of respiratory signs and weight loss. The presentation is alluding to a pancoast tumour (most are non-small cell) in the left apex causing ipsilateral Horner’s syndrome (miosis, anhidrosis, ptosis and enophthalmos). A pancoast tumour can also affect the brachial plexus leading to wasting of the intrinsic muscles of the hand as well as a hoarse bovine cough due to compression of the recurrent laryngeal nerve. Initial investigation of bronchial carcinoma is with a CXR. Diagnosis relies on pathological confirmation from a tissue sample, often obtained from bronchoscopy. First line treatment aims at surgical resection if possible. Small cell lung cancer is treated with chemotherapy and is associated with SIADH and ectopic ACTH. Non-small cell lung cancer is more often associated with clubbing. Squamous cell carcinoma is associated with PTHrp release and is treated with radiotherapy. Adenocarcinomas are usually located peripherally in the lung and are more common in non-smokers although most cases are still associated with smoking. The paraneoplastic syndromes may include Lambert-Eaton myasthenic syndrome.
John is a 35 year old tennis player who complains of breathlessness after games. His sleep has been disturbed recently by coughing during the night. On examination he was a tall, thin but athletic looking young man. There were high-pitched expiratory wheeze in both lungs.
A. Atypical pneumonia B. Bronchial carcinoma C. Pleural effusion D. Sarcoidosis E. Fibrosing alveolitis F. Pneumothorax G. Lung abscess H. Bronchiectasis I. Bronchial asthma J. COPD K. Cystic fibrosis
I. Bronchial asthma - This patient has developed a primary pneumothorax. Primary pneumothoraces occur in young people without known lung conditions. The main investigation is a CXR and pneumothoraces are classified by the BTS as large (>2cm visible rim between the lung margin and the chest wall) or small (
Mark is a 20 year old student who developed acute chest pain with breathlessness after working out at the gym. On examination he was breathless at rest. Expansion of the chest was restricted on one side with hyper-resonant percussion note.
A. Atypical pneumonia B. Bronchial carcinoma C. Pleural effusion D. Sarcoidosis E. Fibrosing alveolitis F. Pneumothorax G. Lung abscess H. Bronchiectasis I. Bronchial asthma J. COPD K. Cystic fibrosis
F. Pneumothorax - A tension pneumothorax shifts the trachea AWAY with hyper-resonance on affected side and reduced/absent breath sounds. A tracheal shift means this is a tension pneumothorax which needs immediate relief with a wide bore cannula inserted into the second intercostal space in the MCL.
A 20 year old man with cystic fibrosis presents to the chest clinic with haemoptysis. He has felt unwell for a fortnight with increased sputum production, fever & rigors. Gram stain of the sputum shows Gram-positive cocci in clusters.
A. Pulmonary metastases B. Pulmonary embolism C. Pulmonary abscess D. Small cell carcinoma E. Streptococcal pneumonia F. Squamous cell carcinoma G. Microscopic polyarteritis H. Tuberculosis I. Goodpasture’s disease J. Myaesthenia gravis K. Mesothelioma
C. Pulmonary abscess - A lung abscess is diagnosed on CXR with a cavitation with an air-fluid level in it. Preceding pneumonia which a patient with CF is at risk of is a risk factor. Of gram positive cocci, staphylococcus occurs in grape-like clusters (this patient) whereas streptococcus occurs in chains. It is worth learning your gram stains for the main organisms. It is worth noting that Staphylococcus aureus is coagulase positive (also Yersinia pestis which causes plague) and Streptoccus pneumoniae is optochin sensitive. Fever and a productive cough are common symptoms and treatment involves antibacterials and drainage/resection if unresponsive. Lung abscesses are commonly caused by aspiration of gastric contents.
A 45 year old doctor from Ethiopia with a 6 week history of fever, drenching night sweats and a cough. He is a heavy smoker. On examination he is thin and looks unwell. He has nicotine stained fingers. Dull to percusion at the right upper zone with reduced breath sounds.
A. Pneumothorax B. Pneumonia C. COPD D. Carcinoma of Bronchus E. Chest injury with rib fractures F. Lung metastases G. Rheumatoid arthritis H. Pleural mesothelioma I. Aspiration pneumonia J. Pulmonary oedema K. Sarcoidosis L. Pulmonary embolus M. Acute asthma N. Pulmonary tuberculosis
N. Pulmonary TB - This is pulmonary TB. The patient is from an endemic area and has presented with fever, drenching night sweats and a cough. There is also anorexia and examination findings are consistent with post-primary TB with apical consolidation. CXR is the first line test to order. Classically, in primary disease there are middle and lower zone infiltrates. Post-primary TB usually involves apical changes with or without cavitation. However, recent students have indicated that both presentations are seen in both primary and post-primary TB. HIV positive patients tend to have a more atypical CXR including effusion, lower zone involvement and a miliary pattern.
25 year old female with acute onset of chills, fever, cough with brown phlegm for three days. On examination she appears toxic, temperture 40degrees C, reduced breath sounds, bronchial breathing and stony dullness left lung base.
A. Pneumothorax B. Pneumonia C. COPD D. Carcinoma of Bronchus E. Chest injury with rib fractures F. Lung metastases G. Rheumatoid arthritis H. Pleural mesothelioma I. Aspiration pneumonia J. Pulmonary oedema K. Sarcoidosis L. Pulmonary embolus M. Acute asthma N. Pulmonary tuberculosis
B. Pneumonia - The rusty coloured phlegm is hinting at a pneumococcal pneumonia.The patient has presented with common symptoms of fever, chills and a cough. There may also be SOB, rigors and pleuritic chest pain. The most specific and sensitive test is a CXR (PA and lateral) and initial treatment of a CAP is empirical with antibiotics. Often diagnosis is made solely on history and examination findings. Bronchial breathing, reduced breath sounds and the presence of a left sided parapneumonic effusion all indicate a pneumonia. Management is guided by the patient’s CURB-65 score.
70 year old male ex-builder with progressive pain in his right chest, and with cough and SOB for a few months. CXR shows pleural thickening and right pleural effusion.
A. Pneumothorax B. Pneumonia C. COPD D. Carcinoma of Bronchus E. Chest injury with rib fractures F. Lung metastases G. Rheumatoid arthritis H. Pleural mesothelioma I. Aspiration pneumonia J. Pulmonary oedema K. Sarcoidosis L. Pulmonary embolus M. Acute asthma N. Pulmonary tuberculosis
H. Pleural mesothelioma - An ex-builder is likely to have a history of asbestos exposure which is the principal risk factor for developing a malignant pleural mesothelioma. CXR shows the presence of a unilateral effusion and irregular pleural thickening. Most patients present with SOB and chest pain. There is a 20-40 year latency period between exposure to asbestos and development of malignancy. Hence, the typical patient presents between 60-80. A more invasive investigation is needed for a definitive diagnosis.
A 70 year old female, heavy smoker, for several years who presents with weight loss, reduced appetite and haemoptysis for 1 month. On examination she is thin, afebrile and is clubbed. She has bronchial breathing right upper zone. Reduced breath sounds and dullness on right base. CXR shows right lung collapse with effusion
A. Pneumothorax B. Pneumonia C. COPD D. Carcinoma of Bronchus E. Chest injury with rib fractures F. Lung metastases G. Rheumatoid arthritis H. Pleural mesothelioma I. Aspiration pneumonia J. Pulmonary oedema K. Sarcoidosis L. Pulmonary embolus M. Acute asthma N. Pulmonary tuberculosis
D. Carcinoma of bronchus - The history of smoking and weight loss combined with examination findings point to a bronchial carcinoma. First line treatment aims at surgical resection if possible. Small cell lung cancer is treated with chemotherapy and is associated with SIADH and ectopic ACTH. Non-small cell lung cancer is more often associated with clubbing. Squamous cell carcinoma is associated with PTHrp release and is treated with radiotherapy. Adenocarcinomas are usually located peripherally in the lung and are more common in non-smokers although most cases are still associated with smoking. The paraneoplastic syndromes may include Lambert-Eaton myasthenic syndrome.
A 50 year old Asian diabetic woman is admitted with increasing shortness of breath and ankle swelling. ECG shows inverted T waves in levels I, AVL and V4-6. Upper lobe blood diversion and bilateral pleural effusions are found on chest X-ray.
A. Pneumothorax B. Pneumonia C. COPD D. Carcinoma of Bronchus E. Chest injury with rib fractures F. Lung metastases G. Rheumatoid arthritis H. Pleural mesothelioma I. Aspiration pneumonia J. Pulmonary oedema K. Sarcoidosis L. Pulmonary embolus M. Acute asthma N. Pulmonary tuberculosis
J. Pulmonary oedema - CXR findings here are consistent with pulmonary oedema. Pulmonary vascular redistribution to the upper zones, Kerley B lines, an increased CTR (cardiomegaly) and pleural effusion may be seen. The patient is also in CCF with evidence of LV dysfunction (SOB) and RV dysfunction (ankle swelling).
A 10 year old boy presents to A&E one winter following a school P.E. lesson with a difficulty in breathing and an audible wheeze.
A. TB B. Emphysema C. Bronchitis D. Pleural effusion E. Pneumonia F. Lung Tumour G. Pulmonary embolism H. Asthma I. Pneumothorax J. Pulmonary fibrosis K. Sarcoidosis
H. Asthma - This is likely to be asthma, exacerbated by the cold air and exercise. Treatment is in this case based on paediatric guidelines which differs from that of adult asthma. Diagnosis is supported by PEFR variation of at least 20% over 3 days in a week over several weeks or an increase of at least 20% to treatment.
Stepwise treatment for adult asthma (BTS guidelines) is outlined here as this is most commonly examined. Step 1: SABA PRN, Step 2: Plus low-dose inhaled corticosteroids (ICS) , Step 3: Plus LABA, Step 4: Increase dose of ICS or add LTRA, SR theophylline or beta agonist tablet, Step 5: Daily steroid tablet and maintain ICS with specialist care.
A 24 year old accountant presented to his GP with a 2 week history of tiredness and a persistant cough and complained of “not being able to complete his normal gym routine”. On examination he was pyrexic, had decreased lung expansion and increased vocal resonance and auscultation revealed bronchial breathing.
A. TB B. Emphysema C. Bronchitis D. Pleural effusion E. Pneumonia F. Lung Tumour G. Pulmonary embolism H. Asthma I. Pneumothorax J. Pulmonary fibrosis K. Sarcoidosis
E. Pneumonia - This patient has pneumonia. Symptoms include chills, fever, cough, SOB and pleuritic chest pain. Examination findings are consistent with his diagnosis. A CXR is the most specific and sensitive test available and antibiotics are indicated.
A 73 year old lady, presented to the respiratory team after a 2 week stay in hospital following a stroke. She was breathless at rest with a central chest pain and was coughing up blood.
A. TB B. Emphysema C. Bronchitis D. Pleural effusion E. Pneumonia F. Lung Tumour G. Pulmonary embolism H. Asthma I. Pneumothorax J. Pulmonary fibrosis K. Sarcoidosis
G. Pulmonary embolism - Patients with a high clinical suspicion of PE should be anticoagulated while waiting a definitive diagnosis unless contraindicated. The underlying pathophysiology is based on Virchow’s triad. SOB and chest pain are common symptoms and there may also be haemoptysis. Strong risk factors include DVT, obesity, surgery in the past 2 months, prolonged bed rest, malignancy, previous VTE, pregnancy and the thrombophilias such as factor V Leiden. ECG may be normal, or may show tachycardia, new RAD, new RBBB or the classical S wave in I, Q wave with T inversion in III. Various clinical probability scores exist for PE and D-dimer can be used to exclude PE as a diagnosis. The study of choice is a CTPA with direct visualisation of the thrombus. If there is a contraindication to a CT scan such as contrast allergy or pregnancy, then a V/Q scan is indicated. If a V/Q scan is not possible, alternatives such as MRA can be requested. It is worth noting that in patients with cardiopulmonary disease, these tests may not be accurate. A TTE can also be used to detect RV strain seen with PE.
A 55 year old male smoker presents to her GP with a 6 month history of a persistant dry cough and unexplained weight loss. On examination she was jaundiced with an enlarged liver.
A. TB B. Emphysema C. Bronchitis D. Pleural effusion E. Pneumonia F. Lung Tumour G. Pulmonary embolism H. Asthma I. Pneumothorax J. Pulmonary fibrosis K. Sarcoidosis
F. Lung tumour - This is a smoker with respiratory complaints and unexplained weight loss pointing to malignancy. The hepatomegaly is likely to be nodular due to hepatic secondaries from a primary lung cancer and this hepatic dysfunction has caused this patient’s jaundice. First line treatment aims at surgical resection if possible. Small cell lung cancer is treated with chemotherapy and is associated with SIADH and ectopic ACTH. Non-small cell lung cancer is more often associated with clubbing. Squamous cell carcinoma is associated with PTHrp release and is treated with radiotherapy. Adenocarcinomas are usually located peripherally in the lung and are more common in non-smokers although most cases are still associated with smoking.
A Nigerian 26 year old female presents to her doctor with a dry cough and a painful and strange red left shin. A subsequent chest X-ray reveals bilateral hilar lymphadenopathy.
A. TB B. Emphysema C. Bronchitis D. Pleural effusion E. Pneumonia F. Lung Tumour G. Pulmonary embolism H. Asthma I. Pneumothorax J. Pulmonary fibrosis K. Sarcoidosis
K. Sarcoidosis - Sarcoidosis is a chronic multisystem disease with an unknown aetiology. Lung involvement is very common. The strange red left shin is erythema nodosum and are tender erythematous nodules. Lupus pernio is another typical skin manifestation of sarcoidosis presenting with indurated plaques with discoloration on the face. CXR will typically show bilateral hilar lymphadenopathy and CXR findings are used in the staging of disease. Additionally, serum calcium and ACE levels may be raised. A transbronchial biopsy is essential for diagnosis in most cases and shows the presence of non-caseating granulomas. Black people have a higher lifetime risk of sarcoidosis, as do those of Scandinavian origin. The mainstay of treatment for severe disease involves systemic corticosteroids.
A 48 year old male pharmeceutical worker presents with difficulty sleeping. On further questioning you find out that it is due to coughing all night. He is also suffering chest tightness, breathlessness and you can detect a wheeze when he talks to you. Although his wife notes that he is not as bad on the weekend.
A. Cystic fibrosis B. Pancreatitis C. Tuberculosis D. Emphysema E. Asthma F. Pneumonia G. Chronic bronchitis H. Bronchiectasis I. Lung cancer J. HIV K. Lung abscess
E. Asthma - This patient has signs and symptoms of asthma. Diagnosis is supported by PEFR variation of at least 20% over 3 days in a week over several weeks or an increase of at least 20% to treatment. Stepwise treatment is based on BTS guidelines:
Step 1: SABA PRN, Step 2: Plus low-dose inhaled corticosteroids (ICS) , Step 3: Plus LABA, Step 4: Increase dose of ICS or add LTRA, SR theophylline or beta agonist tablet, Step 5: Daily steroid tablet and maintain ICS with specialist care.
A 26 year old Italian nightclub DJ presents with abdominal pain. On enquiry he has been unwell with a productive cough, fever and breathlessness. On examination his heart rate is 110bpm and his blood pressure is 110/75. His abdomen is soft, non tender.
A. Cystic fibrosis B. Pancreatitis C. Tuberculosis D. Emphysema E. Asthma F. Pneumonia G. Chronic bronchitis H. Bronchiectasis I. Lung cancer J. HIV K. Lung abscess
F. Pneumonia - This is basal pneumonia which can present with upper abdominal pain. The symptoms this patient gives are consistent with pneumonia. Treatment is guided by the CURB-65 score. A CXR is the most specific and sensitive test available and antibiotics are indicated. CXR may show airspace shadowing with air bronchograms. Make sure you can spot consolidation on a CXR.
A 65 year old American woman suffering from chronic bronchitis presents with weight loss, worsening of cough and red blood in sputum. She recently gave up smoking after being a heavy smoker since she was a teenager.
A. Cystic fibrosis B. Pancreatitis C. Tuberculosis D. Emphysema E. Asthma F. Pneumonia G. Chronic bronchitis H. Bronchiectasis I. Lung cancer J. HIV K. Lung abscess
I. Lung cancer - Weight loss combined with worsening respirtory symptoms and the past history of smoking point to malignancy. First line treatment aims at surgical resection if possible. Small cell lung cancer is treated with chemotherapy and is associated with SIADH and ectopic ACTH. Non-small cell lung cancer is more often associated with clubbing. Squamous cell carcinoma is associated with PTHrp release and is treated with radiotherapy. Adenocarcinomas are usually located peripherally in the lung and are more common in non-smokers although most cases are still associated with smoking.
A 3yr old Caucasian child is brought into clinic by his mother who is concerned about his cough. Looking through his notes you find out he has a history of chest infection. However the only medication he is currently taking is a steroid cream for his eczema. You discover that he opens his bowels twice a day and has offensive loose stools. On examination his height is on the 91st centile and his weight is below the 25th centile.
A. Cystic fibrosis B. Pancreatitis C. Tuberculosis D. Emphysema E. Asthma F. Pneumonia G. Chronic bronchitis H. Bronchiectasis I. Lung cancer J. HIV K. Lung abscess
A. Cystic Fibrosis - This infant has presented with cystic fibrosis which is an autosomal recessive condition characterised by a mutation in the CFTR gene on chromosome 7 (delta-F508). In this case there is failure to thrive, recurrent chest infections and evidence of pancreatic insufficiency (steatorrhoea). The most conclusive diagnostic test is the sweat test which is pisitive if sweat chloride is >60mmol/L. Serum IRT from a heel prick blood spot allows screening of newborns. CF is a genetic condition with abnormal salt and water transport due to mutations in the CFTR (an apical anion channel). Heterozygotes generally do not demonstrate disease.
A 37 year old Malaysian female presents with a productive cough and purulent sputum and night sweats. What concerns her most is the bright red blood which is often also in the sputum.
A. Cystic fibrosis B. Pancreatitis C. Tuberculosis D. Emphysema E. Asthma F. Pneumonia G. Chronic bronchitis H. Bronchiectasis I. Lung cancer J. HIV K. Lung abscess
C. Tuberculosis - It is important to have a high level of suspicion when evaluating patients with risk factors who present with suggestive symptoms. Night sweats, fever, malaise, cough, haemoptysis and erythema nodosum are all suggestive. Other key risk factors for pulmonary TB include exposure to infection and returning from or being born in a high-risk region such as Asia, Africa and Latin America. If TB is suspected, the patient should be placed in isolation and a CXR obtained with 3 sputum samples cultured for AFB being the gold standard of diagnosis. Culture takes several weeks so sputum smears will be done before culture results are known. Interferon-gamma release assays (IGRAs) are now used by some hospitals to rapidly determine a patient’s TB status. All patients who have TB should be tested for HIV within 2 months of diagnosis.
A 35 year old man presents to A&E with a short history of haemoptysis & breathlessness. His pulse is 125bpm & he has recently travelled to Australia. Chest examination is unremarkable. He is allergic to contrast agents.
A. Sputum cultures B. Bronchoscopy C. History only D. D-dimer E. CTPA F. Chest x-ray G. MRA H. Lung function tests I. ABG J. Clotting screen K. V/Q scan L. CT head
K. V/Q scan - This patient has a PE. The study of choice is a CTPA with direct visualisation of the thrombus. If there is a contraindication to a CT scan such as contrast allergy (in this case) or pregnancy, then a V/Q scan is indicated. If a V/Q scan is not possible, alternatives such as MRA can be requested. It is worth noting that in patients with cardiopulmonary disease, these tests may not be accurate. A TTE can also be used to detect RV strain seen with PE. Patients with a high clinical suspicion of PE should be anticoagulated while waiting a definitive diagnosis unless contraindicated. The underlying pathophysiology is based on Virchow’s triad. SOB and chest pain are common symptoms and there may also be haemoptysis. Strong risk factors include DVT, obesity, surgery in the past 2 months, prolonged bed rest, malignancy (which this patient has), previous VTE, pregnancy and the thrombophilias such as factor V Leiden.
A 30 year old Indian lady who has recently returned from a 4 month trip to Delhi complains of a cough & haemoptysis & night sweats over the last month.
A. Sputum cultures B. Bronchoscopy C. History only D. D-dimer E. CTPA F. Chest x-ray G. MRA H. Lung function tests I. ABG J. Clotting screen K. V/Q scan L. CT head
A. Sputum cultures - This sounds like TB. The patient should be placed in isolation and a CXR obtained with 3 sputum samples cultured for AFB being the gold standard of diagnosis. Treatment is with anti-TB medication: generally 2 months of rifampicin, isoniazid (given with pyridoxine to prevent associated neuropathy), pyrazinamide and ethambutol followed by 4 months of rifampicin and isoniazid alone. Treatment of MDR TB is more complicated and involves longer treatment with more drugs. Note that CNS TB requires 12 months of treatment.
A 19 year old man who has been intubated due to a recent RTA is recovering well & is extubated. He complains of coughing up a small amount of blood streaked phlegm.
A. Sputum cultures B. Bronchoscopy C. History only D. D-dimer E. CTPA F. Chest x-ray G. MRA H. Lung function tests I. ABG J. Clotting screen K. V/Q scan L. CT head
C. History only - This is a result of intubation which has caused some iatrogenic trauma to this patient’s upper airway.
A 55 year old lawyer who has had a chronic cough for 3 months complains of 1 episode of haemoptysis. She is a heavy smoker (about 40/day) & has experienced some recent weight loss.
A. Sputum cultures B. Bronchoscopy C. History only D. D-dimer E. CTPA F. Chest x-ray G. MRA H. Lung function tests I. ABG J. Clotting screen K. V/Q scan L. CT head
B. Bronchoscopy - This sounds like bronchial carcinoma from the history of smoking, respiratory complaints and weight loss. Initial investigation of bronchial carcinoma is with a CXR but definitive diagnosis relies on pathological confirmation from a tissue sample, often obtained from bronchoscopy. During bronchoscopy, endobronchial masses can be biopsied and washings/alveolar lavage can also be performed for cytological analysis. Trans-thoracic needle aspiration may be needed for peripheral lesions that cannot be reached by bronchoscopy. First line treatment aims at surgical resection if possible.
A 40 year old man complains of breathlessness & coughing up blood for 3 days. He mentions that prior to this he had fallen onto the corner of a table. Breath sounds seem slightly reduced on the right had side.
A. Sputum cultures B. Bronchoscopy C. History only D. D-dimer E. CTPA F. Chest x-ray G. MRA H. Lung function tests I. ABG J. Clotting screen K. V/Q scan L. CT head
F. Chest x-ray - This is chest trauma which has obviously caused some damage. This will need to be visualised by performing a chest XR. A CT scan may also be necessary. There is a chance of atelectasis given this patient’s history and findings on examination.
A young 23 year old allergic atopic asthmatic woman becomes acutely breathless. She has just taken an Aspirin for headache. Her neck is swollen, eyes puffy, & her breathing is noisy, with marked wheeze throughout the lung fields.
A. Oral prednisolone B. 24% continuous oxygen C. Long acting beta agonist D. Beta agonist via metered dose inhaler E. Antibiotic reserve F. Influenza immunisation G. Intravenous aminophylline H. 40% continuous oxygen I. Beta agonist via nebuliser J. Intravenous hydrocortisone K. Amoxycillin intravenously L. Intramuscular adrenaline
L. Intramuscular adrenaline - There is a sudden onset of respiratory (and cardiovascular) complaints with the recently given aspirin tablet. This patient is having an anaphylactic reaction and the airway needs to be promptly secured and prompt treatment started with adrenaline. IM adrenaline must not be delayed and the anterolateral thigh is the preferred location, with repeated doses as necessary every 10 minutes or so. A 1:1000 solution is used of 0.3-0.5mg adrenaline. Fluid replacement with IV saline is also indicated to correct the intravascular volume redistribution.
A 70 year old man has been newly diagnosed with stage I COPD and has been given inhaled salbutamol PRN and has agreed to stop smoking. What additional measure is needed?
A. Oral prednisolone B. 24% continuous oxygen C. Long acting beta agonist D. Beta agonist via metered dose inhaler E. Antibiotic reserve F. Influenza immunisation G. Intravenous aminophylline H. 40% continuous oxygen I. Beta agonist via nebuliser J. Intravenous hydrocortisone K. Amoxycillin intravenously L. Intramuscular adrenaline
F. Influenza immunisation - The GOLD guidelines published in April 2010 provide a framework for a stepwise approach to treating COPD. Stages of COPD are based on predicted FEV1. For all stages of COPD, influenza vaccination is given yearly and pneumococcal vaccine is given every 5 years. Long term oxygen is only added if there is chronic respiratory failure in stage IV disease. Stage II involves the addition of long acting bronchodilators and rehabilitation. Stage III involves adding inhaled GCs if there are repeated exacerbations.
A 64 year old man has become acutely breathless over the last 4 days, with a productive cough, green sputum & chest tightness. He is a longstanding smoker. Clinically he is hypoxic, with tachycardia, tachypnoea, & central cyanosis. His ABG are as follows: pH 7.35, PaO2 6.7kPa, PaCO2 7.8kPa. He has been given nebulised bronchodilators & intravenous antibiotics & steroids.
A. Oral prednisolone B. 24% continuous oxygen C. Long acting beta agonist D. Beta agonist via metered dose inhaler E. Antibiotic reserve F. Influenza immunisation G. Intravenous aminophylline H. 40% continuous oxygen I. Beta agonist via nebuliser J. Intravenous hydrocortisone K. Amoxycillin intravenously L. Intramuscular adrenaline
B. 24% continuous oxygen - This patient with COPD is having an acute exacerbation. He is being treated accordingly but will need supplemental oxygen as a result of his ABG results. >8kPa is an acceptable level of arterial oxygenation or SaO2 >90%. Ceftriaxone is an acceptable antibiotic. High risk individuals should get tazocin or meropenem (especially if pseudomonas is suspected). Check local antibiotic prescribing policies. Chronic oxygen therapy is indicated for patients with PaO2
A 35 year old man with known asthma who takes low dose inhaled steroids has been using his salbutamol inhaler 8-9 times a day over the last month. He has nocturnal cough & a tight chest early in the morning. There are no signs of infection & his environment hasn’t changed. He is a non-smoker & is allergic to house dust mite. He is physically active ie his peak flow diary shows peak flow of around 85% of expected. What would be the next step in the management of this patient?
A. Oral prednisolone B. 24% continuous oxygen C. Long acting beta agonist D. Beta agonist via metered dose inhaler E. Antibiotic reserve F. Influenza immunisation G. Intravenous aminophylline H. 40% continuous oxygen I. Beta agonist via nebuliser J. Intravenous hydrocortisone K. Amoxycillin intravenously L. Intramuscular adrenaline
C. Long acting beta agonist - This question requires knowledge of stepwise BTS treatment guidelines for adults: Step 1: SABA PRN, Step 2: Plus low-dose inhaled corticosteroids (ICS) , Step 3: Plus LABA, Step 4: Increase dose of ICS or add LTRA, SR theophylline or beta agonist tablet, Step 5: Daily steroid tablet and maintain ICS with specialist care. This patient is still symptomatic on Step 2, so has to be moved up to Step 3. The rest of the question such as his HDM allergy is not particularly relevant in answering this question.
A 9 year old girl attends her GP surgery as an emergency with wheezing, difficulty in breathing & cough. Her PEFR is 250l/min, expected PEFR 500. There is no sign of infection on her chest examination. She is admitted to hospital and given inhaled salbutamol although shows an incomplete response. Which treatment should be given next?
A. Oral prednisolone B. 24% continuous oxygen C. Long acting beta agonist D. Beta agonist via metered dose inhaler E. Antibiotic reserve F. Influenza immunisation G. Intravenous aminophylline H. 40% continuous oxygen I. Beta agonist via nebuliser J. Intravenous hydrocortisone K. Amoxycillin intravenously L. Intramuscular adrenaline
A. Oral prednisolone - Acute asthma exacerbation this time. Bear in mind that treatment guidelines differ for adults and children. The PEFR is 50% predicted making this a moderate exacerbation of asthma (40-69%). A mild exacerbation is defined by a PEFR of 70% of more of predicted whereas severe is defined as 26 to 39% with life-threatening falling under 25%. Note that these values are for children. For moderate exacerbations like this, an oral corticosteroid needs to be prescribed alongside inhaled SABA. If this were life-threatening or severe, then IV may be indicated. It is worth knowing the symptoms and signs of mild, moderate and severe/life threatening asthma. The treatment guidelines that you need to familiarise yourself with tend to be for adults.
A 70 year old male smoker with a 5 year history of productive cough presents with breathlessness. On examination you find hyperinflation & diminished breath sounds when examining the chest.
A. Mitral stenosis B. Pneumonia C. COPD D. Pneumothorax E. Anaemia F. Left ventricular failure G. Thyrotoxicosis H. Epiglottitis I. Asthma J. Anxiety K. Aspirin poisoning L. Pulmonary embolus M. Mitral regurgitation
C. COPD - Smoking is the most important risk factor, accounting for 90% of COPD. COPD has an insidious onset and usually presents in older people with a history of cough, wheeze and SOB. Patients with COPD are at a higher risk of infections and are vaccinated against influenza annually and pneumococcal pneumonia every 5 years. Spirometry is the gold standard for diagnosis, with FEV1/FVC ratio
A 40 year old widowed female presents with a 3 week history of shortness of breath. She also complains of chronic fatigue. On examination she is pale with a pulse of 120.
A. Mitral stenosis B. Pneumonia C. COPD D. Pneumothorax E. Anaemia F. Left ventricular failure G. Thyrotoxicosis H. Epiglottitis I. Asthma J. Anxiety K. Aspirin poisoning L. Pulmonary embolus M. Mitral regurgitation
E. Anaemia - Anaemia is defined by haemoglobin concentration (
An 85 year old male presents with shortness of breath associated with confusion. On examination there is decreased expansion on the left side & the patient with respiratory rate of 35/min.
A. Mitral stenosis B. Pneumonia C. COPD D. Pneumothorax E. Anaemia F. Left ventricular failure G. Thyrotoxicosis H. Epiglottitis I. Asthma J. Anxiety K. Aspirin poisoning L. Pulmonary embolus M. Mitral regurgitation
B. Pneumonia - This patient has pneumonia. Symptoms include chills, fever, cough, SOB and pleuritic chest pain. Examination findings are consistent with his diagnosis. A CXR is the most specific and sensitive test available and antibiotics are indicated.
A 75 year old recently widowed male smoker with a history of angina presents with shortness of breath. He has also vomited & complains of a ringing in his ears. On examination the patient has a BP of 80/50mmHg & fine crackles at both lung bases.
A. Mitral stenosis B. Pneumonia C. COPD D. Pneumothorax E. Anaemia F. Left ventricular failure G. Thyrotoxicosis H. Epiglottitis I. Asthma J. Anxiety K. Aspirin poisoning L. Pulmonary embolus M. Mitral regurgitation
K. Aspirin poisoning - This patient has angina so probably has a stash of aspirin. Tinnitis is common in the early stages of acute salicylate poisoning and reflects CNS toxicity. There may also be deafness and both are reversible. GIT decontamination should be considered as an adjunct on arrival to A&E and activated charcoal can be given. The mainstay of treatment is alkaline diuresis induced by an infusion of sodium bicarbonate. In cases of severe poisoning, it is still started as a bridge to haemodialysis.
An 80 year old female with a history of rheumatic fever in childhood & palpitations presents with shortness of breath. On examination he has an irregularly irregular pulse of 120bpm & loud first heart sound.
A. Mitral stenosis B. Pneumonia C. COPD D. Pneumothorax E. Anaemia F. Left ventricular failure G. Thyrotoxicosis H. Epiglottitis I. Asthma J. Anxiety K. Aspirin poisoning L. Pulmonary embolus M. Mitral regurgitation
A. Mitral stenosis - Practically every single case of mitral stenosis is caused by rheumatic heart disease. The major criteria for rheumatic fever can be remember by CASES: carditis, arthritis, Sydenham’s chorea, erythema marginatum and subcutaneous nodules. The process tends to also cause regurgitation. Mitral stenosis is characteristically a grade 1-2 low pitch murmur heard in mid-diastole which has a rumbling nature and there is no radiation. There can be an associated malar flush, tapping apex beat and a diastolic thrill palpable at the apex, in the 5th intercostal space in the MCL. The first heart sound is also characteristically loud and often this is the most striking feature on ascultation. It is a difficult murmur to pick up so if you are ever asked at this stage to spot this murmur, it will most likely be based on the loud S1. Mitral stenosis is associated with AF caused by LA enlargement (seen by the irregularly irregular pulse).
A 70 year old smoker who has a 6 week history of epigastric discomfort worse on eating. He has lost his appetite as he has a sense of fullness all the time & has lost 3 kilos in weigh. Abdominal examination is unremarkable.
A. History only B. Colonoscopy C. Ultrasound abdomen D. H. pylori antibodies E. Stool examination for pathogens F. Barium meal G. Full blood count, ESR, creatinine, electrolytes & liver function tests H. Gastroscopy I.Barium enema J.Serum Amylase
H. Gastroscopy - Weight loss is one of the most common presenting symptoms in patients with gastric cancer. Epigastric pain is present in about 80% and may resemble that of a gastric ulcer. Although commonly mentioned in EMQs, lymphadenopathy is an uncommon presentation. On examination there may be the presence of a left supraclavicular node (Virchow’s node), a periumbilical nodule (Sister Mary Joseph’s nodule) or a left axillary nodule (Irish node). These are rare findings. A mass may be palpable in advanced disease. More proximal tumours can present with dysphagia. Acanthosis nigricans, a black velvety rash, may also be present.
Strong risk factors include pernicious anaemia, Helicobacter pylori and the consumption of N-nitroso compounds found in cured meats. The peak incidence occurs between 50-70 and men are twice as likely to have gastric cancer. Most are adenocarcinomas. The first test to order for suspected gastric malignancy is an urgent upper GI endoscopy with biopsy of the lesion. The mainstay of treatment is surgical resection unless there is evidence of metastatic disease.
A 45 year old unmarried labourer who increasingly severe central abdominal pain over the last 3 hours. The pain radiates through to his back & makes him vomit. He has no previous history. On examination is cold & sweaty, pulse 120, BP 90/70, & has guarding over his whole abdomen.
A. History only B. Colonoscopy C. Ultrasound abdomen D. H. pylori antibodies E. Stool examination for pathogens F. Barium meal G. Full blood count, ESR, creatinine, electrolytes & liver function tests H. Gastroscopy I.Barium enema J.Serum Amylase
J. Serum Amylase - This patient has acute pancreatitis. He has vomited and is describing mid-epigastric pain radiating around to the back. This pain is classically relieved if the patient curls up and is worse with movement. This patient also vomited too. Complicated haemorrhagic pancreatitis may exhibit Cullen’s sign, Grey-Turner’s sign and Fox’s sign. Make sure you know what these are and you are familiar with the causes of acute pancreatitis. Those caused by hypocalcaemia may display Chvostek’s sign and Trousseau’s sign.
Key to diagnosis is serum amylase or lipase levels which are massively elevated. Prognostic criteria are outlined in Ranson’s criteria applied on admission and after 48 hours, or the modified Glasgow score which you can find in your Oxford Handbook. An abdominal CT is however the most sensitive and specific study and findings may include enlargement of the pancreas with irregular contours, necrosis, pseudocysts and peripancreatic fat obliteration.
For interest, urinary trypsinogen-2 is now considered a better screening test than amylase but is not currently clinically used.
A 20 year old secretary who complains of abdominal discomfort & bloating over the last 3 months. She also has intermittent diarrhoea, but when she opens her bowels, her symptoms are relieved. There is no blood or mucus in her stools. Abdominal & rectal examination is normal.
A. History only B. Colonoscopy C. Ultrasound abdomen D. H. pylori antibodies E. Stool examination for pathogens F. Barium meal G. Full blood count, ESR, creatinine, electrolytes & liver function tests H. Gastroscopy I.Barium enema J.Serum Amylase
A. History only - The intermittent diarrhoea, dyspepsia and bloating without symptoms suggestive of IBD make IBS a more likely diagnosis. IBS is a chronic condition with abdominal pain associated with bowel dysfunction and is a diagnosis of exclusion. The pain or discomfort may be relieved by defecation, as in this case. Examination is usually unremarkable and the diagnosis is based on the patient’s history in line with the Rome Criteria. If the patient presents with any worrying symptoms, then these will warrant a more thorough investigation. Treatment depends on the patient’s predominant symptoms. Antispasmodics relieve abdominal pain or discomfort but do not affect bowel habit. Examples include peppermint oil and dicycloverine. Laxatives can also be used such as lactulose. Lifestyle and dietary modifications combined with reassurance remain the 1st line intervention for functional bowel disease. IBS is linked with stressful jobs such as working as a secretary and there is a female/male ratio of 2:1.
A 50 year old housewife & mother of 5 has sudden severe epigastric pain that radiates to the back on the right & has vomited. She puts it down to her recent meal of fish & chips, as she usually never eats fatty food. Examination is somewhat difficult as she is obese but you think she has some guarding over the epigastium and right hypochondrium.
A. History only B. Colonoscopy C. Ultrasound abdomen D. H. pylori antibodies E. Stool examination for pathogens F. Barium meal G. Full blood count, ESR, creatinine, electrolytes & liver function tests H. Gastroscopy I.Barium enema J.Serum Amylase
C.Ultrasound abdomen - Cholecystitis is acute GB inflammation caused by an obstruction at the cystic duct. It occurs as a major complication of gallstones and classically presents with RUQ pain and fever. Gallstones in EMQs classically involves the Fs (Fat, Forty, Female, Fertile, Fair). USS is the definitive initial investigation. HIDA scanning and MRI may help if the diagnosis remains unclear. Treatment is with cholecystectomy. Make sure you know the difference between ascending cholangitis, cholecystitis and biliary colic.
A 25 year old previously well man who has a 3 day history of abdominal cramps diarrhoea with bloody stools 5 or 6 times a day. Examination shows a soft but tender abdomen.
A. History only B. Colonoscopy C. Ultrasound abdomen D. H. pylori antibodies E. Stool examination for pathogens F. Barium meal G. Full blood count, ESR, creatinine, electrolytes & liver function tests H. Gastroscopy I.Barium enema J.Serum Amylase
E. Stool examination for pathogens - This patient has most likely has infectious diarrhoea. There is a short history in a previous well person. This man has probably eaten something dodgy like a kebab. Do you know the organisms that can cause bloody diarrhoea? Think of the organisms which cause bloody diarrhoea such as EHEC. The mainstay of treatment is rehydration and supportive therapy. Antibiotics may be indicated, particularly in severe cases. The diagnosis would be confirmed by stool examination.
An 82 year old female presents with a large lump which appears at the anus after defecation, & spontaneously on coughing. The lump is uncomfortable & the patient has tenesmus. She also has incontinence & has noticed mucus PR.
A. Irritable bowel syndrome B. Pilonidal sinus C. Haemorrhoids D. Inflammatory bowel disease E. Abscess F. Prolapse G. Fissure H. Fistula I. Intussusception
F. Prolapse - Rectal prolapse tends to present as a mass protruding from the rectum, especially with straining on the toilet. This may be associated with mucous or blood-stained discharge, pain and even faecal incontinence. In contrast, haemorrhoids tend to present with painless PR bleeding or perianal pain with a tender mass in the area. This can be confirmed on examination. Tenesmus is the constant feeling of needing to pass stools, even if there is nothing to pass. It may also be a symptom of rectal carcinoma, and is caused by a SOL.
A 20 year old male presents with painful defecation which persists for 30 mins afterwards. The stool is smeared with blood, & he has noticed recent constipation.
A. Irritable bowel syndrome B. Pilonidal sinus C. Haemorrhoids D. Inflammatory bowel disease E. Abscess F. Prolapse G. Fissure H. Fistula I. Intussusception
G. Fissure - This patient has an anal fissure. This causes severe pain on defecation and may continue for 1 to 2 hours. A small amount of fresh blood is often passed on the stool. Hard stools is a strong risk factor and this patient’s constipation will likely be the cause. Opiates are associated with constipation and subsequently anal fissures too and fissures may also occur in the third trimester of pregnancy or after delivery. Initial treatment is with topical GTN or diltiazem along with supportive measures such a high fibre diet. Resistant or chronic fissures may benefit from surgical measures or botulinum toxin.
A 50 year old female presents with a purulent discharge from the anal region & has recurrent episodes of pain, which is intense & throbbing. On examination there was pruritis ani.
A. Irritable bowel syndrome B. Pilonidal sinus C. Haemorrhoids D. Inflammatory bowel disease E. Abscess F. Prolapse G. Fissure H. Fistula I. Intussusception
E. Abscess - Typically, this presents with perianal pain. The location is important and affects management and diagnosis. If it is inter-sphincteric, then anaesthesia tends to be required to examine the rectal passage adequately for diagnosis – the pain tends to be so bad that a DRE is impossible otherwise. Abscesses found above levator ani may minic an abdominal condition. CT/MRI may be needed to establish the diagnosis. Risk factors include conditions like Crohn’s and anal fistulae (a complication of an abscess can also be a fistula). This accounts for the patient’s symptoms such as pruritis ani. A pilodinal abscess would be difficult to distinguish from an anorectal abscess on history alone but are usually located in the inter-gluteal region and often have a sinus tract in the midline. If the two are difficult to distinguish (such as if the patient cannot be examined), then MRI or CT of the pelvis may be used. Treatment involves drainage of the abscess surgically and the fistula, if present, can also be managed with a fistulotomy or seton insertion. Some patients will also get adjunctive antibiotics, such as the elderly and diabetics.
A 40 year old male presents with PR bleeding & a palpable lump from anus, with associated mucus discharge. There is blood splashed around lavatory pan
A. Irritable bowel syndrome B. Pilonidal sinus C. Haemorrhoids D. Inflammatory bowel disease E. Abscess F. Prolapse G. Fissure H. Fistula I. Intussusception
C. Haemorrhoids - Haemorrhoids are vascular rich cushions in the anal canal and presents, typically, as painless bright PR bleeding or with sudden onset pain in the area associated with a palpable mass. Pruritus ani is common and there is often perianal pain or discomfort. Diagnosis is made visually. Grade 1 is limited to within the anal canal. Grade 2 protrudes but spontaneously reduces when the patient stops straining. Grade 3 protrudes and reduces fully on manual pressure. Grade 4 is irreducible. Treatment includes fibre, ligation, photocoagulation, sclerotherapy or surgical haemorrhoidectomy. Haemorrhoidectomy is the treatment of choice of choice for patients with grade 4 haemorrhoids or for any patient who has failed with more conservative treatment such as sclerotherapy.
A 70 year old male who presents with significant weight loss & progressive painless jaundice. Ultrasound demonstrates a dilated biliary system down to the head of the pancreas.
A. Cholecystitis B. Hepatitis B C. Infectious mononucleosis D. Drug induced hepatitis E. Sickle cell anaemia F. Carcinoma tail of pancreas G. Autoimmune hepatitis H. Gallstone in common bile duct I. Ascending cholangitis J. Hepatitis A K. Cirrhosis L. Carcinoma head of pancreas
L. Carcinoma head of pancreas - Pancreatic cancer (of the head) typically presents with painless obstructive jaundice and weight loss and generally presents late. Whipple’s procedure or Traverso-Longmire procedure (pancreaticoduodenectomy) offers the only hope of a cure but only a small minority are elegible for these procedures. The first tests to order are an abdominal USS and LFTs. Note Courvoisier’s law: Jaundice and a palpable painless gallbladder is unlikely to be caused by gallstones. The tumour marker for pancreatic cancer is CA19-9 which is useful in preoperative staging.
A 46 year old sales rep has recently lost his job & presents with jaundice & ascites. Ultrasound of the biliary tract is normal.
A. Cholecystitis B. Hepatitis B C. Infectious mononucleosis D. Drug induced hepatitis E. Sickle cell anaemia F. Carcinoma tail of pancreas G. Autoimmune hepatitis H. Gallstone in common bile duct I. Ascending cholangitis J. Hepatitis A K. Cirrhosis L. Carcinoma head of pancreas
K. Cirrhosis - This sales rep who has just lost his job has been hitting the bottle. The gynaecomastia is a sign of chronic liver disease and ascites indicates a degree of decompensation. Alcoholic liver disease is the most common cause of cirrhosis in the Western world.
An obese 40 year old woman, with a history of episodic right upper quadrant pain, presents with rapid onset of jaundice with severe abdominal pains, fever & rigors.
A. Cholecystitis B. Hepatitis B C. Infectious mononucleosis D. Drug induced hepatitis E. Sickle cell anaemia F. Carcinoma tail of pancreas G. Autoimmune hepatitis H. Gallstone in common bile duct I. Ascending cholangitis J. Hepatitis A K. Cirrhosis L. Carcinoma head of pancreas
I. Ascending cholangitis - Charcot’s triad of ascending cholangitis: fever with or without rigors, RUQ pain and jaundice. Cholangitis is infection of the biliary tree and can quickly become septic. Drainage of the biliary tree is crucial and is undertaken via ERCP. Make sure you know the difference between ascending cholangitis, cholecystitis and biliary colic.
A 24 year old unemployed rough sleeping male presents unwell, with jaundice, anorexia & lassitude. He has a number of venepuncture marks in the antecubital fossae.
A. Cholecystitis B. Hepatitis B C. Infectious mononucleosis D. Drug induced hepatitis E. Sickle cell anaemia F. Carcinoma tail of pancreas G. Autoimmune hepatitis H. Gallstone in common bile duct I. Ascending cholangitis J. Hepatitis A K. Cirrhosis L. Carcinoma head of pancreas
B. Hepatits B - This could well be HCV although this is not an option. There are HBV risk factors in this patient including IVDU. Serum LFTs will shown a transaminitis. HBsAg, HBcAg and HBeAg can be measured. In HCV, HCV RNA can be measured. HAV is not associated with IVDU. Serum IgM anti-HAV can be measured.
A 20 year old man has been constipated since childhood. He opens his bowels once or twice a week & has noticed faecal soiling.
A. Parkinson’s disease B. Hypercalcaemia C. Colorectal carcinoma D. Diverticular disease E. Hypothyroidism F. Pelvic trauma G. Irritable bowel syndrome H. Chronic laxative abuse I. Hirschsprung’s disease J. Adverse effect of drugs
I. Hirschsprung - In this condition there is colonic obstruction associated with absent intramural ganglion cells. The lumen is hence tonically contracted. Hirschsprung’s disease is commonly diagnosed in the first year of life and presents with vomiting, distension and/or colitis. There tends to be explosure liquid foul stools, delayed meconium passage and fever (enterocolitis). However, sometimes, it can present with intermittent bouts of symptoms later on in life and have minimal or absent symptoms in the first few days/weeks (adult presentations are rare). It can be associated with Down’s and MEN2A.
The diagnosis is definitively made on a rectal biopsy with stain for ganglion cells in the submucosal plexus. This will be absent in Hirschsprung’s with the presence of other features such as thickened non-myelinated nerves and increased acetylcholinesterase. A contrast enema will also be done and will show a contracted distal bowel with the proximal bowel dilated, making the location of the transition zone visible on XR. An AXR is always the first investigation performed but a normal film does not exclude the possibility of this diagnosis (it is a non-specific investigation, though having said that, if there are no distended bowel loops then Hirschsprung’s is unlikely). Manometry is not usually performed, but the reflex where when the rectum is distended, pressure in the anal canal falls (as the internal sphincter relaxes) is absent. Initial treatment is with irrigation followed by surgery.
A 40 year old woman has constipation, weight gain & menorrhagia. She opens her bowels only twice a week. Pulse is 50/min & she has dry skin.
A. Parkinson’s disease B. Hypercalcaemia C. Colorectal carcinoma D. Diverticular disease E. Hypothyroidism F. Pelvic trauma G. Irritable bowel syndrome H. Chronic laxative abuse I. Hirschsprung’s disease J. Adverse effect of drugs
E. Hypothyroidism - This patient has hypothyroidism. Worldwide, the most common cause is iodine deficiency. Other causes include Hashimoto’s or secondary and tertiary hypothyroidism. It can also result from viral de Quervain’s thyroiditis or postpartum thyroiditis. Symptoms include those mentioned (depression, fatigue, weight gain, bradycardia and sluggish reflexes) as well as others such as constipation, cold intolerance, menstrual problems in females, dry skin and muscle cramps. Diagnosis is based on measurement of TSH and thyroid hormones. Treatment is by replacement of T4 with or without T3 in combination. If the patient has normal T3 and T4 but mildly elevated TSH, this is described as subclinical hypothyroidism.
A 35 year old woman has mild long-standing constipation without weight loss. She passes hard pellet-like stools, often with straining & a feeling of incomplete evacuation. Examination is normal.
A. Parkinson’s disease B. Hypercalcaemia C. Colorectal carcinoma D. Diverticular disease E. Hypothyroidism F. Pelvic trauma G. Irritable bowel syndrome H. Chronic laxative abuse I. Hirschsprung’s disease J. Adverse effect of drugs
G. Irritable bowel syndrome - IBS is a chronic condition with abdominal pain associated with bowel dysfunction and is a diagnosis of exclusion. The pain or discomfort may be relieved by defecation. Examination is usually unremarkable and the diagnosis is based on the patient’s history in line with the Rome Criteria. If the patient presents with any worrying symptoms, then these will warrant a more thorough investigation. Treatment depends on the patient’s predominant symptoms. Antispasmodics relieve abdominal pain or discomfort but do not affect bowel habit. Examples include peppermint oil and dicycloverine. Laxatives can also be used such as lactulose. Lifestyle and dietary modifications combined with reassurance remain the 1st line intervention for functional bowel disease. IBS is linked with stressful jobs and there is a female/male ratio of 2:1.
A 50 year old man fell from a ladder & injured his back. He requires regular pain-killers for back pain. Since the accident he has had difficulty opening his bowels & has noticed reduced bowel frequency.
A. Parkinson’s disease B. Hypercalcaemia C. Colorectal carcinoma D. Diverticular disease E. Hypothyroidism F. Pelvic trauma G. Irritable bowel syndrome H. Chronic laxative abuse I. Hirschsprung’s disease J. Adverse effect of drugs
J. Adverse effect of drugs - This patient is on opiates. Let us review opiates such as morphine and codeine. They are commonly prescribed for analgesia and can also be used as an anti-tussive (suppresses ACh and NK release needed for cough activation and inteferes with serotonin receptors in the DRN). They are also abused for their euphoric effects mediated by their action on dopamine release at the nucleus accumbens. They have a host of side effects, including, in this case, GI effects (receptors present on the myenteric and submucosal plexus) of decreased gastric motility, emptying and increased gut water reabsorption… in short, it causes constipation. Other side effects include respiratory depression (desensitises central chemoreceptors to lessen the response to PaCO2), N&V by stimulating the CTZ, pupillary constriction via the Edinger-Westphal nucleus and causes symptoms like pruritis, urticaria, hypotension through histamine release via a direct effect on mast cells.
Arrestins also promote internalisation of opioid receptors which leads to tolerance with prolonged treatment. Prolonged use will also lead to dependance and withdrawl effects if stopped, such as cravings, and the physical effects may be very profound, resembling flu. Naloxone is indicated in an overdose. It is worth looking at the pathways to help you remember the effects.
A 52 year old woman has recently developed constipation & feels that she does not completely empty her rectum on defecation. She has passed blood per rectum on 2 occasions.
A. Parkinson’s disease B. Hypercalcaemia C. Colorectal carcinoma D. Diverticular disease E. Hypothyroidism F. Pelvic trauma G. Irritable bowel syndrome H. Chronic laxative abuse I. Hirschsprung’s disease J. Adverse effect of drugs
C. Colorectal carcinoma - This a rectal carcinoma. Tenesmus, blood and mucus PR alongside weight loss and anorexia are all highly suggestive. Treatment of rectal carcinoma involves surgical excision where possible. This can either be an anterior resection (tumours in the upper 1/3 of the rectum) or an abdominoperineal resection (if the tumour lies lower down). APER involves the formation of a permanent colostomy and has a high incidence of sexual and urinary dysfunction. Anterior resection involves a colo-anal anastamosis.
A 70 year old woman has seen her GP for depression on several occasions. She now complains of abdominal pain, constipation & thirst
A. Parkinson’s disease B. Hypercalcaemia C. Colorectal carcinoma D. Diverticular disease E. Hypothyroidism F. Pelvic trauma G. Irritable bowel syndrome H. Chronic laxative abuse I. Hirschsprung’s disease J. Adverse effect of drugs
B. Hypercalcaemia - Symptoms of high calcium include confusion, constipation, polyuria, polydipsia, depression, kidney stones and lethargy. This can be remembered by ‘stones, bones, abdominal groans and psychiatric moans’. 90% of hypercalcaemia is caused by primary hyperparathyroidism or cancer. Malignancy can cause hypercalcaemia either by direct bony involvement leading to osteolytic lesions or paraneoplastic syndromes involving PTHrp release. The tumour is typically very advanced if hypercalcaemia is a feature. Less common causes include vitamin D overdose, hyperthyroidism, immobilisation, Paget’s and milk-alkali syndrome. The classic bone disease in hyperparathyroidism is osteitis fibrosa cystica which causes pain. The serum PTH level is elevated in primary hyperparathyroidism whereas it may be very low in malignancy due to negative feedback.
A 30 year old male intravenous drug user with a history of tuberculosis develops profuse watery diarrhoea with no abdominal pain.
A. Clostridium difficile B. Staphylococcus aureus C. Chronic pancreatitis D. Drug-induced diarrhoea E. Villous adenoma of the rectum F. Cryptosporidium infection G. Coeliac disease H. Ulcerative colitis I. Irritable bowel syndrome J. Diverticular disease K. Campylobacter L. Crohn’s disease
F. Cryptospradium - I’m sure at some point most of you will see ‘diarrhoea’ and ‘HIV’ together and jump straight to Cryptosporidium. This is a disease caused by a protozoa and diagnosis is made in the lab by detection of oocysts or antigens in stool. The presentation is of watery diarrhoea, often accompanied with severe pain in the tummy, often lasting more than 7 days. It is self-limiting if the patient is immunocompetent but those who are immunocompromised can suffer a chronic sveere course. Those most at risk are those with T cell deficiencies, such as HIV, and those with haematological malignancies, especially children. In immunocompetent people, nitazoxanide can be used in treatment (as can paramomycin). In immunocompromised patients, treatment mainly aims at treating the primary disorder, such as using HAART to improve CD4 cell count and to restore immunity. Protease inhibitors such as ritanovir also act directly to reduce host cell invasion by the sporozoites and reduce parasite development.
A 25 year old male student presents with 12 hours of abdominal pain, vomiting & watery diarrhoea. This has occurred once before.
A. Clostridium difficile B. Staphylococcus aureus C. Chronic pancreatitis D. Drug-induced diarrhoea E. Villous adenoma of the rectum F. Cryptosporidium infection G. Coeliac disease H. Ulcerative colitis I. Irritable bowel syndrome J. Diverticular disease K. Campylobacter L. Crohn’s disease
B. Staphylococcus aureus - This patient has infectious vomiting predominant food poisoning. When vomiting is the main presenting symptom, you should be thinking of Staphylococcus aureus, Bacillus cereus or norovirus. There is a short history in a previous well person. This man has probably eaten something dodgy like a kebab with undercooked chicken, or something like that. The mainstay of treatment is rehydration and supportive therapy. Antibiotics may be indicated, particularly in severe cases.
A 27 year old woman presents with a 2 week history of bloody diarrhoea & abdominal pain. She has also passed mucus per rectum at times
A. Clostridium difficile B. Staphylococcus aureus C. Chronic pancreatitis D. Drug-induced diarrhoea E. Villous adenoma of the rectum F. Cryptosporidium infection G. Coeliac disease H. Ulcerative colitis I. Irritable bowel syndrome J. Diverticular disease K. Campylobacter L. Crohn’s disease
H. Ulcerative colitis - While this could be Crohn’s disease, bloody diarrhoea is more commonly a presentation of UC than Crohn’s. UC is characterised by diffuse mucosal inflammation running a relapsing and remitting course. Bloody diarrhoea is commonly experienced by patients who may also complain of other symptoms such as (lower) abdominal pain, faecal urgency and the host of extra-intestinal manifestations associated with UC. Diagnosis of UC requires endoscopy with biopsy and a negative stool culture to rule out infectious gastroenteritis. Flare ups are usually linked to pathogens so a stool culture will always be needed in these cases. Toxic megacolon is a complication which is associated with a risk of perforation. UC is also linked with bowel adenocarinoma and PSC. Treatment involves mesalazine (5-ASA) used to induce and maintain remission.
A 45 year old man has a long history of drinking excess alcohol. He has a 3 month history of intermittent, severe abdominal pain & diarrhoea with pale, bulky, foul-smelling stools which are hard to flush away
A. Clostridium difficile B. Staphylococcus aureus C. Chronic pancreatitis D. Drug-induced diarrhoea E. Villous adenoma of the rectum F. Cryptosporidium infection G. Coeliac disease H. Ulcerative colitis I. Irritable bowel syndrome J. Diverticular disease K. Campylobacter L. Crohn’s disease
C. Chronic pancreatitis - This is chronic pancreatitis which is most commonly associated with chronic alcohol abuse. Features include the epigastric pain here, which classically radiates to the back, and steatorrhoea from malabsorption, described here with the pale, foul-smelling and difficult to flush stools. There may additionally be DM due to pancreatic failure and the patient may be malnourished. The diagnosis is based on findings and imaging – your options are USS which is less sensitive, or CT, which is more sensitive but involves radiation exposure. AXR is not a sensitive enough test. There is no real definitive treatment, which is mainly symptomatic and the underlying and precipitating factors are treated – in this case, this man’s alcohol excess. Complications of chronic pancreatic imflammation include the development of pseudocysts, calficiation, DM and malabsorption.
A 55 year old man who takes bendrofluazide for hypertension, presents with a 2 month history of watery diarrhoea with occasional blood & mucus mixed in the stool. He has serum potassium of 2.3mmol/l.
A. Clostridium difficile B. Staphylococcus aureus C. Chronic pancreatitis D. Drug-induced diarrhoea E. Villous adenoma of the rectum F. Cryptosporidium infection G. Coeliac disease H. Ulcerative colitis I. Irritable bowel syndrome J. Diverticular disease K. Campylobacter L. Crohn’s disease
E. Villous adenoma of the rectum - Do not be fooled by the bendrofluazide, which is taken by many patients, but does not by itself lead to such profound hypokalaemia (but I’m sure contributes), and in any case would not account for the GI symptoms experienced. This patient has a villous adenoma, which is a type of polyp in the GIT with a malignant potential. Most colorectal cancers arise from an adenoma and polypectomy reduces the incidence of colorectal cancer. The non-neoplastic polyps include hyperplastic ones, hamartomas, inflammatory and lymphoid polyps. Villous adenomas secrete large amounts of mucus and result in hypokalaemia.
Adenomatous polyps are increasingly common with age. FAP and Gardner’s syndrome, for example, predispose to adenomatous polyps (Peutz-Jeghers leads to hamartomatous polyps). This patient is going to need a colonoscopy with biopsy histology to see if the polyp is benign or malignant.
A 40 year old man has just returned from a holiday in Kenya. Since his return, he has developed watery diarrhoea with crampy abdominal pain.
A. Clostridium difficile B. Staphylococcus aureus C. Chronic pancreatitis D. Drug-induced diarrhoea E. Villous adenoma of the rectum F. Cryptosporidium infection G. Coeliac disease H. Ulcerative colitis I. Irritable bowel syndrome J. Diverticular disease K. Campylobacter L. Crohn’s disease
K. Campylobacter - So why is this Campylobacter? Well, it doesn’t have to be. Salmonella, E. Coli, Shigella, Listeria, Vibrio species etc all present with symptoms which are not drastically different and the only way to be sure is to do a stool culture. The only real option here are between Campylobacter, Staphylococcus aureus and Clostridium difficile. However, this patient does not have a history of recent antibiotic use. Staphylococcus tends to present with vomiting as the main feature and the watery diarrhoea here is typical of Campylobacter. UC and CD are chronic conditions (it is worth noting that Yersinia enterocolitis can mimic Crohn’s RLQ pain). This person has most likely eaten something dodgy on holiday. Erythromycin can be used effectively if started early but resistance is a problem and only a small number will benefit. Campylobacter jejuni is the main cause of food poisoning (also coli and fetus species cause disease). Diarrhoea normally resolves in 5-7 days and the patient will need fluid/electrolyte replacement. Campylbacter is one of the infections which is commonly linked to Guillain-Barre (although still a rare phenomenon).
A 21 year old student has been on a drinking binge to celebrate the end of his final exams. He has a 6 hour history of profuse vomiting with small amounts of fresh blood mixed in the vomit. His vital signs are stable.
A. Peptic ulcer B. Mallory-Weiss syndrome C. Angiodysplasia D. Gastric erosions E. Oesophageal varices F. Oesophageal carcinoma G. Peutz-Jeghers syndrome H. GORD I. Osler-Weber-Rendu syndrome J. Mallory-Weiss syndrome K. Carcinoma of the stomach L. Bleeding diathesis
B. Mallory-Weiss Syndrome - This occurs after a rise in abdominal pressure which induces a tear in the oesophageal mucosa, causing subsequent GI bleeding. It commonly presents with haematemesis after an episode of retching/vomiting/coughing/straining. Hence, risk factors include anything which can cause vomiting like heavy alcohol use, which is commonly the case in EMQs. Also, other conditions would include food poisoning, bowel obstruction, hyperemesis gravidarum, bulimia, the chronic cough of COPD, meningitis etc… you name it really. Classically, MWT presents with a small self limiting episode of haematemesis. Definitive diagnosis is made by OGD. Treatment is supportive because most cases, as mentioned, are self limiting and emergency treatment is not offered unless the patient is showing signs of clinical instability. If the patient is actively bleeding, treatment will be with therapeutic endoscopy in most cases, and very very few cases will require more intervention such as angiography with embolisation.
A 24 year old woman has had 24 hours of vomiting & diarrhoea, which she thinks followed eating reheated take-away food. There was fresh blood in the last 3 vomits. Vital signs are stable.
A. Peptic ulcer B. Mallory-Weiss syndrome C. Angiodysplasia D. Gastric erosions E. Oesophageal varices F. Oesophageal carcinoma G. Peutz-Jeghers syndrome H. GORD I. Osler-Weber-Rendu syndrome J. Mallory-Weiss syndrome K. Carcinoma of the stomach L. Bleeding diathesis
B. Mallory-Weiss Syndrome - This occurs after a rise in abdominal pressure which induces a tear in the oesophageal mucosa, causing subsequent GI bleeding. It commonly presents with haematemesis after an episode of retching/vomiting/coughing/straining. Hence, risk factors include anything which can cause vomiting like heavy alcohol use, which is commonly the case in EMQs. Also, other conditions would include food poisoning, bowel obstruction, hyperemesis gravidarum, bulimia, the chronic cough of COPD, meningitis etc… you name it really. Classically, MWT presents with a small self limiting episode of haematemesis. Definitive diagnosis is made by OGD. Treatment is supportive because most cases, as mentioned, are self limiting and emergency treatment is not offered unless the patient is showing signs of clinical instability. If the patient is actively bleeding, treatment will be with therapeutic endoscopy in most cases, and very very few cases will require more intervention such as angiography with embolisation.
A 55 year old man has chronic back pain for which he takes diclofenac. He has epigastric pain after meals & has recently developed black tarry stools & has had an episode of ‘coffee-ground’ vomiting.
A. Peptic ulcer B. Mallory-Weiss syndrome C. Angiodysplasia D. Gastric erosions E. Oesophageal varices F. Oesophageal carcinoma G. Peutz-Jeghers syndrome H. GORD I. Osler-Weber-Rendu syndrome J. Mallory-Weiss syndrome K. Carcinoma of the stomach L. Bleeding diathesis
A. Peptic ulcer - This patient has a bleeding peptic ulcer (the black tarry stools and the coffee-ground vomit from the UGI bleed). Epigastric pain and tenderness related to eating a meal is typical of a peptic ulcer. 80% are duodenal and 20% are gastric. Ulcers may cause iron deficiency anaemia and associated symptoms may feature. Key risk factors are NSAID use, like in this patient, H. pylori infection, smoking and a family history of PUD. Zollinger-Ellison syndrome should be considered if there are multiple ulcers or ulcers refractory to treatment.
Gastric ulcers classically cause pain which is exacerbated by eating and immediately relieved on vomiting. There is usually also weight loss due to a fear of food and its association with pain. Duodenal ulcers are classically made worse by hunger and are relieved by eating and the patient may wake at night with the pain. As a result, weight gain is typically a feature. In reality, it is difficult to differentiate the site of the ulcer based on these features.
The most specific and sensitive test is an upper GI endoscopy which is initially ordered if the patient has ‘red flag’ symptoms, is >55 years of age or fails to respond to treatment. Duodenal ulcers rarely undergo malignant transformation so do not require a compulsory biopsy but gastric ulcers require biopsies to rule this out. In patients who are 55 or younger without ‘red flags’, testing for Helicobacter pylori (breath testing with radiolabelled urea or stool antigen testing) is necessary. Management is aimed at correcting the underlying cause such as discontinuing NSAIDs. H. pylori eradication should be started if the organism is present with triple therapy. Otherwise, a PPI is indicated.
A 35 year old man with a long history of excess drinking of alcohol presents with massive haematemesis. He is also jaundiced, hypotensive & a tachycardia.
A. Peptic ulcer B. Mallory-Weiss syndrome C. Angiodysplasia D. Gastric erosions E. Oesophageal varices F. Oesophageal carcinoma G. Peutz-Jeghers syndrome H. GORD I. Osler-Weber-Rendu syndrome J. Mallory-Weiss syndrome K. Carcinoma of the stomach L. Bleeding diathesis
E. Oesophageal varices - Oesophageal varices occurs as a result of portal hypertension which is a complication of cirrhosis, caused in this patient by his long history of alcohol excess. Other signs may be present such as spider naevi, ascites, caput medusa (vascular collaterals in the abdominal wall), jaundice etc. Splenomegaly is also commonly found and hence patents often have thrombocytopenia and anaemia as a result. The bleeding carries a significant morbidity and mortality, and beta-blockers and/or endoscopic ligation can prevent variceal bleeding prophylactically (though beta blockers are not be used in the acute setting of a variceal bleed – do not get confused here!). Oesophageal varices are basically dilated veins and these can be seen on OGD. Worldwide, HBV and HCV are also major causes of cirrhosis, leading to varices and HIV co-infection can rapidly speed up the progression to cirrhosis in chronic liver failure. The size of the varices is the key predictor of haemorrhage. Acute bleed can be managed with resuscitation, terlipression (DDAVP)/somatostatin analogues/endoscopic ligation. Additionally, a shunt can be deployed and antbiotic prophylaxis started.
A 40 year old woman describes intermittent haemoptysis as well as small amounts of haematemesis. She has telangiectasia on her face.
A. Peptic ulcer B. Mallory-Weiss syndrome C. Angiodysplasia D. Gastric erosions E. Oesophageal varices F. Oesophageal carcinoma G. Peutz-Jeghers syndrome H. GORD I. Osler-Weber-Rendu syndrome J. Mallory-Weiss syndrome K. Carcinoma of the stomach L. Bleeding diathesis
I. Osler-Weber-Rendu synrome - This woman has the facial telangiectasia of OWR, also called hereditary haemorrhagic telangiectasia. This causes abnormal blod vessels pretty much everywhere which are prone to bleed. It is an autosomal dominant condition so a positive FH can often be found.
52 year old man presents with abdominal distension & ankle swelling. He has been drinking 6 pints of beer & half a bottle of whisky a day for some years. On examination he has palmar erythema & spider naevi on his chest.
A. Tuberculous peritonitis B. Heart failure C. Budd Chiari syndrome D. Liver cirrhosis E. Primary liver tumour F. Carcinoma of the ovary G. Bacterial peritonitis H. Primary biliary cirrhosis I. Secondary liver tumours J. Nephrotic syndrome K. Carcinoma of caecum with peritoneal secondaries
D. Liver cirrhosis - Cirrhosis is the end-stage of chronic liver disease, in this case due to alcoholic liver disease. Cirrhosis results in hepatic insufficiency and portal hypertension. This has resulted in the patient’s ascites which is a symptom of decompensated cirrhosis. Other complications include variceal bleeds, jaundice, hepatic encephalopathy, hepatorenal syndrome and the development of HCC. Palmar erythema affects the thenar and hypothenar eminences. Apart from spider naevi and palmar erythema, other signs you might find include telangiectasia, bruising, gynaecomastia, Dupuytren’s contracture, parotid swelling and a red tongue. The patient will have to undergo a diagnostic paracentesis for the ascitic presentation. The treatment of ascites involves restricting salt intake and the use of diuretics (frusemide and spironolactone).
A 50 year old alcoholic man fails to respond to treatment for pancreatitis and has recurrent epigastric pain. There is a palpable epigastric mass. CT scan of the abdomen shows a round well-circumscribed mass in the epigastrium.
A. Hepatoma B. Perforated peptic ulcer C. Sigmoid volvulus D. Splenic rupture E. Pancreatic abscess F. Haematoma of the rectus sheath G. Pancreatic ascites H. Pancratic effusion I. Umbilical hernia J. Oesophageal varices K. Pancreatic pseudocyst L. Divarication of the recti M. Mallory-Weiss tear N. Fractured rib
K. Pancreatic pseudocyst - This patient has developed a pancreatic psuedocyst as a complication of pancreatitis. Pseudocysts are collections of fluids with a high concentration of enzymes. The walls are fibrotic membranes of the peritoneum, mesentery and serosa which stops the fluid from leaking out. The wall is not epithelium and indeed there is no epithelial lining – it is not a real cyst. In patients who fail to respond to treatment, this should be considered as a possible diagnosis. The most common finding is pain, followed by a palpable mass. CT scan is diagnostic. Treatment options include excision, drainage (surgical or percutaneous, or internal e.g. cystojejunostomy Roux-en-Y etc, which I’m sure is going into too much surgery than is necessary now but surgical wannabes can look up the procedures… if they really want to – also cystogastrostomy and cystoduodenostomy).
The pseudocyst can be complicated with infection, rupture and haemorrhage. Pancreatic abscess would give a fever and CT will show a ring-enhancing fluid collection with gas. Treatment would be drainage and antibiotics. Pancreatic ascites is pancreatic fluid accumulating in the peritoneal cavity due to chronic pseudocyst leakage (in most cases, anyway) – there will be weight loss and the ascites will not respond to diuretics. A pancreatic effusion is secondary to a fistula draining into the chest, from the pancreas. These are actually all more common complications to arise from pancreatitis than a pseudocyst.
A 40 year old multiparous woman presents with a midline abdominal mass. The mass is non tender & appears when she is straining. On examination, the midline mass is visible when she raises her head off the examining bed.
A. Hepatoma B. Perforated peptic ulcer C. Sigmoid volvulus D. Splenic rupture E. Pancreatic abscess F. Haematoma of the rectus sheath G. Pancreatic ascites H. Pancratic effusion I. Umbilical hernia J. Oesophageal varices K. Pancreatic pseudocyst L. Divarication of the recti M. Mallory-Weiss tear N. Fractured rib
L. Divaricaction of the recti - Divaricate means to spread apart. The rectus goes from the pubic crest, tubercle and symphysis to the costal cartilages 5,6 and 7, costal margin of 7, sternum and diaphragm. It is innervated by T7-12. Normally, the rectus muscles meet in the midline (linea alba). However, some people have a defect above the umbilicus which causes the gap between the recti to be wider than normal. Hence, when the patient sites up, the rectus muscles will spread apart. Surgica correction is possible but most are asymptomatic. It is not a true hernia, and this is the only option from the list that fits the presentation.
A 19 year old man presents with sudden severe upper abdominal pain after being tackled during rugby practice. He was recently diagnosed with glandular fever.
A. Hepatoma B. Perforated peptic ulcer C. Sigmoid volvulus D. Splenic rupture E. Pancreatic abscess F. Haematoma of the rectus sheath G. Pancreatic ascites H. Pancratic effusion I. Umbilical hernia J. Oesophageal varices K. Pancreatic pseudocyst L. Divarication of the recti M. Mallory-Weiss tear N. Fractured rib
D. Splenic rupture - Infectious mononucleosis caused by EBV infection causes splenomegaly. This makes the patient susceptible to splenic rupture due to trauma, such as during this rugby practice session. Rupture is a cause of splenomegaly, and splenomegaly is a risk factor for rupture. If the patient is previously known to have mono, with an enlarged spleen, then they should really avoid contact sports. It would be an irresponsible doctor to not advise against this. This is a medical emergency as the spleen is a very vascular organ and bleeding can rapidly lead to shock and death.
An 87 year old woman presents with constipation and nausea
A. Hepatoma B. Perforated peptic ulcer C. Sigmoid volvulus D. Splenic rupture E. Pancreatic abscess F. Haematoma of the rectus sheath G. Pancreatic ascites H. Pancratic effusion I. Umbilical hernia J. Oesophageal varices K. Pancreatic pseudocyst L. Divarication of the recti M. Mallory-Weiss tear N. Fractured rib
C. Sigmoid volvulus - While this woman has indeed presented with very non-specific symptoms, the only feasible answer from the list is a sigmoid volvulus. A volvulus is bowel obstruction occuring due to a loop of bowel twisting on its own mesenteric axis. Broadly speaking, there are three types: small bowel, sigmoid and gastric. This is something you need to be able to recognise on AXR and it appears as a dilated loop of large bowel present in the lower abdomen, resembling a coffee bean shape (or like an upside down U shape). The rest of the bowel is usually dilated. For a caecal volvulus, the caecum leaves the RLQ to appear like a second satomach bubble in the centre of the film. There is often associated small bowel dilation. A gastric volvulus is very rare.
A 55 year old male alcoholic presents with vomiting 800ml of blood. His blood pressure is 80/50 with a pulse rate of 120. He also has ascites.
A. Hepatoma B. Perforated peptic ulcer C. Sigmoid volvulus D. Splenic rupture E. Pancreatic abscess F. Haematoma of the rectus sheath G. Pancreatic ascites H. Pancratic effusion I. Umbilical hernia J. Oesophageal varices K. Pancreatic pseudocyst L. Divarication of the recti M. Mallory-Weiss tear N. Fractured rib
J. Oesophageal varices - Oesophageal varices occurs as a result of portal hypertension which is a complication of cirrhosis, caused in this patient by his long history of alcohol excess. Other signs may be present such as spider naevi, ascites, caput medusa (vascular collaterals in the abdominal wall), jaundice etc. Splenomegaly is also commonly found and hence patents often have thrombocytopenia and anaemia as a result. The bleeding (explaining the BP and HR of this patient) carries a significant morbidity and mortality, and beta-blockers and/or endoscopic ligation can prevent variceal bleeding prophylactically (though beta blockers are not be used in the acute setting of a variceal bleed – do not get confused here!). Oesophageal varices are basically dilated veins and these can be seen on OGD. Worldwide, HBV and HCV are also major causes of cirrhosis, leading to varices and HIV co-infection can rapidly speed up the progression to cirrhosis in chronic liver failure. The size of the varices is the key predictor of haemorrhage. Acute bleed can be managed with resuscitation, terlipression (DDAVP)/somatostatin analogues/endoscopic ligation. Additionally, a shunt can be deployed and antbiotic prophylaxis started.
34 year old male with Crohn’s disease had had no bowel motions for 4 days & has been vomiting for 24 hours. Examination reveals a distended abdomen & tinkling bowel sounds.
A. Salmonella B. Viral gastroenteritis C. Appendicitis D. Combined oral contraceptive pill E. Gastric carcinoma F. Intussusception G. Bulimia H. Uraemia I. Bowel obstruction J. Pyloric stenosis K. Pancreatitis L. Oesophageal carcinoma M. Peptic ulcer disease
I. Bowel obstruction - Bowel obstruction has several causes such as adhesions or cancer. The lack of bowel motions, vomiting, distended abdomen and tinkling bowel signs are all indicative. The proximal segment of bowel dilates and distal bowel collapses. Completely obstructed patients generally require surgery. If, on AXR, air is seen to be seeping past the obstruction then the obstruction is partial. As a standard, all patients should be made NBM and given supplemental oxygen, IV fluids and NG decompression (to reduce flow/gastric contents/air towards the obstruction), unless they are rushed off for an emergency laparotomy because, for example, they have complete SBO and are peritonitic.
83 years old man with longstanding heart failure for which he takes digoxin & diuretics. For the last 24 hours he has been vomiting & passed very little urine. On examination he is pale & mildly dehydrated; examination of the abdomen is normal.
A. Salmonella B. Viral gastroenteritis C. Appendicitis D. Combined oral contraceptive pill E. Gastric carcinoma F. Intussusception G. Bulimia H. Uraemia I. Bowel obstruction J. Pyloric stenosis K. Pancreatitis L. Oesophageal carcinoma M. Peptic ulcer disease
H. Uraemia - This patient has developed acute renal failure, probably associated with the longstanding CCF. Advanced heart failure will lead to depressed renal perfusion and ARF. The decreased urine output is a symptom and the vomiting here is caused by uraemia or a general build up of waste products. An acute increase in creatinine will be seen, commonly with hyperkalaemia, hyperphosphataemia and a metabolic acidosis. There may also be respiratory compensation for this. Treatment is largely supportive, managing, in this case, the heart failure, and correcting abnormalities like volume status and the metabolic acidosis. Dialysis may be required.
54 year old publican has 48 hour history of severe epigastric pain & vomiting. On examination he is unwell. Pulse rate is 110/min, BP 130/90. Temp 380C. Upper abdomen very tender. Amylase 1000U/l.
A. Salmonella B. Viral gastroenteritis C. Appendicitis D. Combined oral contraceptive pill E. Gastric carcinoma F. Intussusception G. Bulimia H. Uraemia I. Bowel obstruction J. Pyloric stenosis K. Pancreatitis L. Oesophageal carcinoma M. Peptic ulcer disease
K. Pancreatitis - This patient has acute pancreatitis. He has vomited and is describing mid-epigastric pain radiating around to the back. This patient can also have nausea and vomiting too, with agitation and confusion. A pleural effusion is seen in half of patients with acute pancreatitis. Complicated haemorrhagic pancreatitis may exhibit Cullen’s sign, Grey-Turner’s sign and Fox’s sign. Make sure you know what these are and you are familiar with the causes of acute pancreatitis (GET SMASHED). Those caused by hypocalcaemia may display Chvostek’s sign and Trousseau’s sign.
Key to diagnosis is serum amylase or lipase levels which are massively elevated. Prognostic criteria are outlined in Ranson’s criteria applied on admission and after 48 hours, or the modified Glasgow score which you can find in your Oxford Handbook. An abdominal CT is however the most sensitive and specific study and findings may include enlargement of the pancreas with irregular contours, necrosis, pseudocysts and peripancreatic fat obliteration.
For interest, urinary trypsinogen-2 is now considered a better screening test than amylase but is not currently clinically used.
34 year old man has had vomiting 2-3 times a day for 3 days. Complains of severe crampy abdominal pain & blood stained watery diarrhoea. On examination temp 37.70C, abdomen soft; complains of generalised tenderness. No masses/rebound/guarding.
A. Salmonella B. Viral gastroenteritis C. Appendicitis D. Combined oral contraceptive pill E. Gastric carcinoma F. Intussusception G. Bulimia H. Uraemia I. Bowel obstruction J. Pyloric stenosis K. Pancreatitis L. Oesophageal carcinoma M. Peptic ulcer disease
A. Salmonella - The only option on the list which fits is Salmonella. This could obviously be E coli or Campylobacter too, for instance. This is infectious gastroenteritis and Salmonella is a common cause, pretty much able to contaminate any food. It is commonly linked to poultry, dairy items and undercooked eggs. It is self-limiting and diagnosis is on isolating the organism from a stool culture. Treatment is supportive with fluid and electrolyte replacement and antibiotics are generally used only for patients with risk factors for severe disease or those with extra-GI complications.
A 25 year old travelling salesman is awoken in his hotel with crampy abdominal pain, feeling very ill & vomits 3 times over the next half an hour. He asks the receptionist to call a doctor.
A. Salmonella B. Viral gastroenteritis C. Appendicitis D. Combined oral contraceptive pill E. Gastric carcinoma F. Intussusception G. Bulimia H. Uraemia I. Bowel obstruction J. Pyloric stenosis K. Pancreatitis L. Oesophageal carcinoma M. Peptic ulcer disease
B. Viral gastroenteritis - Viral gastroenteritis often presents with mainly UGI symptoms like N&V more so than diarrhoea. It is on this basis that this is likely to be viral gastroenteritis caused by organisms such as rotavirus, norovirus and adenovirus. Staphylococcus aureus and Bacillus cereus also tend to cause mostly UGI symptoms and acts by preformed toxins, though these are not on the list of available options. The differentiation is made on studying the stool and identifying the organism.
A 30 year old woman who has recently returned from holiday in the Gambia. She is in the 3rd trimester of pregnancy & complains of headaches & fever. On examination her BP is 110/70, there is a soft ESM, shotty lymphadenopathy & hepatosplenomegaly. Examination of the skin is unremarkable. Investigations revealed a Hb of 10.5g/dl, WBC of 5x109/l, platelet count of 80x109/l.
A. RA B. Right heart failure C. Haemachromatosis D. CML E. Malaria F. CLL G. Toxoplasmosis H. Portal vein thrombosis I. Systemic amyloidosis J. Cirrhosis with hepatoma K. Polycythaemia rubra vera L. Congestive cardiac failure M. Malignant melanoma N. Severe emphysema
E. Malaria - In the Western world, almost all cases of malaria occurs in travellers so an adequate travel history is crucial or the diagnosis may be missed. Patients typically present with non-specific symptoms such as a fever, sweats, chills and myalgia. This woman has just returned from an endemic area. Sometimes EMQs will describe patterns of fevers occuring at regular intervals of 48-72 hours associated with P. vivax, P. ovale and P. malariae infections but in most patients there is no specific pattern. Hepatosplenomegaly is a common presenting sign although not common at presentation in a first world setting. Thrombocytopenia is common with falciparum infection and a mild degree of anaemia are commonly seen. WCC can be high, low or normal.
Pregnant women affected by P. falciparum are susceptible to the complications of pregnancy due to placental parasite sequestration. Treatment of malaria in pregnancy must be managed with an ID specialist and should be treated with IV antimalarial therapy.
The test of choice is Giesma-stained thick and thin blood smears. Thick films sensitively detect parasites whereas thin films allow species identification and calculation of parasitaemia to guide treatment. Studies have shown that for P falciparum, the most effective treatment is artesunate which is more effective than quinine without the risk of cinchonism. Numerous studies such as the AQUAMAT study in The Lancet showing that quinine should no longer be the established treatment of choice.
A 78 year old woman attends complaining of widespread itching. Examination reveals hepatosplenomegaly. The patient appears plethoric with no lymphadenopathy
A. RA B. Right heart failure C. Haemachromatosis D. CML E. Malaria F. CLL G. Toxoplasmosis H. Portal vein thrombosis I. Systemic amyloidosis J. Cirrhosis with hepatoma K. Polycythaemia rubra vera L. Congestive cardiac failure M. Malignant melanoma N. Severe emphysema
K. Polycythaemia rubra vera - PRV is a disease of middle and older age and is strongly associated with the JAK2V617 mutation. Pruritis is a common feature and is often severe and evoked by contact with water. Facial redness and fullness is commonly observed and splenomegaly is a common finding. It is a myeloproliferative disorder with raised Hct, Hb and RBC count. Blood hence becomes very viscous. There is a clear link between Budd-Chiari syndrome and subsequent PRV. Treatment is with venesection. Around 30% will go on to develop myelofibrosis.
A 78 year old woman attends complaining of recent onset of tiredness. She is pale, has hepatosplenomegaly and generalised painless lymphadenopathy in the neck, axillae and groin. Coombs’ (DAT) test is positive.
A. RA B. Right heart failure C. Haemachromatosis D. CML E. Malaria F. CLL G. Toxoplasmosis H. Portal vein thrombosis I. Systemic amyloidosis J. Cirrhosis with hepatoma K. Polycythaemia rubra vera L. Congestive cardiac failure M. Malignant melanoma N. Severe emphysema
F. CLL - This elderly woman has CLL. CLL presents in older adults (generally >60) and is often asymptomatic. Smear cells can be seen in peripheral blood smear and it is associated with a warm type AIHA accounting for her pallor and fatigue (hence the Coombs’ test is positive). Painless lymphadenopathy may be present and splenomegaly is a common finding. A WCC with differential is required to make a diagnosis. An absolute lymphocytosis will be seen. CML is not associated with an AIHA and tends to present at a younger age.
A 60 year old woman is found to have hepatomegaly. She has a history of moderate alcohol use. She had an anterior MI 2 years previously. Examination reveals significant ankle oedema, elevated JVP & 1-2 spider naevi on her chest.
A. RA B. Right heart failure C. Haemachromatosis D. CML E. Malaria F. CLL G. Toxoplasmosis H. Portal vein thrombosis I. Systemic amyloidosis J. Cirrhosis with hepatoma K. Polycythaemia rubra vera L. Congestive cardiac failure M. Malignant melanoma N. Severe emphysema
L. COngestive heart failure - The significant ankle oedema and raised JVP point towards right sided heart failure.The presence of 1-2 spider naevi is completely normal. Initial investigations should include ECG, CXR, TTE and bloods including BNP levels. First line treatment is with an ACE inhibitor which reduces morbidity and mortality. Salt and fluid restriction is also beneficial. All patients with chronic heart failure will also receive a beta blocker such as carvedilol. Other adjuncts include spironolactone, diuretics, hydralazine and a nitrate, and digoxin.
A 50 year old male with haemophilia & hepatitis C presents with weight loss & abdominal discomfort. He is mildly icteric with features of chronic liver disease & a large left lobe of the liver
A. RA B. Right heart failure C. Haemachromatosis D. CML E. Malaria F. CLL G. Toxoplasmosis H. Portal vein thrombosis I. Systemic amyloidosis J. Cirrhosis with hepatoma K. Polycythaemia rubra vera L. Congestive cardiac failure M. Malignant melanoma N. Severe emphysema
J. Cirrhosis with hepatoma - HCV has caused this patient’s hepatic cirrhosis which has resulted in a hepatoma (HCC). It is likely that his HCV infection has resulted from contaminated blood products due to his haemophilia. This was a key problem before blood donor screening took place. Unlike HBV, HCV infection almost always results in cirrhosis before a hepatoma develops. The length of time the patient has HCV is a good correlate to the development of HCC. HCV is also strongly associated with IVDU which accounts for most infections. Treatment of the hepatoma is guided by staging and prognosis. Treatment includes resection, transplant, percutaneous ablation and chemo-embolisation.
A 65 year old heavy smoker. He has been progressively short of breath over a few years. He has a smooth liver edge 2cms below the costal margin.
A. RA B. Right heart failure C. Haemachromatosis D. CML E. Malaria F. CLL G. Toxoplasmosis H. Portal vein thrombosis I. Systemic amyloidosis J. Cirrhosis with hepatoma K. Polycythaemia rubra vera L. Congestive cardiac failure M. Malignant melanoma N. Severe emphysema
N. Severe emphysema - The liver is palpable in this man because severe emphysema has resulted in hyperexpanded lung fields.
A 25 year old female trainee solicitor presents complaining of bloating & excessive flatus. She is passing pellet-like stools associated with abdominal pains. Her symptoms have been intermittent for several years
A. Radiation proctitis B. Infectious diarrhoea C. Colorectal cancer D. Diverticular disease E. Irritable bowel syndrome F. Benign colonic stricture G. Hyperthyroidism H. Inflammatory bowel disease I. Ischaemic colitis
E. Irritable bowel syndrome - IBS is a chronic condition with abdominal pain associated with bowel dysfunction and is a diagnosis of exclusion. The pain or discomfort may be relieved by defecation. Examination is usually unremarkable and the diagnosis is based on the patient’s history in line with the Rome Criteria. If the patient presents with any worrying symptoms, then these will warrant a more thorough investigation. Treatment depends on the patient’s predominant symptoms. Antispasmodics relieve abdominal pain or discomfort but do not affect bowel habit. Examples include peppermint oil and dicycloverine. Laxatives can also be used such as lactulose. Lifestyle and dietary modifications combined with reassurance remain the 1st line intervention for functional bowel disease. IBS is linked with stressful jobs such as working as a secretary and there is a female/male ratio of 2:1. ‘Pellet-like’ stools is an indicator in EMQs.
A 75 year old man with a 6 month history of straining at stool. He also thinks he is not emptying his rectum completely. He is passing blood & mucus per rectum. He has some weight loss & anorexia.
A. Radiation proctitis B. Infectious diarrhoea C. Colorectal cancer D. Diverticular disease E. Irritable bowel syndrome F. Benign colonic stricture G. Hyperthyroidism H. Inflammatory bowel disease I. Ischaemic colitis
C. Colorectal cancer - This a rectal carcinoma. Tenesmus, blood and mucus PR alongside weight loss and anorexia are all highly suggestive. Treatment of rectal carcinoma involves surgical excision where possible. This can either be an anterior resection (tumours in the upper 1/3 of the rectum) or an abdominoperineal resection (if the tumour lies lower down). APER involves the formation of a permanent colostomy and has a high incidence of sexual and urinary dysfunction. Anterior resection involves a colo-anal anastamosis.
A 32 year old female presents with a 4 week history of bloody liquid stool with mucus, 9 times a day. She has anorexia, weight loss & anaemia.
A. Radiation proctitis B. Infectious diarrhoea C. Colorectal cancer D. Diverticular disease E. Irritable bowel syndrome F. Benign colonic stricture G. Hyperthyroidism H. Inflammatory bowel disease I. Ischaemic colitis
H. Inflammatory bowel disease - This patient gives a history of IBD. The history would be more suggestive of UC where the mainstay of treatment is with 5-ASA. A colonoscopy is required to assess the extent of disease and for a definitive diagnosis. Biopsy in CD will show transmural granulomatous inflammation. CD can affect the whole GIT but favours the TI and proximal colon and is macroscopically characterised by skip lesions. UC on the other hand is characterised by the presence of crypt abscesses, which is pathognomic. CD risk is increased 3-4 fold by smoking whereas smoking seems protective in UC. The mainstay of treatment in CD is with steroids and azathioprine to revent relapses and for those suffering side effects of steroid treatment. TNF-alpha inhibitors also have a role. Surgery in CD is only indicated in a small number of patients who bleed, for bowel perforation and cases of complete obstruction. The aim is to rest distal disease by temporarily diverting faecal flow.
A 19 year old male returns from a recent back packing holiday in India. He is passing bloody liquid stools about 15 times a day. He has lassitude, anorexia & a temperature of 37.50C
A. Radiation proctitis B. Infectious diarrhoea C. Colorectal cancer D. Diverticular disease E. Irritable bowel syndrome F. Benign colonic stricture G. Hyperthyroidism H. Inflammatory bowel disease I. Ischaemic colitis
B. Infectious diarrhoea - This patient has low grade pyrexia and recent foreign travel making infectious diarrhoea very likely. This man has probably eaten something dodgy while on holiday in India. Think of the organisms which cause bloody diarrhoea such as EHEC. E coli is the most common cause of traveller’s diarrhoea. There may be a contact history due to faecal-oral spread. The mainstay of treatment is rehydration and supportive therapy. Antibiotics may be indicated, particularly in severe ETEC.
A 65 year old man had an elective aortic aneurysm repair 5 days ago. He now has abdominal distension & left sided abdominal pain. He is passing a small amount of blood & mucus per rectum.
A. Radiation proctitis B. Infectious diarrhoea C. Colorectal cancer D. Diverticular disease E. Irritable bowel syndrome F. Benign colonic stricture G. Hyperthyroidism H. Inflammatory bowel disease I. Ischaemic colitis
I. Ischaemic collitis - Ischaemic colitis causes focal or diffuse abdominal pain and often has a more insidious onset than mesenteric ischaemia (over several hours or days). The recent operation in the approximate area has resulted in an incomplete blood supply to that part of the bowel. Mesenteric ischaemia and ischaemic colitis all form part of ‘ischaemic bowel disease’.
A 71 year old man who has had a MI 6 months ago presents with shortness of breath & fatigue. On examination, the JVP is raised. He has pitting oedema to the knees. There is tenderness in the right upper quadrant with a smooth liver edge at 5cm.
A. Tuberculous peritonitis B. Heart failure C. Budd Chiari syndrome D. Liver cirrhosis E. Primary liver tumour F. Carcinoma of the ovary G. Bacterial peritonitis H. Primary biliary cirrhosis I. Secondary liver tumours J. Nephrotic syndrome K. Carcinoma of caecum with peritoneal secondaries
B. Heart failure - This patient has heart failure, which has possibly occured as a consequence of his MI. SOB indicates pulmonary oedema due to LV failure. The raised JVP, peripheral oedema and tender hepatomegaly indicates RV failure. Initial investigations should include ECG, CXR, TTE and bloods including BNP levels. First line treatment is with an ACE inhibitor which reduces morbidity and mortality. Salt and fluid restriction is also beneficial. All patients with chronic heart failure will also receive a beta blocker such as carvedilol. Other adjuncts include spironolactone, diuretics, hydralazine and a nitrate, and digoxin.
A 50 year old man with a 1 month history of progressive abdominal distension preceded by increased tiredness, shortness of breath on exertion & weight loss of 10kg. There is a non-tender irregular mass in the right iliac fossa.
A. Tuberculous peritonitis B. Heart failure C. Budd Chiari syndrome D. Liver cirrhosis E. Primary liver tumour F. Carcinoma of the ovary G. Bacterial peritonitis H. Primary biliary cirrhosis I. Secondary liver tumours J. Nephrotic syndrome K. Carcinoma of caecum with peritoneal secondaries
K. Carcinoma of the caecum with peritoneal secondaries - The weight loss of 10kg, fatigue and non-tender irregular RIF mass point to caecal carcinoma. Right sided colorectal cancer tends to present with anaemic symptoms. Almost 90% are anaemic at diagnosis. The progressive abdominal distension indicates the presence of peritoneal secondaries, which causes vague symptoms. Treatment in this case for a cancer that has become widely metastatic will be palliative.
A 20 year old man presents with generalised swelling of the limbs, face & abdomen. Tests show that he has a normal blood count & liver function tests, except a low albumin level of 18g/L and a high cholesterol of 9.5mmol/L.
A. Tuberculous peritonitis B. Heart failure C. Budd Chiari syndrome D. Liver cirrhosis E. Primary liver tumour F. Carcinoma of the ovary G. Bacterial peritonitis H. Primary biliary cirrhosis I. Secondary liver tumours J. Nephrotic syndrome K. Carcinoma of caecum with peritoneal secondaries
J. Nephrotic syndrome - Nephrotic syndrome is the triad of proteinuria (>3.5g/24h), hypoalbuminaemia (
A 35 year old woman has a 10 year history of low retrosternal dysphagia & painless regurgitation of food in the mouth
A. Cerebrovascular accident B. Carcinoma of oesophagus C. Plummer-Vinson syndrome D. Gastric volvulus E. Hiatus hernia F. Pneumonia G. Myasthenia gravis H. Thyroid goitre I. Carcinoma of bronchus J. Pharyngeal pouch K. Achalasia
E. Hiatus hernia - A hiatus hernia is where intraabdominal contents protrude through the oesophageal hiatus of the diaphragm. Risk factors inclyde obesity and high intra-abdominal pressure. The condition may be asymptomatic, or it may present with symptoms (which are non-specific) such as heartburn, dysphagia, pain on swallowing, wheezing, hoarseness and chest pain. A CXR is the first test done and may show an air bubble in the wrong place but barium studies are diagnostic and treatment depends on the symptoms and anatomy of the hernia. Hernias can be sliding or rolling (or mixed, or giant), uncomplicated or complicated by, for instance, obstruction and bleeding. Do you know the difference between a sliding and a rolling hiatal hernia?
A 65 year old woman has progressive low retrosternal dysphagia, initially to solids, but now also to liquids – coming on for the last 4 months. There has also been loss of appetite & 3kg weight loss.
A. Cerebrovascular accident B. Carcinoma of oesophagus C. Plummer-Vinson syndrome D. Gastric volvulus E. Hiatus hernia F. Pneumonia G. Myasthenia gravis H. Thyroid goitre I. Carcinoma of bronchus J. Pharyngeal pouch K. Achalasia
B. Carcinoma of oesophagus - Dysphagia (in this progressive pattern) coupled with weight loss points to malignancy. Dysphagia occurs when there is obstruction of more than 2/3 of the lumen and presence indicates locally advanced disease. There may additionally be odynophagia. Men are twice as likely to develop oesophageal cancer. GORD, Barrett’s oesophagus, FH, tobacco and alcohol are all risk factors. The two main types are squamous cell carcinoma and adenocarcinoma. Tumours in the upper 2/3 of the oesophagus are SCC whereas those that lie in the lower 1/3 are adenocarcinomas. The main test to order is an OGD with biopsy. Treatment is either surgical resection or with chemo or radiotherapy alongside endoscopic ablation with or without stenting and brachytherapy.
A 45 year old lady presents with high retrosternal dysphagia. She has spoon-shaped nails & is noted to be pale.
A. Cerebrovascular accident B. Carcinoma of oesophagus C. Plummer-Vinson syndrome D. Gastric volvulus E. Hiatus hernia F. Pneumonia G. Myasthenia gravis H. Thyroid goitre I. Carcinoma of bronchus J. Pharyngeal pouch K. Achalasia
C. Plummer-Vinson syndrome - Plummer-Vinson syndrome is the association of chronic Iron Deficiency Anaemia (IDA) (shown here by the koilonychia and paleness on examination) with dysphagia due to a post cricoid web. Roughly 7% of those with IDA may complain of gradual onset dysphagia with the discomfort found in the area of the cricoid cartilage. Invasive procedures may be needed for management such as endoscopic dilation of the web but treatment is largely aimed at correcting the IDA.
A 40 year old man presents with dysphagia that worsens as he eats. He has droopy eyelids & sometimes has difficulty in speaking.
A. Cerebrovascular accident B. Carcinoma of oesophagus C. Plummer-Vinson syndrome D. Gastric volvulus E. Hiatus hernia F. Pneumonia G. Myasthenia gravis H. Thyroid goitre I. Carcinoma of bronchus J. Pharyngeal pouch K. Achalasia
G. Myesthenia gravis - Myasthenia gravis is an autoimmune condition with antibodies affecting the NMJ, mostly the nAChR at the post-synaptic muscle membrane. Although some have antibodies against MuSK, and there are other proteins involved. MG is characterised by muscle weakness which increases with exercise (fatigue, demonstrated here as the dysphagia gets worse as he eats, unlike Lambert-Eaton myasthenic syndrome). Commonly, presentations include diplopia and drooping eyelids like this patient, and there may also be SOB, proximal limb weakness, facial paresis and oropharyngeal weakness. MG is associated with thymic hyperplasia in 70% or thymoma in 10%, and these associations can also crop up in EMQs. There will be elevated serum AChR receptor antibody titres or MuSK antibodies. Electrophysiology will demonstrate a decremental response on repetitive nerve stimulation. Treatment includes anticholinesterases (pyridostigmine, and immunotherapy. Patients may also require a thymectomy. Some 15-20% may experience a myasthenic crisis (which needs mechanical ventilation). Do you know what the Tensilon test is and why edrophonium is given in this test?
A 50 year old describes a 5 month history of heartburn and cramp-like chest pain relived by drinking cold water, both unrelated to food. There has also been intermittent dysphagia to both liquids and solids, regurgitation and weight loss of 2kg.
A. Cerebrovascular accident B. Carcinoma of oesophagus C. Plummer-Vinson syndrome D. Gastric volvulus E. Hiatus hernia F. Pneumonia G. Myasthenia gravis H. Thyroid goitre I. Carcinoma of bronchus J. Pharyngeal pouch K. Achalasia
K. Achalasia - This is achalasia which is a motility disorder with loss of peristalsis in the distal oesophagus and failure of the LOS to relax in response to swallowing. This presents commonly with dysphagia to both liquids and solids, regurgitation and retrosternal chest pain, which can be slowly progressive over time. In structural obstruction such as cancer, dysphagia to liquids is uncommon unless the disease is very advanced. Retrosternal pressure experienced can be precipitated by drinking liquids but is eased by continuing to drink, and the pain may be relieved by cold water. This may wake the individual from sleep. A UGI endoscopy is needed to exclude malignancy as a cause of dysphagia. The diagnosis is established on manometry or barium studies. Treatment is symptomatic.
A 45 year old lady presenting with intense pruritus, joint pains & tiredness. She has also noticed that she is becoming yellow. On examination she was clubbed with xanthelasma around her eyes. She has dry eyes and a dry mouth. Her spleen was also palpably enlarged.
A. Reflux oesophagitis B. Cancer of the liver C. Hiatus hernia D. Cancer of the pancreas E. Gastric ulcer F. Liver cirrhosis G. Irritable bowel syndrome H. Coeliac’s disease I. Carcinoma of oesophagus J. Duodenal ulcer K. Inflammatory bowel disease L. Chronic hepatitis M. Primary biliary cirrhosis N. Pancreatitis
M. Primary biliary cirrhosis - Primary biliary cirrhosis (PBC) is a chronic condition where the intrahepatic small bile ducts are progressively damaged (and eventually lost) occuring on a background of portal tract inflammation. Fibrosis develops, ultimately leading to cirrhosis (which is defined as fibrosis with nodular regeneration). It is widely believed to be autoimmune in aetiology as almost all patients have AMA. The pointers in this question which would raise your suspicion, is xanthelasma around the eyes, pruritis in the absence of an obvious dermatological cause, fatigue and the features of liver disease typical of cirrhosis (jaundice) and splenomegaly as a feature of portal hypertension. The patient also has dry eyes and dry mouth from associated Sjogren’s syndrome. The joint pains could indicate RA.
A 70 year old man presents with general malaise, weakness & right upper quadrant abdominal pain. On examination he looked ill & was clinically jaundiced. There were spider naevi, palmar erythema, leuconychia, Dupuytren’s contracture & gynaecomastia.
A. Reflux oesophagitis B. Cancer of the liver C. Hiatus hernia D. Cancer of the pancreas E. Gastric ulcer F. Liver cirrhosis G. Irritable bowel syndrome H. Coeliac’s disease I. Carcinoma of oesophagus J. Duodenal ulcer K. Inflammatory bowel disease L. Chronic hepatitis M. Primary biliary cirrhosis N. Pancreatitis
F. Liver cirrhosis - Cirrhosis is the end-stage of chronic liver disease, resulting in hepatic insufficiency and portal hypertension. This has resulted in this man’s jaundice. Palmar erythema affects the thenar and hypothenar eminences. Other signs include spider naevi, telangiectasia, Dupuytren’s contracture, parotid swelling, leuconychia from hypoalbuminaemia, gynaecomastia and bruising. Management is aimed at treating the underlying liver disease. The only curative option, once decompensated, is liver transplantation.
A 25 year old lady presented with fever, abdominal pains & weight loss. She was opening her bowels x 10-12/day with blood & mucus.
A. Reflux oesophagitis B. Cancer of the liver C. Hiatus hernia D. Cancer of the pancreas E. Gastric ulcer F. Liver cirrhosis G. Irritable bowel syndrome H. Coeliac’s disease I. Carcinoma of oesophagus J. Duodenal ulcer K. Inflammatory bowel disease L. Chronic hepatitis M. Primary biliary cirrhosis N. Pancreatitis
K. Inflammatory bowel disease - This patient gives a history of IBD. The history would be more suggestive of UC (but could be CD) where the mainstay of treatment is with 5-ASA. A colonoscopy is required to assess the extent of disease and for a definitive diagnosis. Biopsy in CD will show transmural granulomatous inflammation. CD can affect the whole GIT but favours the TI and proximal colon and is macroscopically characterised by skip lesions. UC on the other hand is characterised by the presence of crypt abscesses, which is pathognomic. CD risk is increased 3-4 fold by smoking whereas smoking seems protective in UC. The mainstay of treatment in CD is with steroids and azathioprine to revent relapses and for those suffering side effects of steroid treatment. TNF-alpha inhibitors also have a role. Surgery in CD is only indicated in a small number of patients who bleed, for bowel perforation and cases of complete obstruction. The aim is to rest distal disease by temporarily diverting faecal flow.
A 40 year old anxious lady with intermittent abdominal pain relieved by defecation. Her abdomen feels distended & her stool, alternated between diarrhoea & constipation.
A. Reflux oesophagitis B. Cancer of the liver C. Hiatus hernia D. Cancer of the pancreas E. Gastric ulcer F. Liver cirrhosis G. Irritable bowel syndrome H. Coeliac’s disease I. Carcinoma of oesophagus J. Duodenal ulcer K. Inflammatory bowel disease L. Chronic hepatitis M. Primary biliary cirrhosis N. Pancreatitis
G. Irritable bowel syndrome - IBS is a chronic condition with abdominal pain associated with bowel dysfunction and is a diagnosis of exclusion. The pain or discomfort may be relieved by defecation. Examination is usually unremarkable and the diagnosis is based on the patient’s history in line with the Rome Criteria. If the patient presents with any worrying symptoms, then these will warrant a more thorough investigation. Treatment depends on the patient’s predominant symptoms. Antispasmodics relieve abdominal pain or discomfort but do not affect bowel habit. Examples include peppermint oil and dicycloverine. Laxatives can also be used such as lactulose. Lifestyle and dietary modifications combined with reassurance remain the 1st line intervention for functional bowel disease. IBS is linked with stressful jobs such as working as a secretary and there is a female/male ratio of 2:1. ‘Pellet-like’ stools is an indicator in EMQs.
A 70 year old diabetic man, who smoked 40 cigs/day for 40 years. He presents with abdominal pain worse at night & radiating to his back. He is losing weight, suffers from dyspepsia & pruritus. On examination he is cachectic, jaundiced & has an enlarged gallbladder.
A. Reflux oesophagitis B. Cancer of the liver C. Hiatus hernia D. Cancer of the pancreas E. Gastric ulcer F. Liver cirrhosis G. Irritable bowel syndrome H. Coeliac’s disease I. Carcinoma of oesophagus J. Duodenal ulcer K. Inflammatory bowel disease L. Chronic hepatitis M. Primary biliary cirrhosis N. Pancreatitis
D. Cancer of the pancreas - Pancreatic cancer typically presents with painless obstructive jaundice and weight loss and generally presents late. It can however, like this case, present with abdominal pain which is typically non-specific in the upper abdomen. If the patient presents with persistent back pain, then this symptom is consistent with retroperitoneal metastases. It is estimated that 1 in 4 cases can be linked to smoking. Whipple’s procedure or Traverso-Longmire procedure (pancreaticoduodenectomy) offers the only hope of a cure but only a small minority are elegible for these procedures. The first tests to order are an abdominal USS and LFTs. Note Courvoisier’s law with regard to this question: Jaundice and a palpable painless gallbladder is unlikely to be caused by gallstones. The tumour marker for pancreatic cancer is CA19-9 which is useful in preoperative staging.
A 50 year old man: BR 50umol/l, ALP 200iu/l, ALT 120iu/l, GGT 600iu/l.
A. Alcoholic cirrhosis B. Gilbert’s syndrome C. Acute pancreatitis D. Hepatocellular carcinoma E. Paracetamol overdose F. Cholangiocarcinoma G. Drug-induced cholestasis H. CMV infection I. Haemolysis J. Pancreatic cancer K. Wilson’s disease L. Gallstones M. Viral hepatitis
A. Alcoholic cirrhosis - Cirrhosis is the end-stage of chronic liver disease, in this case due to alcoholic liver disease. Cirrhosis results in hepatic insufficiency and portal hypertension. The high GGT here is as a result of high alcohol consumption.
Liver function tests are mistakenly named really because they don’t test liver function. They are better called ‘liver tests’ or ‘tests of liver chemistry’. The tests of liver synthetic function come from measuring albumin levels (produced by the liver) and the prothrombin time, as the liver has a key role in the manufacture of clotting factors. Some general points on liver tests to consider: These tests tend to include bilirubin (breakdown product of RBC after hepatic conjugation, and is secreted in the biliary system), AST, ALT, GGT, ALP (alk phos) and LDH. It’s really all about pattern recognition. Isolated elevation of liver tests tends to make you think of non-hepatic causes and you should take into account the patient’s history during interpretation. A normal liver panel does not mean that the person does not have liver disease, and liver tests are elevated in a small percentage of normals.
You can split the possible causes into three broad categories. It is worth noting that people with any pattern can have cirrhosis. AST is also present in heart, skeletal muscle, kidney, brain and in RBCs too. 80% of AST is found in the mitochondria and 20% in the cytoplasmic matrix. ALP, which is a cytoplasmic enzyme, can come from bone, kidney, intestines or the placenta. GGT may come from the heart or RBCs.
The first is predominantly elevated AST/ALT, which is described as a hepatocellular pattern and occurs in conditions such as viral hepatitis, alcoholic disease, Wilson’s etc. Here the hepatocytes get damaged and enzymes leak out.
The second is predominantly elevated ALP (GGT too, but an isolated rise in GGT is common so many hospitals remove GGT from the liver panel. GGT is only useful if there is an isolated rise in ALP). This is seen in a cholestatic (here, the hepatobiliary system is affected) or infiltrative pattern in bile duct obstruction, malignancy e.g. HCC or pancreatic, PBC/PSC etc or infiltrative conditions such as TB and lymphomas.
The last is an isolated hyperbilirubinaemia seen in haemolysis, intra-abdominal bleeding or with conditions such as Gilbert’s. BR elevation can be further divided into conjugated or unconjugated. Unconjugated is normally due to breakdown of RBC beyond the liver’s capacity to conjugate. Conjugated occurs in liver disease and problems with obstructed bile flow.
A 50 year old man: BR 110umol/l, ALP 300iu/l, ALT 110iu/l, AFP is elevated.
A. Alcoholic cirrhosis B. Gilbert’s syndrome C. Acute pancreatitis D. Hepatocellular carcinoma E. Paracetamol overdose F. Cholangiocarcinoma G. Drug-induced cholestasis H. CMV infection I. Haemolysis J. Pancreatic cancer K. Wilson’s disease L. Gallstones M. Viral hepatitis
D. Hepatocellular carcinoma - The pattern here is cholestatic. Furthermore, elevated AFP is given, which is a tumour marker for HCC, differentiating this from other potential causes like pancreatic cancer. AST and ALT may be normal or elevated.
Liver function tests are mistakenly named really because they don’t test liver function. They are better called ‘liver tests’ or ‘tests of liver chemistry’. The tests of liver synthetic function come from measuring albumin levels (produced by the liver) and the prothrombin time, as the liver has a key role in the manufacture of clotting factors. Some general points on liver tests to consider: These tests tend to include bilirubin (breakdown product of RBC after hepatic conjugation, and is secreted in the biliary system), AST, ALT, GGT, ALP (alk phos) and LDH. It’s really all about pattern recognition. Isolated elevation of liver tests tends to make you think of non-hepatic causes and you should take into account the patient’s history during interpretation. A normal liver panel does not mean that the person does not have liver disease, and liver tests are elevated in a small percentage of normals.
You can split the possible causes into three broad categories. It is worth noting that people with any pattern can have cirrhosis. AST is also present in heart, skeletal muscle, kidney, brain and in RBCs too. 80% of AST is found in the mitochondria and 20% in the cytoplasmic matrix. ALP, which is a cytoplasmic enzyme, can come from bone, kidney, intestines or the placenta. GGT may come from the heart or RBCs.
The first is predominantly elevated AST/ALT, which is described as a hepatocellular pattern and occurs in conditions such as viral hepatitis, alcoholic disease, Wilson’s etc. Here the hepatocytes get damaged and enzymes leak out.
The second is predominantly elevated ALP (GGT too, but an isolated rise in GGT is common so many hospitals remove GGT from the liver panel. GGT is only useful if there is an isolated rise in ALP). This is seen in a cholestatic (here, the hepatobiliary system is affected) or infiltrative pattern in bile duct obstruction, malignancy e.g. HCC or pancreatic, PBC/PSC etc or infiltrative conditions such as TB and lymphomas.
The last is an isolated hyperbilirubinaemia seen in haemolysis, intra-abdominal bleeding or with conditions such as Gilbert’s. BR elevation can be further divided into conjugated or unconjugated. Unconjugated is normally due to breakdown of RBC beyond the liver’s capacity to conjugate. Conjugated occurs in liver disease and problems with obstructed bile flow.
A 35 year old woman: BR 80umol/l, ALP 300iu/l, ALT 30iu/l, GGT 30iu/l. Abdominal USS shows biliary dilation. The patient has RUQ tenderness.
A. Alcoholic cirrhosis B. Gilbert’s syndrome C. Acute pancreatitis D. Hepatocellular carcinoma E. Paracetamol overdose F. Cholangiocarcinoma G. Drug-induced cholestasis H. CMV infection I. Haemolysis J. Pancreatic cancer K. Wilson’s disease L. Gallstones M. Viral hepatitis
L. Gallstones - This is again a cholestatic pattern and the ultrasound scan means this is either choledocholithiasis, pancreatic cancer or cholangiocarcinoma. The fact there is RUQ pain points more towards gallstones. The other two tend to present painlessly.
Liver function tests are mistakenly named really because they don’t test liver function. They are better called ‘liver tests’ or ‘tests of liver chemistry’. The tests of liver synthetic function come from measuring albumin levels (produced by the liver) and the prothrombin time, as the liver has a key role in the manufacture of clotting factors. Some general points on liver tests to consider: These tests tend to include bilirubin (breakdown product of RBC after hepatic conjugation, and is secreted in the biliary system), AST, ALT, GGT, ALP (alk phos) and LDH. It’s really all about pattern recognition. Isolated elevation of liver tests tends to make you think of non-hepatic causes and you should take into account the patient’s history during interpretation. A normal liver panel does not mean that the person does not have liver disease, and liver tests are elevated in a small percentage of normals.
You can split the possible causes into three broad categories. It is worth noting that people with any pattern can have cirrhosis. AST is also present in heart, skeletal muscle, kidney, brain and in RBCs too. 80% of AST is found in the mitochondria and 20% in the cytoplasmic matrix. ALP, which is a cytoplasmic enzyme, can come from bone, kidney, intestines or the placenta. GGT may come from the heart or RBCs.
The first is predominantly elevated AST/ALT, which is described as a hepatocellular pattern and occurs in conditions such as viral hepatitis, alcoholic disease, Wilson’s etc. Here the hepatocytes get damaged and enzymes leak out.
The second is predominantly elevated ALP (GGT too, but an isolated rise in GGT is common so many hospitals remove GGT from the liver panel. GGT is only useful if there is an isolated rise in ALP). This is seen in a cholestatic (here, the hepatobiliary system is affected) or infiltrative pattern in bile duct obstruction, malignancy e.g. HCC or pancreatic, PBC/PSC etc or infiltrative conditions such as TB and lymphomas.
The last is an isolated hyperbilirubinaemia seen in haemolysis, intra-abdominal bleeding or with conditions such as Gilbert’s. BR elevation can be further divided into conjugated or unconjugated. Unconjugated is normally due to breakdown of RBC beyond the liver’s capacity to conjugate. Conjugated occurs in liver disease and problems with obstructed bile flow.
A 20 year old man: BR 45umol/l, (Conjugated 7, unconjugated 38), ALP 40iu/l, AST 12iu/l, Hb 15g/dl, Normal blood film
A. Alcoholic cirrhosis B. Gilbert’s syndrome C. Acute pancreatitis D. Hepatocellular carcinoma E. Paracetamol overdose F. Cholangiocarcinoma G. Drug-induced cholestasis H. CMV infection I. Haemolysis J. Pancreatic cancer K. Wilson’s disease L. Gallstones M. Viral hepatitis
B. Gilberts syndrome - Gilbert’s occurs in an asymptomatic patient, often as an incidental finding or mild jaundice occuring in adolescence/young adult age. There is elevated unconjugated BR with other liver tests being normal. The blood smear is also normal with normal reticulocyte count, and normal Hb indicating that this is not due to haemolysis. It is a common syndrome and is not really a disease, more a physiological variant. No treatment is needed and this condition is due to decreased UDPGT activity leading to decreased conjugation of unconjugated bilirubin, leading to elevated levels.
Liver function tests are mistakenly named really because they don’t test liver function. They are better called ‘liver tests’ or ‘tests of liver chemistry’. The tests of liver synthetic function come from measuring albumin levels (produced by the liver) and the prothrombin time, as the liver has a key role in the manufacture of clotting factors. Some general points on liver tests to consider: These tests tend to include bilirubin (breakdown product of RBC after hepatic conjugation, and is secreted in the biliary system), AST, ALT, GGT, ALP (alk phos) and LDH. It’s really all about pattern recognition. Isolated elevation of liver tests tends to make you think of non-hepatic causes and you should take into account the patient’s history during interpretation. A normal liver panel does not mean that the person does not have liver disease, and liver tests are elevated in a small percentage of normals.
You can split the possible causes into three broad categories. It is worth noting that people with any pattern can have cirrhosis. AST is also present in heart, skeletal muscle, kidney, brain and in RBCs too. 80% of AST is found in the mitochondria and 20% in the cytoplasmic matrix. ALP, which is a cytoplasmic enzyme, can come from bone, kidney, intestines or the placenta. GGT may come from the heart or RBCs.
The first is predominantly elevated AST/ALT, which is described as a hepatocellular pattern and occurs in conditions such as viral hepatitis, alcoholic disease, Wilson’s etc. Here the hepatocytes get damaged and enzymes leak out.
The second is predominantly elevated ALP (GGT too, but an isolated rise in GGT is common so many hospitals remove GGT from the liver panel. GGT is only useful if there is an isolated rise in ALP). This is seen in a cholestatic (here, the hepatobiliary system is affected) or infiltrative pattern in bile duct obstruction, malignancy e.g. HCC or pancreatic, PBC/PSC etc or infiltrative conditions such as TB and lymphomas.
The last is an isolated hyperbilirubinaemia seen in haemolysis, intra-abdominal bleeding or with conditions such as Gilbert’s. BR elevation can be further divided into conjugated or unconjugated. Unconjugated is normally due to breakdown of RBC beyond the liver’s capacity to conjugate. Conjugated occurs in liver disease and problems with obstructed bile flow.
A 55 year old man presents with a few months history of weight loss, lethargy & fever. On examination, he has a large liver & spleen. His WBC is 10.2x109/l & his blood film shows increased granulocytes & granulocyte precursors.
A. Sarcoidosis B. Polycythaemia C. Gaucher’s disease D. Portal hypertension E. Infectious mononucleosis F. IDA G. Bacterial endocarditis H. Hodgkin’s disease I. Malaria J. Idiopathic myelofibrosis K. Metastatic carcinoma L. CML
L. CML - This is CML which tends to present in the 30-60 age group. At presentation 1/3 may be asymptomatic though if symptomatic, it presents with symptoms including fever, weight loss and night sweats. There is myeloid stem cell proliferation and presents with raised neutrophils, metamyelocytes and basophils. The patient’s granulocytosis is suggestive of CML. CML is associated with the philadelphia chromosome characterised by t(9;22) of bcr-abl. There tends to be massive splenomegaly which is the most common physical finding on examination. This conditon may transform to AML or ALL in what is known as a ‘blast crisis’. CML responds to imatinib, which is an anti-bcr-abl antibody and gives long term remission in most patients.
A 27 year old Afro-Caribbean man presents with fever, weight loss and an intractable itch. His spleen is just palpable and he has two 3cm nodes in his right neck. Hb is low.
A. Sarcoidosis B. Polycythaemia C. Gaucher’s disease D. Portal hypertension E. Infectious mononucleosis F. IDA G. Bacterial endocarditis H. Hodgkin’s disease I. Malaria J. Idiopathic myelofibrosis K. Metastatic carcinoma L. CML
H. Hodgkin’s disease - This is a case of lymphoma. Reed-Sternberg cells are binucleate cells characteristically seen in Hodgkin’s lymphoma. Hodgkin’s is localised to a single group of nodes (normally the cervical and/or supraclavicular) and extranodal involvement is rare. Mediastinal involvement is common. Spread is contiguous and B symptoms may be present such as a low grade fever, weight loss and night sweats. Pruritis may be found in approximately 10% of cases but has no prognostic significance. 50% of cases is associated with EBV infection and distribution is bimodal with peaks in young and old. There is classically pain in lymph nodes on alcohol consumption.
An 18 year old Caucasian shop assistant presents with fever & a sore throat. She is found to have enlarged but soft cervical lymph nodes & a soft spleen palpable 3cm below the costal margin. Blood film shows atypical lymphocytes.
A. Sarcoidosis B. Polycythaemia C. Gaucher’s disease D. Portal hypertension E. Infectious mononucleosis F. IDA G. Bacterial endocarditis H. Hodgkin’s disease I. Malaria J. Idiopathic myelofibrosis K. Metastatic carcinoma L. CML
E. Infectious mononucleosis - This is caused by EBV and characterised by fever, pharyngitis and lymphadenopathy with atypical lymphocytosis. Positive heterophile antibody test and serological testing for EBV antibodies are diagnostic. Splenomegaly is common and enlargement occurs in the first week, lasting 3-4 weeks. It is worth remembering that splenomegaly is always an abnormal examination finding. IM is commonly named the ‘kissing’ disease as EBV is most commonly transmitted by saliva. Penetrative sex and general promiscuity in young women also increases the risk.
A 70 year old English woman presents with fever, slight splenomegaly, splinter haemorrhages & a diastolic murmur at the apex of the heart.
A. Sarcoidosis B. Polycythaemia C. Gaucher’s disease D. Portal hypertension E. Infectious mononucleosis F. IDA G. Bacterial endocarditis H. Hodgkin’s disease I. Malaria J. Idiopathic myelofibrosis K. Metastatic carcinoma L. CML
G. Bacterial endocarditis - Any patient presenting with fever and a new murmur should always make you think of bacterial endocarditis. The classic new or worsening murmur is actually rare. As are splinter haemorrhages, which this patient has. Other uncommon signs you may find include Janeway lesions (painless macular haemorrhagic plaques on the palms and soles) and Osler nodes (painful nodules on the pads of the fingers and toes). Roth spots may also be seen on fundoscopy. Three sets of bood cultures are required and this patient will have to go for an echocardiogram.The Duke criteria is used for diagnosis.
A 65 year old man presents with angina & claudication. He is found to have a firm spleen extending 20cm below the costal margin. His Hb is 7.5g/dl & his blood film is leuco-erythroblastic.
A. Sarcoidosis B. Polycythaemia C. Gaucher’s disease D. Portal hypertension E. Infectious mononucleosis F. IDA G. Bacterial endocarditis H. Hodgkin’s disease I. Malaria J. Idiopathic myelofibrosis K. Metastatic carcinoma L. CML
J. Ideopathic myelofibrosis - This is a case of myelofibrosis. Leucoerythroblastosis and splenomegaly are common findings. Strong risk factors include exposure to radiation and industrial solvents. BM biopsy is essential for diagnosis. Extramedullary haematopoiesis leads to dacrocytes in the peripheral blood smear. Those without symptoms can be managed with folate and pyridoxine supplements. Otherwise options such as a BM transplant and hydroxycarbamide can be considered.
A 35 year old lawyer with a history of recurring piles & having been treated by his GP with sclerosing therapy continues to bleed. He is referred to surgical outpatients for more definitive treatment.
A. Palliative care B. Hemicolectomy C. IV immunoglobulin D. Anterior resection E. Topical GTN F. Haemorrhoidectomy G. Blood transfusion H. High fibre diet I. Colostomy J. Loperamide (Imodium) K. IV corticosteroids
F. Haemorrhoidectomy - Haemorrhoidectomy is the treatment of choice of choice for patients with grade 4 haemorrhoids or for any patient who has failed with more conservative treatment such as sclerotherapy. Grade 1 is limited to within the anal canal. Grade 2 protrudes but spontaneously reduces when the patient stops straining. Grade 3 protrudes and reduces fully on manual pressure. Grade 4 is irreducible. Lord’s anal stretch is no longer recommended due to the risk of faecal incontinence.
A 33 year old man has severe pain and fresh bleeding on defecation.
A. Palliative care B. Hemicolectomy C. IV immunoglobulin D. Anterior resection E. Topical GTN F. Haemorrhoidectomy G. Blood transfusion H. High fibre diet I. Colostomy J. Loperamide (Imodium) K. IV corticosteroids
E. Topical GTN - This patient has an anal fissure. This causes severe pain on defecation and may continue for 1 to 2 hours. A small amount of fresh blood is often passed on the stool. Initial treatment is with topical GTN or diltiazem along with supportive measures such a high fibre diet. Resistant or chronic fissures may benefit from surgical measures or botulinum toxin.
A 39 year old woman who is on mesalazine for her ulcerative colitis presents with 12 daily bowel movements and massive continuous bleeding PR; she is noted to have a raised ESR.
A. Palliative care B. Hemicolectomy C. IV immunoglobulin D. Anterior resection E. Topical GTN F. Haemorrhoidectomy G. Blood transfusion H. High fibre diet I. Colostomy J. Loperamide (Imodium) K. IV corticosteroids
K. IV corticosteroid - IV corticosteroids are used in fulminant disease. These patients need to be admitted. If there is no response to IV corticosteroids within 24 to 48 hours then surgery is indicated.
A 24 year old woman following a viral infection was diagnosed as having idiopathic thrombocytopaenia. She presents to A&E & complains of multiple bruising & rectal bleeding. She is on oral prednisolone 30mg/day. Her Hb is 12.5g/dl.
A. Palliative care B. Hemicolectomy C. IV immunoglobulin D. Anterior resection E. Topical GTN F. Haemorrhoidectomy G. Blood transfusion H. High fibre diet I. Colostomy J. Loperamide (Imodium) K. IV corticosteroids
C. IV immunoglobulin - ITP is thought to be due to an autoimmune phenomenon. Treatment is based on the patient’s platelet count and bleeding symptoms. This patient has severe active bleeding and must be started on IVIG plus corticosteroids, which she is already on. Platelet transfusions should be considered with tranexamic acid as an adjunct.
A 58 year old man was admitted complaining of abdominal pain. He is found to have rectal carcinoma.
A. Palliative care B. Hemicolectomy C. IV immunoglobulin D. Anterior resection E. Topical GTN F. Haemorrhoidectomy G. Blood transfusion H. High fibre diet I. Colostomy J. Loperamide (Imodium) K. IV corticosteroids
D. Anterior resection - Treatment of rectal carcinoma involves surgical excision where possible. This can either be an anterior resection (tumours in the upper 1/3 of the rectum) or an abdominoperineal resection (if the tumour lies lower down). APER involves the formation of a permanent colostomy and has a high incidence of sexual and urinary dysfunction. Anterior resection involves a colo-anal anastamosis.
A 64 year old man presents with a 1 day history of dark rectal bleeding. He has complained of some diarrhoea for the last 6 months, & has noticed some loss of weight. Rectal examination & proctoscopy are normal.
A. Infective colitis B. Haemorrhoids C. Anal fissure D. Colonic carcinoma E. Anal carcinoma F. Crohn’s disease G. Ulcerative colitis H. Colonic polyp I. Diverticular disease J. Ischaemic colitis
D. Colonic carcinoma - The symptoms of colorectal cancer are non-specific but the GI symptoms here combined with weight loss should make you suspicious. Symptoms include change in bowel habit, rectal bleeding and anaemia, commonly associated with right sided cancer. Distension, weight loss and vomiting are usually associated with advanced disease. Colonoscopy will be needed in this patient for diagnosis and a biopsy of any suspicious lesion found. Main curative treatment is surgical resection. CEA is the classic tumour marker, which is raised in about 80% but is not really sensitive or specific enough to be useful in diagnosis or screening, but is used to monitor treatment and recurrence in those who have already been diagnosed.
A 24 year old woman presents with a 3 month history of lower abdominal colicky pain, diarrhoea (bowels open 6-10 times per day) & passage of blood mixed with the stool.
A. Infective colitis B. Haemorrhoids C. Anal fissure D. Colonic carcinoma E. Anal carcinoma F. Crohn’s disease G. Ulcerative colitis H. Colonic polyp I. Diverticular disease J. Ischaemic colitis
G. Ulcerative collitis - While this could be Crohn’s disease, bloody diarrhoea is more commonly a presentation of UC than Crohn’s. UC is characterised by diffuse mucosal inflammation running a relapsing and remitting course. Bloody diarrhoea is commonly experienced by patients who may also complain of other symptoms such as (lower) abdominal pain, faecal urgency and the host of extra-intestinal manifestations associated with UC. Diagnosis of UC requires endoscopy with biopsy and a negative stool culture to rule out infectious gastroenteritis. Flare ups are usually linked to pathogens so a stool culture will always be needed in these cases. Toxic megacolon is a complication which is associated with a risk of perforation. UC is also linked with bowel adenocarinoma and PSC. Treatment involves mesalazine (5-ASA) used to induce and maintain remission.
A 54 year old man with no previous abdominal symptoms complains of several episodes of painless bright red rectal bleeding which is separate from the stool. Abdominal, rectal examination & proctoscopy are normal.
A. Infective colitis B. Haemorrhoids C. Anal fissure D. Colonic carcinoma E. Anal carcinoma F. Crohn’s disease G. Ulcerative colitis H. Colonic polyp I. Diverticular disease J. Ischaemic colitis
H. Colonic polyp - Most polyps are asymptomatic and rectal bleeding is indeed an unusual symptom to present with (though FOBT may be positive), but the lack of previous symptoms and normal examination findings makes this likely to be a bleeding polyp. The next step here would be endoscopy with polypectomy for histological examination. Indeed they are frequently found incidentally on screening with FOBT or colonoscopy for another reason. If there are a few polyps, all of them will be removed. If there are many, a sample will be removed for biopsy. Polyps can also be inherited in FAP, also seen in Gardner syndrome and Peutz-Jeghers. They can be neoplastic such as adenomas or non-neoplastic such as hyperplastic polyps.
A 24 year old man presents with a 3 month history of episodes of painless, bright red rectal bleeding on straining at stool. He has noticed some blood in the bowl, separate from the stool & some on the paper after wiping.
A. Infective colitis B. Haemorrhoids C. Anal fissure D. Colonic carcinoma E. Anal carcinoma F. Crohn’s disease G. Ulcerative colitis H. Colonic polyp I. Diverticular disease J. Ischaemic colitis
B Haemorrhoids - Haemorrhoids are vascular rich cushions in the anal canal and presents, typically, as painless bright PR bleeding or with sudden onset pain in the area associated with a palpable mass. Pruritus ani is common and there is often perianal pain or discomfort. Diagnosis is made visually. Grade 1 is limited to within the anal canal. Grade 2 protrudes but spontaneously reduces when the patient stops straining. Grade 3 protrudes and reduces fully on manual pressure. Grade 4 is irreducible. Treatment includes fibre, ligation, photocoagulation, sclerotherapy or surgical haemorrhoidectomy. Haemorrhoidectomy is the treatment of choice of choice for patients with grade 4 haemorrhoids or for any patient who has failed with more conservative treatment such as sclerotherapy.
A 28 year old female medical student returns from her elective in Africa with a short history of severe lower abdominal cramps & the passage of bloody diarrhoea.
A. Infective colitis B. Haemorrhoids C. Anal fissure D. Colonic carcinoma E. Anal carcinoma F. Crohn’s disease G. Ulcerative colitis H. Colonic polyp I. Diverticular disease J. Ischaemic colitis
A Infective collitis - This patient has recent foreign travel and a short history, previously normal, making infectious diarrhoea very likely. Probably eaten something dodgy while on elective. Think of the organisms which cause bloody diarrhoea such as EHEC. E coli is the most common cause of traveller’s diarrhoea. There may be a contact history due to faecal-oral spread. The mainstay of treatment is rehydration and supportive therapy. Antibiotics may be indicated, particularly in severe ETEC.
A 40 year old lady with multiple painful lumps in her breast, which are painful & tender premenstrually.
A. Sebaceous cyst B. Fibroadenoma C. Fibroadenosis D. Gynaecomastia E. Breast abscess F. Carcinoma of the breast G. Breast cyst H. Lipoma I. Duct ectasia
C. Fibroadenosis - Fibrocystic breasts are characterised by ‘lumpy’ breasts associated with pain which fluctuates with the menstrual cycle (it is worse during the luteal phase of menses). Risk factors include obesity, nulliparity, HRT and late onset menopause and first childbirth. It is a diagnosis of exclusion, and is considered to be an exaggerated physiological phenomenon rather than a disease (54% of clinically normal breasts are found on autopsy to have fibrocystic changes). Symptoms typically arise between the 3rd and 4th decases of life. There may also be a nipple discharge, which can be suspicious if bloody or profuse etc and may indicate the presence of an intraductal papilloma, cancer, or duct ectasia. Cysts can be aspirated if symptomatic (asymptomatic or small ones do not require intervention). If the aspirate is straw coloured and completely aspirated, there is no need for cytology, but if the aspirate is bloody, cytology or biopsy is needed to exclude cancer. There is improvement of mastalgia and cysts at menopause and until then it runs a chronic relapsing course.
A 25 year old lady with a discrete, non-tender, mobile lump in one breast
A. Sebaceous cyst B. Fibroadenoma C. Fibroadenosis D. Gynaecomastia E. Breast abscess F. Carcinoma of the breast G. Breast cyst H. Lipoma I. Duct ectasia
B. Fibroadenoma - This is a fibroadenoma which tends to be asymptomatic and found incidentally, typically in a patient
A 35 year old lady is generally unwell with a tachycardia & a fever. A segment of the right breast is painful, tender, red & warm.
A. Sebaceous cyst B. Fibroadenoma C. Fibroadenosis D. Gynaecomastia E. Breast abscess F. Carcinoma of the breast G. Breast cyst H. Lipoma I. Duct ectasia
E. Breast abscess - Breast abscess presents with mastalgia and fever. Breast infection typically affects women who are lactating and the most commonly implicated pathogen is staphylococcus aureus. The painful, hard and red lump indicates the development of an abscess. Antibiotic therapy is indicated with surgical intervention such as aspiration and drainage with possible duct excision. Prompt management of mastitis when it presents will usually lead to a good timely resolution and prevent the development of complications such as an abscess. An USS can help to identify the underlying abscess which usually forms a hypoechoic lesion. Needle aspiration can be used both therapeutically and diagnostically and can be guided by ultrasound.
A 14 year old boy with bilateral breast enlargement.
A. Sebaceous cyst B. Fibroadenoma C. Fibroadenosis D. Gynaecomastia E. Breast abscess F. Carcinoma of the breast G. Breast cyst H. Lipoma I. Duct ectasia
D. Gynaecomasatia - This is a boy who has enlarged breasts. Normal to see gynaecomastia in puberty. Other causes include liver disease and as a side effect of drugs such as digoxin, spironolactone and cimetidine.
A 40 year old lady with a green nipple discharge & tender lumpiness beneath the areola.
A. Sebaceous cyst B. Fibroadenoma C. Fibroadenosis D. Gynaecomastia E. Breast abscess F. Carcinoma of the breast G. Breast cyst H. Lipoma I. Duct ectasia
I. Duct ectasia - Duct ectasia happens because the lactiferous duct gets blocked. Ectasia means widening. It can mimic breast cancer as the discharge can be bloody sometimes and signs can include nipple inversion. The green nipple discharge is typical in EMQs. It is a self limiting condition.
A 40 year old lady with a hard lump in the right breast. The skin overlying the lump has an orange peel appearance.
A. Sebaceous cyst B. Fibroadenoma C. Fibroadenosis D. Gynaecomastia E. Breast abscess F. Carcinoma of the breast G. Breast cyst H. Lipoma I. Duct ectasia
F. Carcinoma of the breast - Patient’s with breast cancer, on examination (familiarise yourself with how to conduct a breast exam), tend to demonstrate a firm hard lump which may be associated with axillary lymphadenopathy, skin changes such as the orange peel (peau d’orange) and nipple changes/discharge. The skin changes here are most likely associated with locally advanced cancer. Many breast cancers are also diagnosed on routine mammography which can show microcalcifications, in the absence of a palpable mass. MRI is more sensitive but less specific so is recommended only in patients who are at high risk, such as BRCA1/2 mutation, history of chest radiation or certain syndromes like Cowden’s or Li-Fraumeni. FNA is also useful in rapid diagnosis, although is operator dependent when it comes to how sensitive and specific it is, and a core biopsy is preferred in most cases for diagnosis as it can differentiate pre-invasive and invasive disease. Treatment is MDT involving surgeons, oncologists, radiation oncologists etc. Do you know about the current NHS breast screening programme?
A 48 year old women presents with mild breast pain which improves in the days after her menstrual period. Her breasts are lumpy on examination.
A. Fibrocystic changes B. Breast cancer C. Necrotising fasciitis D. Galactocoele E. Costochondritis F. Phylloides tumour G. Diabetic breast lesion H. Mondor’s disease I. Raynaud’s phenomenon J. Fibroadenoma K. Breast abscess L. Fat necrosis
A. Fibrocystic changes - Fibrocystic breasts are characterised by ‘lumpy’ breasts associated with pain which fluctuates with the menstrual cycle (it is worse during the luteal phase of menses). Risk factors include obesity, nulliparity, HRT and late onset menopause and first childbirth. It is a diagnosis of exclusion, and is considered to be an exaggerated physiological phenomenon rather than a disease (54% of clinically normal breasts are found on autopsy to have fibrocystic changes). Symptoms typically arise between the 3rd and 4th decases of life. There may also be a nipple discharge, which can be suspicious if bloody or profuse etc and may indicate the presence of an intraductal papilloma, cancer, or duct ectasia. Cysts can be aspirated if symptomatic (asymptomatic or small ones do not require intervention). If the aspirate is straw coloured and completely aspirated, there is no need for cytology, but if the aspirate is bloody, cytology or biopsy is needed to exclude cancer. There is improvement of mastalgia and cysts at menopause and until then it runs a chronic relapsing course.
A 17 year old school girl presented with breast pain , changed from dull ache to severe continuous throbbing pain. The breast feels hot on palpation and she complains of hot flushes. On examination the whole breast is tender and engorged and nipple is tender.
A. Fibrocystic changes B. Breast cancer C. Necrotising fasciitis D. Galactocoele E. Costochondritis F. Phylloides tumour G. Diabetic breast lesion H. Mondor’s disease I. Raynaud’s phenomenon J. Fibroadenoma K. Breast abscess L. Fat necrosis
K. Breast abscess - Breast abscess presents with mastalgia and fever. Breast infection typically affects women who are lactating and the most commonly implicated pathogen is staphylococcus aureus. The painful, hot, engorged and red breast suggests the possible development of an abscess. Antibiotic therapy is indicated with surgical intervention such as aspiration and drainage with possible duct excision. Prompt management of mastitis when it presents will usually lead to a good timely resolution and prevent the development of complications such as an abscess. An USS can help to identify the underlying abscess which usually forms a hypoechoic lesion. Needle aspiration can be used both therapeutically and diagnostically and can be guided by ultrasound.
A 20 year old woman presented with a painless lump. On examination there was a 5cm smooth bosselated firm mass, which is highly mobile.
A. Fibrocystic changes B. Breast cancer C. Necrotising fasciitis D. Galactocoele E. Costochondritis F. Phylloides tumour G. Diabetic breast lesion H. Mondor’s disease I. Raynaud’s phenomenon J. Fibroadenoma K. Breast abscess L. Fat necrosis
J. Fibroadenoma - This is a fibroadenoma which tends to be asymptomatic and found incidentally, typically in a patient
A 40 year old female presented with a lump in the breast, enlargement of breasts, no pain. There is a history of the lump fluctuating in size. On examination the lump is found in the upper outer quadrant, moderately hard and mobile. It lacks skin or deep attachment.
A. Fibrocystic changes B. Breast cancer C. Necrotising fasciitis D. Galactocoele E. Costochondritis F. Phylloides tumour G. Diabetic breast lesion H. Mondor’s disease I. Raynaud’s phenomenon J. Fibroadenoma K. Breast abscess L. Fat necrosis
B. Breast cancer - There is a painless lump here which fluctuates in size. Breast cancer progressively increases in size regardless of the menstrual cycle, unlike fibrocystic breasts which may vary in size with the menstrual cycle. When examining the breasts, they should first be examined sat upright to inspect for any changes in skin colour, any dimpling and asymmetry. The axilla, supra and infra-clavicular nodes should also be checked for any nodal metastases. Then, with the patient supine (and arm behind the head), the breast tissue should be felt and the tissue at the beneath the nipple.
Many breast cancers are also diagnosed on routine mammography which can show microcalcifications, in the absence of a palpable mass. MRI is more sensitive but less specific so is recommended only in patients who are at high risk, such as BRCA1/2 mutation, history of chest radiation or certain syndromes like Cowden’s or Li-Fraumeni. The estimated cumulative lifetime incidence of breast cancer in those with BRCA mutations is 87%. FNA is also useful in rapid diagnosis, although is operator dependent when it comes to how sensitive and specific it is, and a core biopsy is preferred in most cases for diagnosis as it can differentiate pre-invasive and invasive disease. Treatment is MDT involving surgeons, oncologists, radiation oncologists etc. Treatment with less than a total mastectomy will require radiotherapy to the remaining breast tissue afterwards.
A 47 year old gentleman presents with bilateral breast enlargement. His PMH includes dyspepsia that is treated with Cimetidine.
A. Benign cyst B. SLE C. Breast abcess D. Fat necrosis E. Fibrocystic breasts F. Pagets disease G. Duct ectasia H. Fibroadenoma I. Mastalgia J. Breast carcinoma K. Gynaecomastia
K. Gynaecomastia - This is a man who has presented with breast enlargement i.e. gynaecomastia. The main cause is either chronic liver disease or drug induced. Drugs include the H2 receptor antagonist cimetidine, as well as digoxin and spironolactone. Gynaecomastia is benign in 99% of cases and results from relative excess of oestrogen or lack of testosterone. Oestrogenisation can follow from anabolic steroid use/abuse. This can hence occur physiologically during the newborn period, puberty and with age and obesity (not true gynaecomastia). Treatment can be offered if there is pain or embarassment including anti-oestrogens such as tamoxifen, androgens or surgical options like liposuction. The drugs that induce gynaecomastia can be subdivided into those that reduce testosterone synthesis (GnRH agonists, cancer drugs, ketoconazole, metronidazole, spironolactone), those that impair the action of testosterone (spironolactone again, finasteride, H2 blockers, PPIs) and those that act via oestrogen (digoxin, PHT, anabolic steroids).
A 50 year old extremely obese lady presents with a tender, hard 3cm lump in the left breast which has appeared quite rapidly. She remembers knocking the breast against a table edge 1 month ago.
A. Benign cyst B. SLE C. Breast abcess D. Fat necrosis E. Fibrocystic breasts F. Pagets disease G. Duct ectasia H. Fibroadenoma I. Mastalgia J. Breast carcinoma K. Gynaecomastia
D. Fat necrosis - Prior breast trauma or breast surgery such as augmentation can lead to fat necrosis. Examination will reveal a firm mass with irregular borders which can resemble malignancy. Biopsy will reveal fat necrosis. USS and mammography shows indistinct margins, sometimes with calcifications which may again resemble findings seen in carcinoma.
A 19 year old girl has noticed a lump in the right breast. It is smooth, 2cm in diameter, non tender and highly mobile.
A. Benign cyst B. SLE C. Breast abcess D. Fat necrosis E. Fibrocystic breasts F. Pagets disease G. Duct ectasia H. Fibroadenoma I. Mastalgia J. Breast carcinoma K. Gynaecomastia
H. Fibroadenoma - Fibroadenomas are typically asymptomatic and are found incidentally in patients
A 30 year old lady, who is breast feeding, has developed an extremely painful, hard, red 4 cm lump at the edge of the nipple.
A. Benign cyst B. SLE C. Breast abcess D. Fat necrosis E. Fibrocystic breasts F. Pagets disease G. Duct ectasia H. Fibroadenoma I. Mastalgia J. Breast carcinoma K. Gynaecomastia
C. Breast abscess - Breast abscess presents with mastalgia and fever. Breast infection typically affects women who are lactating and the most commonly implicated pathogen is staphylococcus aureus. The painful, hard and red lump indicates the development of an abscess. Antibiotic therapy is indicated with surgical intervention such as aspiration and drainage with possible duct excision. Prompt management of mastitis when it presents will usually lead to a good timely resolution and prevent the development of complications such as an abscess. An USS can help to identify the underlying abscess which usually forms a hypoechoic lesion. Needle aspiration can be used both therapeutically and diagnostically and can be guided by ultrasound.
A 65 year old lady has noticed a 3cm lump behind the right nipple. It is hard, non-tender, mobile and has an irregular edge.
A. Benign cyst B. SLE C. Breast abcess D. Fat necrosis E. Fibrocystic breasts F. Pagets disease G. Duct ectasia H. Fibroadenoma I. Mastalgia J. Breast carcinoma K. Gynaecomastia
J. Breast carcinoma - This lady’s advanced age and the fact the lump is hard, irregular and non-tender all point towards malignancy. The mass is typically fixed but can be mobile in early stages. There may be associated changes such as nipple inversion, peau d’orange, lymphadenopathy and skin retraction. Skin changes are always associated with locally advanced cancer. Triple assessment is indicated with a mammogram and biopsy. Risk factors include FH, prolonged oestrogen exposure and high levels of alcohol consumption.
29 y/o lady presents 3 weeks post partum with pain and a lump in her left breast. O/E you notice swelling, redness, the area feels hot.
A. Lipoma B. Paget's disease C. Acute pyogenic mastitis D. Duct ectasia E. Fibrocystic disease F. Sarcoma G. Radial scar H. Adenoma I. Fibroadenoma J. Breast cancer K. Intraductal papilloma
C. Acute pyogenic mastitis - Breast infection typically affects women who are lactating and the most commonly implicated pathogen is staphylococcus aureus. The painful lump indicates the development of an abscess. Antibiotic therapy is indicated with surgical intervention such as aspiration and drainage with possible duct excision. Prompt management of mastitis when it presents will usually lead to a good timely resolution and prevent the development of complications such as an abscess. An USS can help to identify the underlying abscess which usually forms a hypoechoic lesion. Needle aspiration can be used both therapeutically and diagnostically and can be guided by ultrasound.
33 y/o lady complains of tenderness in her right breast typically in the second half of the menstrual cycle. O/E breast feels lumpy.
A. Lipoma B. Paget's disease C. Acute pyogenic mastitis D. Duct ectasia E. Fibrocystic disease F. Sarcoma G. Radial scar H. Adenoma I. Fibroadenoma J. Breast cancer K. Intraductal papilloma
E. Fibrocysitic - Fibrocystic breasts are characterised by ‘lumpy’ breasts associated with pain which fluctuates with the menstrual cycle (it is worse during the luteal phase of menses). Risk factors include obesity, nulliparity, HRT and late onset menopause and first childbirth. It is a diagnosis of exclusion, and is considered to be an exaggerated physiological phenomenon rather than a disease (54% of clinically normal breasts are found on autopsy to have fibrocystic changes). Symptoms typically arise between the 3rd and 4th decases of life. There may also be a nipple discharge, which can be suspicious if bloody or profuse etc and may indicate the presence of an intraductal papilloma, cancer, or duct ectasia. Cysts can be aspirated if symptomatic (asymptomatic or small ones do not require intervention). If the aspirate is straw coloured and completely aspirated, there is no need for cytology, but if the aspirate is bloody, cytology or biopsy is needed to exclude cancer. There is improvement of mastalgia and cysts at menopause and until then it runs a chronic relapsing course.
63 y/o lady presents with a hard mass in upper outer quadrant of left breast, fixed to underlying structures. There is skin dimpling and nipple retraction. Family history for breast cancer is negative.
A. Lipoma B. Paget's disease C. Acute pyogenic mastitis D. Duct ectasia E. Fibrocystic disease F. Sarcoma G. Radial scar H. Adenoma I. Fibroadenoma J. Breast cancer K. Intraductal papilloma
J. Breast cancer - This lady’s advanced age and the fact the lump is hard and fixed all point towards malignancy. There may be associated changes like in this patient, ranging from nipple inversion, peau d’orange, dimpling and lymphadenopathy to skin retraction. Skin changes are always associated with locally advanced cancer. Triple assessment is indicated with a mammogram and biopsy. Risk factors include FH, prolonged oestrogen exposure and high levels of alcohol consumption.
Cytology of nipple discharge shows inflammatory cells with no evidence of epithelial cells. Discharge is very thick and cheesy.
A. Lipoma B. Paget's disease C. Acute pyogenic mastitis D. Duct ectasia E. Fibrocystic disease F. Sarcoma G. Radial scar H. Adenoma I. Fibroadenoma J. Breast cancer K. Intraductal papilloma
D. Duct ectasia - Duct ectasia happens because the lactiferous duct gets blocked. Ectasia means widening. It can mimic breast cancer as the discharge can be bloody sometimes and signs can include nipple inversion. It is a self limiting condition. The cytology of inflammatory cells confirms this diagnosis as duct ectasia is where the duct is dilated and is associated with inflammation.
A 55 year old woman presented with recent onset and rapid enlargement of the breast. On palpation, there is a large breast mass. Biopsy showed stromal and epithelial elements.
A. Intraductal papilloma B. Lobular carcinoma in situ C. Gynaecomastia D. Phylloides tumour E. Pregnancy F. Breast Abscess G. Fibroadenoma H. Mastalgia I. Nodularity J. Invasive breast cancer K.Adenoma
D. Phylloides tumour - A phylloides tumour is a fast growing fibroepithelial mass, as demonstrated here by the history and biopsy findings. It can be benign or malignant. The only cure is surgical, commonly a WLE.
A 17 year old school girl presents to her GP with pain in her breast. This has changed from dull ache to severe continuous throbbing pain. The breast feels hot on palpation and she complains of hot flushes. On examination, there is a red, warm and fluctuant breast mass.
A. Intraductal papilloma B. Lobular carcinoma in situ C. Gynaecomastia D. Phylloides tumour E. Pregnancy F. Breast Abscess G. Fibroadenoma H. Mastalgia I. Nodularity J. Invasive breast cancer K.Adenoma
F. Breast abscess - Breast abscess presents with mastalgia and fever. Breast infection typically affects women who are lactating and the most commonly implicated pathogen is staphylococcus aureus. The painful, hard and red lump indicates the development of an abscess. Antibiotic therapy is indicated with surgical intervention such as aspiration and drainage with possible duct excision. Prompt management of mastitis when it presents will usually lead to a good timely resolution and prevent the development of complications such as an abscess. An USS can help to identify the underlying abscess which usually forms a hypoechoic lesion. Needle aspiration can be used both therapeutically and diagnostically and can be guided by ultrasound.
A 25 year old woman finds a smooth, rubbery and mobile mass on her left breast. She is overwise asymptomatic.
A. Intraductal papilloma B. Lobular carcinoma in situ C. Gynaecomastia D. Phylloides tumour E. Pregnancy F. Breast Abscess G. Fibroadenoma H. Mastalgia I. Nodularity J. Invasive breast cancer K. Adenoma
G. Fibroadenoma - Fibroadenomas are typically asymptomatic and are found incidentally in patients
A 40 year old lady with a hard lump in the right breast. The skin overlying the lump has an orange peel appearance.
A. Intraductal papilloma B. Lobular carcinoma in situ C. Gynaecomastia D. Phylloides tumour E. Pregnancy F. Breast Abscess G. Fibroadenoma H. Mastalgia I. Nodularity J. Invasive breast cancer K. Adenoma
J. Invasive breast cancer - Patient’s wth breast cancer, on examination (familiarise yourself with how to conduct a breast exam), tend to demonstrate a firm hard lump which may be associated with axillary lymphadenopathy, skin changes such as the orange peel (peau d’orange) and nipple changes/discharge. The skin changes here are most likely associated with locally advanced cancer. Many breast cancers are also diagnosed on routine mammography which can show microcalcifications, in the absence of a palpable mass. MRI is more sensitive but less specific so is recommended only in patients who are at high risk, such as BRCA1/2 mutation, history of chest radiation or certain syndromes like Cowden’s or Li-Fraumeni. FNA is also useful in rapid diagnosis, although is operator dependent when it comes to how sensitive and specific it is, and a core biopsy is preferred in most cases for diagnosis as it can differentiate pre-invasive and invasive disease. Treatment is MDT involving surgeons, oncologists, radiation oncologists etc. Do you know about the current NHS breast screening programme?
A 50 year old man with a painless enlargement of the left breast. He denies finding any definite lumps
A. Intraductal papilloma B. Lobular carcinoma in situ C. Gynaecomastia D. Phylloides tumour E. Pregnancy F. Breast Abscess G. Fibroadenoma H. Mastalgia I. Nodularity J. Invasive breast cancer K. Adenoma
C. Gynaecomastia - This is a man who has presented with breast enlargement i.e. gynaecomastia. The main cause is either chronic liver disease or drug induced. Drugs include the H2 receptor antagonist cimetidine, as well as digoxin and spironolactone. Gynaecomastia is benign in 99% of cases and results from relative excess of oestrogen or lack of testosterone. Oestrogenisation can follow from anabolic steroid use/abuse. This can hence occur physiologically during the newborn period, puberty and with age and obesity (not true gynaecomastia). Treatment can be offered if there is pain or embarassment including anti-oestrogens such as tamoxifen, androgens or surgical options like liposuction.
The drugs that induce gynaecomastia can be subdivided into those that reduce testosterone synthesis (GnRH agonists, cancer drugs, ketoconazole, metronidazole, spironolactone), those that impair the action of testosterone (spironolactone again, finasteride, H2 blockers, PPIs) and those that act via oestrogen (digoxin, PHT, anabolic steroids).